Fundamentals of Nursing Book End of Chapter Questions

Ace your homework & exams now with Quizwiz!

A nurse assesses a patient and formulates the following nursing diagnosis: Risk for Impaired Skin Integrity related to prescribed bedrest as evidenced by reddened areas of skin on the heels and back. Which phrase represents the etiology of this diagnostic statement? a. Risk for Impaired Skin Integrity b. Related to prescribed bedrest c. As evidenced by d. As evidenced by reddened areas of skin on the heels and back

b."Related to prescribed bedrest" is the etiology of the state- ment. The etiology identifies the contributing or causative factors of the problem. "Risk for Impaired Skin Integrity" is the problem, and "as evidenced by reddened areas of skin on the heels and back" are the defining characteristics of the problem.

A nurse is using personal protective equipment (PPE) when bathing a patient diagnosed with C. difficile infection. Which nursing action related to this activity promotes safe, effective patient care? a. The nurse puts on PPE after entering the patient room. b. The nurse works from "clean" areas to "dirty" areas during bath. c. The nurse personalizes the care by substituting glasses for goggles. d. The nurse removes PPE prior to leaving the patient room.

b. When using PPE, the nurse should work from "clean" areas to "dirty" ones, put on PPE before entering the patient room, always use goggles instead of personal glasses, and remove PPE in the doorway or anteroom.

An emergency department nurse is using a manual resuscita- tion bag (Ambu bag) to assist ventilation in a patient with lung cancer who has stopped breathing on his own. What is an appropriate step in this procedure? a. Tilt the patient's head forward. b. Hold the mask tightly over the patient's nose and mouth. c. Pull the patient's jaw backward. d. Compress the bag twice the normal respiratory rate for the patient.

b. With the patient's head tilted back, jaw pulled forward, and airway cleared, the mask is held tightly over the patient's nose and mouth. The bag also fits easily over tracheostomy and endotracheal tubes. The operator's other hand compresses the bag at a rate that approximates normal respiratory rate (e.g., 16-20 breaths/min in adults).

A nurse is documenting the care given to a 56-year-old patient diagnosed with an osteosarcoma, whose right leg was amputated. The nurse accidentally documents that a dressing changed was performed on the left leg. What would be the best action of the nurse to correct this documentation? a. Erase or use correcting fluid to completely delete the error. b. Draw a single line through the entry and rewrite it above or beside it. c. Use a permanent marker to block out the mistaken entry and rewrite it. d. Remove the page with the error and rewrite the data on that page correctly.

b. The nurse should not use dittos, erasures, or correcting flu- ids. A single line should be drawn through an incorrect entry, and the words "mistaken entry" or "error in charting" should be printed above or beside the entry and signed. The entry should then be rewritten correctly.

A nurse is providing instruction to a patient regarding the procedure to change his colostomy bag. During the teach- ing session, he asks, "What type of foods should I avoid to prevent gas?" The patient's question allows for what type of communication? a. A closed-ended answer b. Information clarification c. The nurse to give advice d. Assertive behavior

b. The patient's question allows the nurse to clarify informa- tion that is new to the patient or that requires further explana- tion.

A sophomore in high school has missed a lot of school this year because of leukemia. He said he feels like he is falling behind in everything and misses "hanging out at the mall" with his friends most of all. a. Personal Identity Disturbance b. Body Image Disturbance c. Self-Esteem Disturbance d. Altered Role Performance

d. Important roles for this patient are being a student and a friend. His illness is preventing him from doing either of these well. This self-concept disturbance is basically one that concerns role performanc

Data must be collected to evaluate the effectiveness of a plan to reduce urinary incontinence in an older adult patient. Which information is least important for the evaluation process? a. The incontinence pattern b. State of physical mobility c. Medications being taken d. Age of the patient

d. Incontinence is not a natural consequence of the aging process. All the other factors are necessary information for the plan of care.

A nurse orients an older patient to the safety features in her hospital room. What is a priority component of this admission routine? a. Explain how to use the telephone. b. Introduce the patient to her roommate. c. Review the hospital policy on visiting hours. d. Explain how to operate the call bell.

d. Knowing how to use the call bell is a safety priority; knowing how to use the phone, meeting the roommate, and knowledge of visiting hours will not necessarily prevent an accidental injury.

A patient is postoperative following an emergency cesarean section birth. The patient asks the nurse about the use of pain medications following surgery. What would be a correct response by the nurse? a. "It's not a good idea to ask for pain medication regularly as it can be addictive." b. "It is better to wait until the pain gets unbearable before asking for pain medication." c. "It's natural to have to put up with pain after surgery and it will lessen in intensity in a few days." d. "Your doctor has ordered pain medications for you, which you should not be afraid to request any time you have pain."

d. Many pain medications are ordered on a PRN (as needed) basis. Therefore, nurses must be diligent to assess patients for pain and administer medications as needed. A patient should not be afraid to request these medications and should not wait until the pain is unbearable. Few people become addicted to the medications if used for a short period of time. Pain fol- lowing surgery can be controlled and should not be consid- ered a natural part of the experience that will lessen in time.

A patient has an order for an analgesic medication to be given PRN. When would the nurse administer this medication? a. Every three hours b. Every four hours c. Daily d. As needed

d. PRN means "as needed."

When you receive the shift report, you learn that your patient has no special skin care needs. You are surprised during the bath to observe reddened areas over bony prominences. What action is appropriate? a. Correct the initial assessment form. b. Redo the initial assessment and document current findings. c. Conduct and document an emergency assessment. d. Perform and document a focused assessment of skin integrity

d. Perform and document a focused assessment on skin integrity since this is a newly identified problem. The initial assessment stands as is and cannot be redone or corrected. This is not a life-threatening event; therefore, there is no need for an emergency assessment.

The nurse practitioner is performing a short assessment of a newborn who is displaying signs of jaundice. The nurse observes the infant's skin color and orders a test for bilirubin levels to report to the primary care provider. What type of assessment has this nurse performed? a. Comprehensive b. Initial c. Time-lapsed d. Quick priority

d. Quick priority assessments (QPA) are short, focused, prioritized assessments nurses do to gain the most important information they need to have first. The comprehensive initial assessment is performed shortly after the patient is admitted to a health care agency or service. The time-lapsed assess- ment is scheduled to compare a patient's current status to baseline data obtained earlier.

A female patient who is having a myocardial infarction com- plains of pain that is situated in her jaw. The nurse documents this as what type of pain? a. Transient pain b. Superficial pain c. Phantom pain d. Referred pain

d. Referred pain is perceived in an area distant from its point of origin, whereas transient pain is brief and passes quickly. Superficial pain originates in the skin or subcutaneous tissue. Phantom pain may occur in a person who has had a body part amputated, either surgically or traumatically.

A 76-year-old patient states, "I have been experiencing com- plications of diabetes." The nurse needs to direct the patient to gain more information. What is the most appropriate com- ment or question to elicit additional information? a. "Do you take two injections of insulin to decrease the complications?" b. "Most physicians recommend diet and exercise to regulate blood sugar." c. "Most complications of diabetes are related to neuropathy." d. "What specific complications have you experienced?"

d. Requesting specific information regarding complications of diabetes will elicit specific information to guide the nurse in further interview questions and specific assessment techniques.

A student health nurse is counseling a female college student who wants to lose 20 pounds. The nurse develops a plan to increase the student's activity level and decrease the con- sumption of the wrong types of foods and excess calories. When the student arrives for her first "weigh-in," the nurse discovers that instead of the projected weight loss of 5 pounds, the student has lost only 1 pound. Which is the best nursing response? a. Congratulate the student and continue the plan of care. b. Terminate the plan of care since it is not working. c. Try giving the student more time to reach the targeted outcome. d. Modify the plan of care after discussing possible reasons for the student's partial success.

d. Since the student has only partially met her outcome, the nurse should first explore the factors making it difficult for her to reach her outcome and then modify the plan of care. It would not be appropriate to continue the plan as it is since it is not working, and it is premature to terminate the plan of care since the student has not met her targeted outcome. The student may need more than time to reach her outcome, which makes (c) the wrong response.

What consideration should the nurse keep in mind regarding the use of side rails for a confused patient? a. They prevent confused patients from wandering. b. A history of a previous fall from a bed with raised side rails is insignificant. c. Alternative measures are ineffective to prevent wandering. d. A person of small stature is at increased risk for injury from entrapment.

d. Studies of restraint-related deaths have shown that people of small stature are more likely to slip through or between the side rails. The desire to prevent a patient from wandering is not sufficient reason for the use of side rails. Creative use of alternative measures indicates respect for the patient's dignity and may in fact prevent more serious fall-related injuries. A history of falls from a bed with raised side rails carries a significant risk for a future serious incident.

A nurse is formulating a clinical question in PICO format. What does the letter P represent? a. Comparison to another similar treatment b. Clearly defined, focused literature review c. Specific identification of the desired outcome d. Explicit descriptions of the population of interest

d. The P in the PICO format represents an explicit descrip- tion of the patient population of interest. I represents the intervention, C represents the comparison, and O stands for the outcome.

During rounds, a charge nurse hears the patient care techni- cian yelling loudly to a patient regarding a transfer from the bed to chair. When entering the room, what is the nurse's best response? a. "You need to speak to the patient quietly. You are disturb- ing the patient." b. "Let me help you with your transfer technique." c. "When you are finished, be sure to apologize for your rough demeanor." d. "When your patient is safe and comfortable, meet me at the desk."

d. The charge nurse should direct the patient care techni- cian to determine the patient's safety. Then the nurse should address any concerns regarding the patient care technician's communication techniques privately. The nurse should direct the patient care technician on aspects of therapeutic commu- nication.

A nurse is writing an evaluative statement for a patient who is trying to lower her cholesterol through diet and exercise. Which evaluative statement is written correctly? a. "Outcome not met." b. "1/21/15—Patient reports no change in diet." c. "Outcome not met. Patient reports no change in diet or activity level." d. "1/21/15—Outcome not met. Patient reports no change in diet or activity level."

d. The evaluative statement must contain a date; the words "outcome met," "outcome partially met," or "out- come not met"; and the patient data or behaviors that support this decision. Answers a, b, and c are incomplete statements.

An unresponsive patient is wearing gas-permeable contact lenses. How would the nurse remove these lenses? a. Gently irrigate the eye with an irrigating solution from the inner canthus outward. b. Grasp the lens with a gentle pinching motion. c. Don sterile gloves before attempting the removal procedure. d. Ensure that the lens is centered on the cornea before gently manipulating the lids to release it.

d. The lens must be situated on the cornea, not the sclera, before removal. To remove hard contact lenses, the upper and lower eyelids are gently maneuvered to help loosen the lens and slide it out of the eye. An attempt to grasp a hard lens might result in a scratch on the cornea. Clean, not sterile, gloves are used.

When a nurse enters the patient's room to begin a nursing history, the patient's wife is there. What should the nurse do? a. Introduce oneself and thank the wife for being present. b. Introduce oneself and ask the wife if she wants to remain. c. Introduce oneself and ask the wife to leave. d. Introduce oneself and ask the patient if he would like the wife to stay.

d. The patient has the right to indicate whom he would like to be present for the nursing history and exam. You should nei- ther presume that he wants his wife there nor that he does not want her there. Similarly, the choice belongs to the patient, not the wife.

A nurse is caring for an obese 62-year-old patient with arthri- tis who has developed an open reddened area over his sacrum. What is a priority nursing diagnosis for this patient? a. Imbalanced Nutrition: More Than Body Requirements related to immobility b. Impaired Physical Mobility related to pain and discomfort c. Chronic Pain related to immobility d. Risk for Infection related to altered skin integrity

d. The priority diagnosis in this situation is the possibility of an infection developing in the open skin area. The others may be potential or probable diagnoses for this patient and may also require nursing interventions after the first diagnosis is addressed.

A 49-year-old who injured his spine in a motorcycle accident is receiving rehabilitation services in a short-term rehabilita- tion center. The nurse caring for him correctly tells the aide not to place him in which position? a. Side-lying b. Fowler's c. Sims' d. Prone

d. The prone position is contraindicated in patients who have spinal problems because the pull of gravity on the trunk when the patient lies prone produces a marked lordosis, or forward curvature of the lumbar spine.

A pregnant woman is receiving an epidural analgesic prior to delivery. The nurse provides vigilant monitoring of this patient to prevent the occurrence of: a. Pruritus b. Urinary retention c. Vomiting d. Respiratory depression

d. Too much of an opioid drug given by way of an epidural catheter or a displaced catheter may result in the occurrence of respiratory depression. Pruritus, urinary retention, and vomiting may occur but are not life threatening.

A nurse is looking for trends in a postoperative patient's vital signs. Which documents would the nurse consult first? a. Admission sheet b. Admission nursing assessment c. Activity flow sheet d. Graphic record

d. While one recording of vital signs should appear on the admission nursing assessment, the best place to find sequen- tial recordings that show a pattern or trend is the graphic record. The admission sheet does not include vital sign docu- mentation, and neither does the activity flow sheet.

In early civilizations, the theory of animism attempted to explain the mysterious changes occurring in bodily functions. Which statement describes a component of the development of nursing that occurred in this era? a. Women who committed crimes were recruited into nursing the sick in lieu of serving jail sentences. b. Nurses identified the personal needs of the patient and their role in meeting those needs. c. Women called deaconesses made the first visits to the sick and male religious orders cared for the sick and buried the dead. d. The nurse was the mother who cared for her family during sickness by using herbal remedies.

d. The theory of animism was based on the belief that every- thing in nature was alive with invisible forces and endowed with power. In this era, the nurse usually was the mother who cared for her family during sickness by providing physical care and herbal remedies. At the beginning of the 16th century the shortage of nurses led to the recruitment of women who had committed crimes to provide nursing care instead of going to jail. In the early Christian period, women called deacon- esses made the first organized visits to sick people, and mem- bers of male religious orders gave nursing care and buried the dead. The influences of Florence Nightingale were apparent from the middle of the 19th century to the 20th century; one of her accomplishments was identifying the personal needs of the patient and the nurse's role in meeting those needs.

A 3-year-old child is being admitted to a medical division for vomiting, diarrhea, and dehydration. During the admis- sion interview, the nurse should implement which commu- nication techniques to elicit the most information from the parents? a. The use of reflective questions b. The use of closed questions c. The use of assertive questions d. The use of clarifying questions

d. The use of the clarifying question or comment allows the nurse to gain an understanding of a patient's comment. When used properly, this technique can avert possible misconcep- tions that could lead to an inappropriate nursing diagnosis. The reflective question technique involves repeating what the person has said or describing the person's feelings. Open- ended questions encourage free verbalization and expression of what the parents believe to be true. Assertive behavior is the ability to stand up for oneself and others using open, hon- est, and direct communication.

A nurse enters the room of a patient with cancer. The patient is crying and states, "I feel so alone." Which statement is the most therapeutic? a. The nurse stands at the patient's bedside and states, "I understand how you feel. My mother said the same thing when she was ill." b. The nurse places a hand on the patient's arm and states, "You feel so alone." c. The nurse stands in the patient's room and asks, "Why do you feel so alone? Your wife has been here every day." d. The nurse holds the patient's hand and asks, "What makes you feel so alone?"

d. The use of touch conveys acceptance, and the implemen- tation of an open-ended question allows the patient time to verbalize freely.

A nurse writes the following outcome for a patient who is trying to stop smoking: "The patient appreciates or values a healthy body sufficiently to stop smoking." This is an exam- ple of what type of outcome? a. Cognitive b. Psychomotor c. Affective d. Physical changes

c. Affective outcomes pertain to changes in patient values, beliefs, and attitudes. Cognitive outcomes involve increases in patient knowledge; psychomotor outcomes describe the patient's achievement of new skills; physical changes are actual bodily changes in the patient (e.g., weight loss, increased muscle tone).

A nursing instructor teaching classes in gerontology to nurs- ing students discusses myths related to the aging of adults. Which statement is a myth about older adults? a. Most older adults live in their own homes. b. Healthy older adults enjoy sexual activity. c. Old age means mental deterioration. d. Older adults want to be attractive to others.

c. Although response time may be longer, intelligence does not normally decrease because of aging. Most older adults own their own homes, and although sexual activity may be less frequent, the ability to perform and enjoy sexual activity lasts well into the 90s in healthy older adults. Older adults want to be attractive to others.

What is the leading cause of cognitive impairment in old age? a. Stroke b. Malnutrition c. Alzheimer disease d. Loss of cardiac reserve

c. Alzheimer disease is the most common degenerative neurologic illness and the most common cause of cognitive impairment. It is irreversible, progressing from deficits in memory and thinking skills to an inability to perform even the simplest of tasks.

While discussing home safety with the nurse, a patient admits that she always smokes a cigarette in bed before falling asleep at night. Which nursing diagnosis would be the priority for this patient? a. Impaired Gas Exchange related to cigarette smoking b. Anxiety related to inability to stop smoking c. Risk for Suffocation related to unfamiliarity with fire prevention guidelines d. Deficient Knowledge related to lack of follow-through of recommendation to stop smoking

c. Because Mrs. Fuller is not aware that smoking in bed is extremely dangerous, she is at risk for suffocation from fire. The other three nursing diagnoses are correctly stated but are not a priority in this situation.

A nursing student is nervous and concerned about the work she is about to do at the clinical facility. To allay anxiety and be successful in her provision of care, it is most important for her to: a. Determine the established goals of the institution b. Be sure her verbal and nonverbal communication is congruent c. Engage in self-talk to plan her day and decrease her fear d. Speak with her fellow colleagues about how they feel

c. By engaging in self-talk, or intrapersonal communication, the nursing student can plan her day and enhance her clinical performance to decrease fear and anxiety.

When conducting quantitative research, the researcher col- lects information to support a hypothesis. This information would be identified as: a. The subject b. Variables c. Data d. The instrument

c. Data refers to information that the researcher collects from subjects in the study (expressed in numbers). A vari- able is something that varies and has different values that can be measured. Instruments are devices used to collect and record the data, such as rating scales, pencil-and-paper tests, and biologic measurements.

A nurse is explaining to a patient the anticipated effect of the application of cold to an injured area. What response indi- cates that the patient understands the explanation? a. "I can expect to have more discomfort in the area where the cold is applied." b. "I should expect more drainage from the incision after the ice has been in place." c. "I should see less swelling and redness with the cold treatment." d. "My incision may bleed more when the ice is first applied."

c. The local application of cold constricts peripheral blood vessels, reduces muscle spasms, and promotes comfort. Cold reduces blood flow to tissues, decreases the local release of pain-producing substances, decreases metabolic needs, and capillary permeability. The resulting effects include decreased edema, coagulation of blood at the wound site, promotion of comfort, decreased drainage from wound, and decreased bleeding.

A nurse asks a patient to close her eyes, state when she feels something, and describe the feeling. The nurse then brushes the patient's skin with a cotton ball, and touches the patient's skin with both sides of a safety pin. Which sense is the nurse assessing? a. Gustatory b. Olfactory c. Tactile d. Kinesthetic

c. The nurse is assessing for tactile (touch) disturbances by brushing the skin with a cotton ball and touching the skin with a safety pin. Gustatory disturbances involve taste, olfac- tory disturbances involve the sense of smell, and kinesthetic disturbances are related to body positioning.

A patient with COPD is unable to perform activities of daily living (ADLs) without becoming exhausted. Which nursing diagnosis best describes this alteration in oxygenation as the etiology? a. Decreased Cardiac Output related to difficulty breathing b. Impaired Gas Exchange related to use of bronchodilators c. Fatigue related to impaired oxygen transport system d. Ineffective Airway Clearance related to fatigue

c. Fatigue related to an impaired oxygen transport system is an example of a nursing diagnosis with alteration in oxygena- tion as the etiology or cause of other problems. Ineffective Airway Clearance, Decreased Cardiac Output and Impaired Gas Exchange are examples of nursing diagnoses indicating alterations in oxygenation as the problem.

A nurse is caring for a 73-year-old male patient who is hos- pitalized with pneumonia and is experiencing some difficulty breathing. The nurse most appropriately assists him into which position to promote maximal breathing in the thoracic cavity? a. Dorsal recumbent position b. Lateral position c. Fowler's position d. Sims' position

c. Fowler's position promotes maximal breathing space in the thoracic cavity and is the position of choice when someone is having difficulty breathing. Lying flat on the back or side or Sims' position would not facilitate respiration and would be difficult for the patient to maintain.

A nurse has volunteered to give influenza immunizations at a local clinic. What level of care is the nurse demonstrating? a. Tertiary b. Secondary c. Primary d. Promotive

c. Giving influenza injections is an example of primary health promotion and illness prevention.

A nurse is collecting evaluative data for a patient who is finished receiving chemotherapy for an osteosarcoma. Which nursing action represents this step of the nursing process? a. The nurse collects data to identify health problems. b. The nurse collects data to identify patient strengths. c. The nurse collects data to justify terminating the plan of care. d. The nurse collects data to measure outcome achievement.

d, The nurse collects evaluative data to measure outcome achievement. While this may justify terminating the plan of care, that is not necessarily so. Data to assess health problems and patient variables are collected during the first step of the nursing process.

A nurse is assisting a patient who is 2 days postoperative from a cesarean section to sit in a chair. After assisting the patient to the side of the bed and to stand up, the patient's knees buckle and she tells the nurse she feels faint. What should the nurse do in this situation? a. Wait a few minutes and then continue the move to the chair. b. Call for assistance and continue the move with the help of another nurse. c. Lower the patient back to the side of the bed and pivot her back into bed. d. Have the patient sit down on the bed and dangle her feet before moving.

c. If a patient becomes faint and knees buckle when moving from bed to a chair, the nurse should not continue the move to the chair. The nurse should lower the patient back to the side of the bed, pivot her back into bed, cover her, and raise the side rails. Assess the patient's vital signs and for the presence of other symptoms. Another attempt should be made with the assistance of another staff member if vital signs are stable. Instruct the patient to remain in the sitting position on the side of the bed for several minutes to allow the circulatory system to adjust to a change in position, and avoid hypoten- sion related to a sudden change in position.

A nurse is finished with patient care. How would the nurse remove PPE when leaving the room? a. Remove gown, goggles, mask, gloves, and exit the room. b. Remove gloves, perform hand hygiene, then remove gown, mask, and goggles. c. Untie gown waiststrings, remove gloves, goggles, gown, mask; perform hand hygiene. d. Remove goggles, mask, gloves, gown, and perform hand hygiene.

c. If an impervious gown has been tied in front of the body at the waist, the nurse should untie the waist strings before removing gloves. Gloves are always removed first because they are most likely to be contaminated, followed by the goggles, gown, and mask, and hands should be washed thoroughly after the equipment has been removed and before leaving the room.

A nurse is performing intermittent closed-catheter irrigation for a patient with an indwelling catheter. After attaching the syringe to the access port on the catheter, the nurse finds that the irrigant will not enter the catheter. What intervention would the nurse appropriately perform next? a. Apply pressure to the catheter to force the solution into the catheter. b. Disconnect and reconnect the drainage system quickly. c. Notify the primary care provider. d. Change the catheter.

c. If the irrigation solution will not enter the catheter, the nurse should not force the solution into the catheter; instead, the nurse should notify the primary care provider and prepare to change the catheter.

A nurse who is working with women in a drop-in shelter studies Carol Gilligan's theory of morality in women to use when planning care. According to Gilligan, what is the moti- vation for female morality? a. Law and justice b. Obligations and rights c. Response and care d. Order and selfishness

c. In Gilligan's theory, males and females have different ways of looking at the world. Males are more likely to associate morality with obligations, rights, and justice. Females are more likely to see moral requirements emerging from the needs of others within the context of a relationship.

A resident who is called to see a patient in the middle of the night is leaving the unit but then remembers that he forgot to write a new order for a pain medication a nurse had requested for another patient. Tired and already being paged to another unit, he verbally tells the nurse the order and asks the nurse to document it on the physician's order sheet. The nurse's best response is: a. "Thank you for taking care of this!" b. Get a second nurse to listen to the order, and after writing the order on the physician order sheet, have both nurses sign it. c. "I am sorry, but verbal orders can only be given in an emergency situation that prevents us from writing them out. I'll bring the chart and we can do this quickly." d. Try calling another resident for the order or wait until the next shift.

c. In most agencies, the only circumstance in which an attending physician, nurse practitioner, or house officer may issue orders verbally is in a medical emergency, when the physician/nurse practitioner is present but finds it impossible, due to the emergency situation, to write the order.

During a physical assessment, a nurse inspects a patient's abdomen. What assessment technique would the nurse perform next? a. Percussion b. Palpation c. Auscultation d. Whichever is more comfortable for the patient

c. When assessing the abdomen, the sequence is inspection, auscultation, percussion, and palpation. Auscultation follows inspection because percussion and palpation stimulate bowel sounds.

A patient, age 16, was in an automobile accident and received a wound across her nose and cheek. After surgery to repair the wound, the patient says, "I am so ugly now." Based on this statement, what nursing diagnosis would be most appropriate? a. Pain b. Impaired Skin Integrity c. Disturbed Body Image d. Disturbed Thought Processes

c. Wounds cause emotional as well as physical stress.

A nurse is teaching parents of toddlers how to spend quality time with their children. Which activity would be develop- mentally appropriate for this age group? a. Playing video games b. Playing peek-a-boo c. Playing in a sand box d. Playing board games

c. Playing in a sand box with toys that emphasize gross motor skills and creativity is a developmentally appropriate activity for a toddler. Video games are appropriate for school-aged children and adolescents, but should be monitored. Playing peek-a-boo is developmentally appropriate for an infant, and playing board games usually begins with preschool and older children.

Which type of quantitative research is often conducted to examine the effects of nursing interventions on patient outcomes? a. Descriptive research b. Correlational research c. Quasi-experimental research d. Experimental research

c. Quasi-experimental research is often conducted in clini- cal settings to examine the effects of nursing interventions on patient outcomes. Descriptive research is often used to generate new knowledge about topics with little or no prior research. Correlational research examines the type and degree of relationships between two or more variables. Experimental research examines cause-and-effect relationships between variables under highly controlled conditions.

An older patient with an unsteady gait requests a tub bath. Which action would be most appropriate? a. Add Alpha-Keri oil to the water to prevent dry skin. b. Allow the patient to lock the door to guarantee privacy. c. Assist the patient in and out of the tub to prevent falling. d. Keep the water temperature very warm because the patient chills easily.

c. Safe nursing practice requires that the nurse assist a patient with an unsteady gait in and out of the tub. Adding Alpha- Keri oil to the bath water is dangerous for this patient because it makes the tub slippery. Although privacy is important, if the patient locks the door, the nurse cannot help if there is an emergency. The water should be comfortably warm at 43°C to 46°C. Older patients have an increased susceptibility to burns due to diminished sensitivity.

After an initial skin assessment, the nurse documents the presence of a reddened area that has blistered. According to recognized staging systems, this ulcer would be classified as: a. Stage I b. Stage II c. Stage III d. Stage IV

b. A stage II pressure ulcer involves partial thickness loss of dermis and presents as a shallow open ulcer with a red pink wound bed, without slough. It may also present as an intact or open/ruptured serum-filled blister.

A nurse is scheduling hygiene for patients on her unit. What is the most important consideration when planning a patient's personal hygiene? a. When the patient had his or her most recent bath b. The patient's usual hygiene practices and preferences c. Where the bathing fits in the nurse's schedule d. The time that is convenient for the patient care assistant

b. Bathing practices and cleansing habits and rituals vary widely. The patient's preferences should always be taken into consideration, unless there is a clear threat to health. The patient and nurse should work together to come to a mutu- ally agreeable time and method to accomplish the patient's personal hygiene. The availability of staff to assist may be important, but the patient's preferences are a higher priority.

A nurse is about to perform pin site care for a patient who has a halo traction device installed. What is the first nursing action that should be taken prior to performing this care? a. Administer pain medication. b. Reassess the patient. c. Prepare the equipment. d. Explain the procedure to the patient.

b. Before implementing any nursing action, the nurse should reassess the patient to determine whether the action is still needed. Then the nurse may collect the equipment, explain the procedure, and if necessary administer pain medications.

A student nurse is learning to assess blood pressure. What does the blood pressure measure? a. Flow of blood through the circulation b. Force of blood against arterial walls c. Force of blood against venous walls d. Flow of blood through the heart

b. Blood pressure is the measurement of the force of blood against arterial walls.

A nurse caring for patients in a pediatrician's office assesses infants and toddlers for physical developmental milestones. Which patient would the nurse refer to a specialist based on failure to achieve these milestones? a. A 4-month-old infant who is unable to roll over b. A 6-month-old infant who is unable to hold his head up himself c. An 11-month-old infant who cannot walk unassisted d. An 18-month-old toddler who cannot jump

b. By 5 months, head control is usually achieved. An infant usually rolls over by 6 to 9 months. By 15 months, most toddlers can walk unassisted. By 2 years, most toddlers can jump.

The nurse caring for patients postoperatively uses careful hand hygiene and sterile techniques when handling patients. Which of Maslow's basic human needs is being met by this nurse? a. Physiologic b. Safety and security c. Self-esteem d. Love and belonging

b. By carrying out careful hand hygiene and using sterile technique, nurses provide safety from infection. An example of a physiologic need is clearing a patient's airway. Self- esteem needs may be met by allowing an older adult to talk about a past career. An example of helping meet a love and belonging need is contacting a hospitalized patient's family to arrange a visit.

A nurse observes that a patient who has cataracts is sitting closer to the television than usual. The nurse would interpret that the etiologic basis of this sensory problem is an altera- tion in: a. Environmental stimuli b. Sensory reception c. Nerve impulse conduction d. Impulse translation

b. Cataracts are interfering with the patient's ability to receive visual stimuli: altered sensory reception. The nature of incoming stimuli, the conduction of nerve impulses, and the translation of incoming impulses in the brain are not a problem here.

A college freshman away from home for the first time says, "Why did I have to be born into a family of big bottoms and short fat legs! No one will ever ask me out for a date. Oh, why can't I have long thin legs like everyone else in my class? What a frump I am." a. Personal Identity Disturbance b. Body Image Disturbance c. Self-Esteem Disturbance d. Altered Role Performance

b. Clearly, this patient's concern is with body image.

A nurse in the rehabilitation division states to her head nurse, Mr. Tyler, "I need the day off and you didn't give it to me!" The head nurse replies, "Well, I wasn't aware you needed the day off, and it isn't possible since staffing is so inadequate." Instead of this exchange, what communication by the nurse would have been more effective? a. "Mr. Tyler, I placed a request to have August 8th off, but I'm working and I have a doctor's appointment." b. "Mr. Tyler, I would like to discuss my schedule with you. I requested the 8th of August off for a doctor's appointment. Could I make an appointment?" c. "Mr. Tyler, I will need to call in on the 8th of August because I have a doctor's appointment." d. "Mr. Tyler, since you didn't give me the 8th of August off, will I need to find someone to work for me?"

b. Effective communication by the sender involves the imple- mentation of nonthreatening information by showing respect to the receiver. The nurse should identify the subject of the meeting and be sure it occurs at a mutually agreed upon time.

A patient is having dyspnea. What would the nurse do first? a. Remove pillows from under the head b. Elevate the head of the bed c. Elevate the foot of the bed d. Take the blood pressure

b. Elevating the head of the bed allows the abdominal organs to descend, giving the diaphragm greater room for expansion and facilitating lung expansion.

Which method of qualitative research was developed by the discipline of anthropology? a. Historical b. Ethnography c. Grounded theory d. Phenomenology

b. Ethnographic research was developed by the discipline of anthropology and is used to examine issues of culture of interest to nursing. Historical research examines events of the past to increase understanding of the nursing profession today. The basis of grounded theory methodology is the discovery of how people describe their own reality and how their beliefs are related to their actions in a social scene. The purpose of phenomenology (both a philosophy and a research method) is to describe experiences as they are lived by the subjects being studied.

A nurse is performing an initial comprehensive assessment of an 84-year-old male patient admitted to a long-term care facility from home. The nurse begins the assessment by ask- ing the patient, "How would you describe your health status and well-being?" The nurse also asks the patient, "What do you do to keep yourself healthy?" Which model for organiz- ing data is this nurse following? a. Maslow's human needs b. Gordon's functional health patterns c. Human response patterns d. Body system model

b. Gordon's functional health patterns begin with the patient's perception of health and well-being and progress to data about nutritional-metabolic patterns, elimination patterns, activity, sleep/rest, self-perception, role relationship, sexual- ity, coping, and values/beliefs. Maslow's model is based on the human needs hierarchy. Human responses include exchanging, communicating, relating, valuing, choosing, moving, perceiving, knowing, and feeling. The body system model is based on the functioning of the major body systems.

When assessing pain in a child, the nurse needs to be aware of what considerations? a. Immature neurologic development results in reduced sensation of pain. b. Inadequate or inconsistent relief of pain is widespread. c. Reliable assessment tools are currently unavailable. d. Narcotic analgesic use should be avoided.

b. Health care personnel are only now becoming aware of pain relief as a priority for children in pain. The evidence supports the fact that children do indeed feel pain and reli- able assessment tools are available specifically for use with children. Opioid analgesics may be safely used with children as long as they are carefully monitored.

After surgery, a patient is having difficulty voiding. Which nursing action would most likely lead to an increased dif- ficulty with voiding? a. Pouring warm water over the patient's fingers. b. Having the patient ignore the urge to void until her bladder is full. c. Using a warm bedpan when the patient feels the urge to void. d. Stroking the patient's leg or thigh.

b. Ignoring the urge to void makes urination even more difficult and should be avoided. The other activities are all recommended nursing activities to promote voiding.

A nurse teaches a patient at home to use clean technique when changing a wound dressing. This practice is considered: a. The nurse's preference b. Safe for the home setting c. Unethical behavior d. Grossly negligent

b. In the home setting, where the patient's environment is more controlled, medical asepsis is usually recommended, with the exception of self-injection. This is the appropriate procedure for the home and is neither unethical nor grossly negligent.

When describing safety issues and related mortality to a local senior citizens group, what would the nurse identify as the leading cause of hospital admissions for trauma in older adults? a. Fires b. Exposure to temperature extremes c. Intimate partner violence d. Falls

d. Falls among older adults are the most common cause of hospital admissions for trauma. Fires and temperature extremes are also significant hazard for older adults but are not the most common cause of trauma admissions. Intimate partner violence occurs more frequently in adults as opposed to older adults.

A nurse has taught a diabetic patient how to administer his daily insulin. The nurse should evaluate the teaching-learning process by: a. Determining the patient's motivation to learn b. Deciding if the learning outcomes have been achieved c. Allowing the patient to practice the skill he has just learned d. Documenting the teaching session in the patient's medical record

b. The nurse cannot assume that the patient has actually learned the content unless there is some type of proof of learning. The key to evaluation is meeting the learner out- comes stated in the teaching plan.

A nurse forms the following nursing diagnosis for a patient: Impaired Urinary Elimination related to maturational enu- resis. Based on this diagnosis, for which patient is the nurse caring? a. An adult older than 65 years of age who is incontinent b. A child older than 4 years of age who has involuntary urination c. A 12-month-old child who has involuntary urination d. A patient with neurologic damage resulting in bladder dysfunction

b. Maturational enuresis is involuntary urination after an age when continence should be present. A 12-month-old child is not expected to be continent, and incontinence and neurologic damage are not maturational problems.

Based on Erikson's theory, middle adults who do not achieve their developmental tasks may be considered to be in stagna- tion. Which statement is one example of this finding? a. "I am helping my parents move into an assisted-living facility." b. "I spend all of my time going to the doctor to be sure I am not sick." c. "I have enough money to help my son and his wife when they need it." d. "I earned this gray hair and I like it!"

b. Middle adults who do not reach generativity tend to become overly concerned about their own physical and emotional health needs.

Applying the gate control theory of pain, what would be an effective nursing intervention for a patient with lower back pain? a. Encouraging regular use of analgesics b. Applying a moist heating pad to the area at prescribed intervals c. Reviewing the pain experience with the patient d. Ambulating the patient after administering medication

b. Nursing measures such as applying warmth to the lower back stimulate the large nerve fibers to close the gate and block the pain. The other choices do not involve attempts to stimulate large nerve fibers that interfere with pain transmis- sion as explained by the gate control theory.

A nurse is caring for an 80-year-old patient who has become weak and fatigues easily. He is unable to wash his body and always asks the nurse to brush his teeth. Based on this information, what is an appropriate nursing diagnosis for this patient? a. Risk for Impaired Skin Integrity related to immobility b. Bathing/Hygiene Self-Care Deficit related to decreased strength and endurance c. Social Isolation related to lack of visitors d. Impaired Oral Mucous Membrane related to inability to brush his teeth

b. Risk for Impaired Skin Integrity, Social Isolation, and Impaired Oral Mucous Membrane may be appropriate nurs- ing diagnoses for this patient. However, not enough is known, based on the information given, to formulate these diagnoses. The priority at this time, based on the given information, is Bathing/Hygiene Self-Care Deficit.

When assessing a patient receiving a continuous opioid infu- sion, the nurse immediately notifies the physician when the patient has: a. A respiratory rate of 10/min with normal depth b. A sedation level of 4 c. Mild confusion d. Reported constipation

b. Sedation level is more indicative of respiratory depression because a drop in level usually precedes it. A sedation level of 4 calls for immediate action because the patient has minimal or no response to stimuli. A respiratory level of 10 with normal depth of breathing is usually not a cause for alarm. Mild confu- sion may be evident with the initial dose and then disappear; additional observation is necessary. Constipation should be reported to the physician, but is not the priority in this situation.

The nurse encourages parents of hospitalized infants and tod- dlers to stay with their child to help decrease what potential problem? a. Problems with attachment b. Separation anxiety c. Risk for injury d. Failure to thrive

b. Separation anxiety, with crying initially and then appearing depressed, is common during late infancy in infants who are hospitalized.

A patient has intravenous fluids infusing in the right arm. When taking a blood pressure on this patient, what would the nurse do in this situation? a. Take the blood pressure in the right arm b. Take the blood pressure in the left arm c. Use the smallest possible cuff d. Report inability to take the blood pressure

b. The blood pressure should be taken in the arm opposite the one with the infusion.

The student nurse learns that illnesses are classified as either acute or chronic. Which are examples of chronic illnesses? Select all that apply. a. Diabetes mellitus b. Bronchial pneumonia c. Rheumatoid arthritis d. Cystic fibrosis e. Fractured hip f. Otitis media

a, c, d. Diabetes, arthritis, and cystic fibrosis are chronic diseases because they are permanent changes caused by irreversible alterations in normal anatomy and physiology, and they require patient education along with a long period of care or support. Pneumonia, fractures, and otitis media are acute illnesses because they have a rapid onset of symptoms that last a relatively short time.

A nurse is attempting to improve care on the pediatric ward of a hospital. Which nursing improvements might the nurse employ when following the recommendations of the Institute of Medicine's Committee on Quality of Health Care in America? Select all that apply. a. Basing patient care on continuous healing relationships b. Customizing care to reflect the competencies of the staff c. Using evidence-based decision making d. Having a charge nurse as the source of control e. Using safety as a system priority f. Recognizing the need for secrecy to protect patient privacy

a, c, e. Care should be based on continuous healing relation- ships and evidence-based decision making. Customization should be based on patient needs and values with the patient as the source of control. Safety should be used as a system priority, and the need for transparency should be recognized.

It is important to have the appropriate cuff size when taking the blood pressure. What error may result from a cuff that is too large or too small? a. An incorrect reading b. Injury to the patient c. Prolonged pressure on the arm d. Loss of Korotkoff sounds

a. A blood pressure cuff that is not the right size may cause an incorrect reading.

A nurse is assessing the following children. Which child would the nurse identify as having the greatest risk for choking and suffocating? a. A toddler playing with his 9-year-old brother's construction set b. A 4-year-old eating yogurt for lunch c. An infant covered with a small blanket and asleep in the crib d. A 3-year-old drinking a glass of juice

a. A young child may place small or loose parts in the mouth; a toy that is safe for a 9-year-old could kill a toddler. An infant sleeping in a crib without a pillow or large blanket and a 3-year-old and a 4-year-old drinking juice and eating yogurt are not particular safety risks.

When completing a safety event report, the nurse should: a. Include suggestions on how to prevent the incident from recurring. b. Provide minimal information about the incident. c. Discuss the details with the patient before documenting them. d. Objectively describe the incident in detail.

d. A safety event report is a legal document, which must be as objective and complete as possible. It is not a collabora- tive effort with the patient, and any suggestions to prevent the occurrence from happening again should be discussed at a postincident conference.

A nurse writes the following outcome for a patient who is trying to lose weight: "The patient can explain the relationship between weight loss, increased exercise, and decreased calorie intake." This is an example of what type of outcome? a. Cognitive b. Psychomotor c. Affective d. Physical changes

a. Cognitive outcomes involve increases in patient knowl- edge; psychomotor outcomes describe the patient's achieve- ment of new skills; affective outcomes pertain to changes in patient values, beliefs, and attitudes; and physical changes are actual bodily changes in the patient (e.g., weight loss, increased muscle tone).

A nurse is preparing to teach a 45-year-old male patient with asthma how to use his inhaler. Which teaching tool is one of the best methods to teach the patient this skill? a. Demonstration b. Lecture c. Discovery d. Panel session

a. Demonstration of techniques, procedures, exercises, and the use of special equipment is an effective patient teaching strategy for a skill. Lecture can be used to deliver information to a large group of patients but is more effective when the session is interactive; it is rarely used for individual instruc- tion, except in combination with other strategies. Discovery is a good method for teaching problem-solving techniques and independent thinking. Panel discussions can be used to impart factual material but are also effective for sharing experiences and emotions.

World War II had a tremendous effect on the nursing profes- sion. Which development occurred during this period? a. The role of the nurse was broadened. b. There was a decreased emphasis on education. c. Nursing was practiced mainly in hospital settings. d. There was an overabundance of nurses.

a. During World War II, large numbers of women worked outside the home. They became more independent and asser- tive, which led to an increased emphasis on education. The war itself created a need for more nurses and resulted in a knowledge explosion in medicine and technology. This trend broadened the role of nurses to include practicing in a wide variety of health care settings.

A patient who has pneumonia has had a fever for 3 days. What characteristics would the nurse anticipate related to the patient's urine output? a. Decreased and highly concentrated b. Decreased and highly dilute c. Increased and concentrated d. Increased and dilute

a. Fever and diaphoresis cause the kidneys to conserve body fluids. Thus, the urine is concentrated and decreased in amount.

A 2-year-old grabs a handful of cake from the table and stuffs it in his mouth. According to Freud, what part of the mind is the child satisfying? a. Id b. Superego c. Ego d. Unconscious mind

a. Freud defined the id as the part of the mind concerned with self-gratification by the easiest and quickest available means.

A nurse follows accepted guidelines for a healthy lifestyle. How can this promote health in others? a. By being a role model for healthy behaviors b. By not requiring sick days from work c. By never exposing others to any type of illness d. By spending less money on food

a. Good personal health enables the nurse to serve as a role model for patients and families.

A nurse is practicing community-based nursing in a mobile health clinic. What would be the central focus of this nurse's care? a. Individual and family health care needs b. Populations within the community c. Local health care facilities d. Families in crisis

a. In contrast to community health nursing, which focuses on populations within a community, community-based nurs- ing is centered on individual and family health care needs. Community-based nurses may help families in crisis and work in health care facilities, but these are not the focus of community-based nursing.

Which factor initially influences moral development as described in Kohlberg's theory? a. Parent-child communications b. Societal rules and regulations c. Social and religious rules d. One's beliefs and values

a. Moral development in the young child results from com- munications as the child tries to please his or her parents.

An elderly patient is confined to bedrest following cervical spine surgery to treat nerve pinching. The nurse is vigilant about turning the patient and assessing the patient regularly to prevent the formation of pressure ulcers. What type of agent is the stimulus for pressure ulcers? a. Mechanical b. Thermal c. Chemical d. Electrical

a. Receptors in the skin and superficial organs may be stimu- lated by mechanical, thermal, chemical, and electrical agents. Friction from bed linens causing pressure sores and pressure from a cast are mechanical stimulants. Sunburn is a thermal stimulant. An acid burn is the result of a chemical stimulant. The jolt from a lightening bolt is an electrical stimulant.

What action does the nurse perform to follow safe technique when using a portable oxygen cylinder? a. Checking the amount of oxygen in the cylinder before using it b. Using a cylinder for a patient transfer that indicates available oxygen is 500 psi c. Placing the oxygen cylinder on the stretcher next to the patient d. Discontinuing oxygen flow by turning cylinder key counterclockwise until tight

a. The cylinder must always be checked before use to ensure that enough oxygen is available for the patient. It is unsafe to use a cylinder that reads 500 psi or less because not enough oxygen remains for a patient transfer. A cylinder that is not secured prop- erly may result in injury to the patient. Oxygen flow is discontin- ued by turning the valve clockwise until it is tight.

The agent-host-environment model of health and illness is based on what concept? a. Risk factors b. Demographic variables c. Behaviors to promote health d. Stages of illness

a. The interaction of the agent, host, and environment creates risk factors that increase the probability of disease.

A public health nurse is leaving the home of a young mother who has a special needs baby. The neighbor states, "How is she doing, since the baby's father is no help?" What is the nurse's best response to the neighbor? a. "New mothers need support." b. "The lack of a father is difficult." c. "How are you today?" d. "It is a very sad situation."

a. The nurse must maintain confidentiality when providing care. The statement "New mothers need support" is a general statement that all new parents need help. The statement is not judgmental of the family's roles.

Which patient would a nurse assess as being at greatest risk for sensory deprivation? a. An older man confined to bed at home after a stroke b. An adolescent in an oncology unit working on homework supplied by friends c. A woman in labor d. A toddler in a playroom awaiting same-day surgery

a. The patient confined to bedrest at home is at risk for greatly reduced environmental stimuli. All of the other patients are in environments in which environmental stimuli are at least adequate.

A nurse is inserting an oropharyngeal airway for a patient who vomits when it is inserted. Which action would be the first that should be taken by the nurse related to this occurrence? a. Quickly position the patient on his or her side. b. Put on disposable gloves and remove the oral airway. c. Check that the airway is the appropriate size for the patient. d. Put on sterile gloves and suction the airway.

a. When a patient vomits upon insertion of an oropharyngeal airway, the nurse should immediately position the patient on his or her side to prevent aspiration, remove the oral airway, and suction the mouth if needed.

A nurse is using the SOAP format of documentation to docu- ment care of a patient who is diagnosed with type 2 diabetes. Which source of information would be the nurse's focus when completing this documentation? a. A patient problem list b. Notes describing the patient's condition c. Overall trends in patient status d. Planned interventions and patient outcomes

a. When using the SOAP format, the problem list at the front of the chart alerts all caregivers to patient priorities. Narra- tive notes allow nurses to describe a condition, situation, or response in their own terms. Abnormal status can be seen immediately when using charting by exception, and planned interventions and patient expected outcomes are the focus of the case management model.

An experienced nurse tells a younger nurse who is working in a retirement home that older adults are different and do not have the same desires, needs, and concerns as other age groups. The nurse also comments that most older adults have "outlived their usefulness." What is the term for this type of prejudice? a. Harassment b. Whistle blowing c. Racism d. Ageism

d. Ageism is a form of prejudice in which older adults are stereotyped by characteristics found in only a few members of their age group. Harassment occurs when a dominant per- son takes advantage of or overpowers a less dominant person (may involve sexual harassment or power struggles). Whistle blowing involves reporting illegal or unethical behavior in the workplace. Racism is prejudice against other races and ethnic groups.

A patient is admitted with a nonhealing surgical wound. Which nursing action is most effective in preventing a wound infection? a. Using sterile dressing supplies b. Suggesting dietary supplements c. Applying antibiotic ointment d. Performing careful hand hygiene

d. Although all of the answers may help in preventing wound infections, careful hand washing (medical asepsis) is the most important.

The nursing process ensures that nurses are person centered rather than task centered. Rather than simply approach- ing a patient to take vital signs, the nurse thinks, "How is Mrs. Barclay today? Are our nursing actions helping her to achieve her goals? How can we better help her?" This dem- onstrates which characteristic of the nursing process? a. Systematic b. Interpersonal c. Dynamic d. Universally applicable in nursing situations

b, interpersonal. All of the other options are characteristics of the nursing process, but the conversation and thinking quoted best illustrates the interpersonal dimension of the nursing process.

A nurse assesses patients in a physician's office who are experiencing different levels of health and illness. Which statements best define the concepts of health and illness? Select all that apply. a. Health and illness are the same for all people. b. Health and illness are individually defined by each person. c. People with acute illnesses are actually healthy. d. People with chronic illnesses have poor health beliefs. e. Health is more than the absence of illness. f. Illness is the response of a person to a disease.

b,e,f.Eachpersondefineshealthandillnessindividually, based on a number of factors. Health is more than just the absence of illness; it is an active process in which a person moves toward one's maximum potential. An illness is the response of the person to a disease.

A nurse researcher studies the effects of genomics on cur- rent nursing practice. Which statements identify genetic principles that will challenge nurses to integrate genomics in their research, education, and practice? Select all that apply. a. Genetic tests plus family history tools have the potential to identify people at risk for diseases. b. Pharmacogenetic tests can determine if a patient is likely to have a strong therapeutic response to a drug or suffer adverse reactions from the medication. c. Evidence-based review panels are in place to evaluate the possible risks and benefits related to genetic testing. d. Valid and reliable national data are available to establish baseline measures and track progress toward targets. e. Genetic variation can either accelerate or slow the metabo- lism of many drugs. f. It is beyond the role of the nurse to answer questions and discuss the impact of genetic findings on health and illness.

a, b, e. In the very near future, all health care providers will be challenged to integrate genomics into their research, education, and practice (Healthy People 2020, 2012). Genetic tests plus family history tools have the potential to identify people at risk for diseases. Pharmacogenetics is the study of how genetic variation affects an individual's response to drugs. Pharmacogenetic tests can determine if a patient is likely to have a strong therapeutic response to a drug or suf- fer adverse reactions from the medication. Genetic variation can either accelerate or slow the metabolism of many drugs (Lehne, 2013; Human Genome Project, 2011). Two emerg- ing challenges related to genomic discoveries are (1) the need for evidence-based review panels to thoroughly evaluate the possible benefits and harms related to the expanding number of genetic tests and family health history tools, and (2) valid and reliable national data are needed to establish baseline measures and track progress toward targets (Healthy People 2020, 2012). Nurses must be prepared to answer questions and discuss the impact of genetic findings on health and illness.

The nurse caring for infants in a hospital nursery knows that newborns continue to grow and develop according to individual growth patterns and developmental levels. Which terms describe these patterns? Select all that apply. a. Orderly b. Simple c. Sequential d. Unpredictable e. Differentiated f. Integrated

a, c, e, f. Growth and development are orderly and sequential, as well as continuous and complex. Growth and development follow regular and predictable trends, and are both differenti- ated and integrated.

One of the most common distinctions of pain is whether it is acute or chronic. Which examples describe chronic pain? Select all that apply. a. A patient is receiving chemotherapy for bladder cancer. b. An adolescent is admitted to the hospital for an appendec- tomy. c. A patient is experiencing a ruptured aneurysm. d. A patient who has fibromyalgia requests pain medication. e. A patient has back pain related to an accident that occurred last year. f. A patient is experiencing pain from second-degree burns.

a, d, e. Chronic pain is pain that may be limited, intermit- tent, or persistent but that lasts beyond the normal healing period. Examples are cancer pain, fibromyalgia pain, and back pain. Acute pain is generally rapid in onset and varies in intensity from mild to severe, as occurs with an emer- gency appendectomy, a ruptured aneurysm, and pain from burns.

A patient has a blood pressure reading of 130/90 mm Hg when visiting a clinic. What would the nurse recommend to the patient? a. Follow-up measurements of blood pressure b. Immediate treatment by a physician c. No action, because the nurse considers this reading is due to anxiety d. A change in dietary intake

a. A single blood pressure reading that is mildly elevated is not significant, but the measurement should be taken again over time to determine if hypertension is a problem. The nurse would recommend a return visit to the clinic for a recheck.

A quality-assurance program reveals a higher incidence of falls and other safety violations on a particular unit. A nurse manager states, "We'd better find the folks responsible for these errors and see if we can replace them." This is an example of: a. Quality by inspection b. Quality by punishment c. Quality by surveillance d. Quality by opportunity

a. Quality by inspection focuses on finding deficient work- ers and removing them. Quality as opportunity (d) focuses on finding opportunities for improvement and fosters an environment that thrives on teamwork, with people sharing the skills and lessons they have learned. Answers b and c are distractors.

A nurse is providing postural drainage for a patient with cystic fibrosis. In which position should the nurse place the patient to drain the right lobe of the lung? a. High Fowler's position b. Left side with pillow under chest wall c. Lying position/half on abdomen and half on side d. Trendelenberg position

b. For postural drainage, the nurse should place the patient lying on the left side with a pillow under the chest wall to drain the right lobe of the lung, use high Fowler's position to drain the apical sections of the upper lobes of the lungs, place the patient in a lying position, half on the abdomen and half on the side, right and left, to drain the posterior sec- tions of the upper lobes of the lungs, and place the patient in the Trendelenburg position to drain the lower lobes of the lungs.

A 17-year-old has a fractured left leg, which has been casted. Following teaching from the physical therapist for using crutches, the nurse reinforces which teaching point with the patient? a. Use the axillae to bear body weight. b. Keep elbows close to the sides of the body. c. When rising, extend the uninjured leg to prevent weight bearing. d. To climb stairs, place weight on affected leg first.

b. The patient should keep the elbows at the sides, prevent pressure on the axillae to avoid damage to nerves and circula- tion, extend the injured leg to prevent weight bearing when rising from a chair, and advance the unaffected leg first when climbing stairs.

A nurse is using the FOUR Coma Scale to assess the neuro- logic status of a patient following surgery to remove a brain tumor. The nurse rates the patient as M2 for motor response. What condition does this number represent? a. Localizing to pain b. Flexion response to pain c. Extension response to pain d. No response to pain

b. To assess motor response, patients are asked to make a peace sign, a fist, and show thumbs up. Patients are scored as follows: M4 M3 M2 M1 M0 Thumbs-up, fist, or peace sign Localizing to pain Flexion response to pain Extension response to pain No response to pain

Which phrase describes a purpose of the ANA's Nursing's Social Policy Statement? Select all that apply. a. To describe the nurse as a dependent caregiver b. To provide standards for nursing educational programs c. To define the scope of nursing practice d. To establish a knowledge base for nursing practice e. To describe nursing's social responsibility f. To regulate nursing research

c, d, e. The ANA Social Policy Statement (2010) describes the social context of nursing, a definition of nursing, the knowledge base for nursing practice, the scope of nursing practice, standards of professional nursing practice, and the regulation of professional nursing.

A nurse is assisting a patient to empty and change an ostomy appliance. When the procedure is finished, the nurse notes that the stoma is protruding into the bag. What would be the nurse's first action in this situation? a. Reassure the patient that this is a normal finding with a new ostomy. b. Notify the primary care provider that the stoma is prolapsed. c. Have the patient rest for 30 minutes to see if the prolapse resolves. d. Remove the appliance and redo the procedure using a larger appliance.

c. If the stoma is protruding into the bag after changing the appliance on an ostomy, the nurse should have the patient rest for 30 minutes. If the stoma is not back to normal size within that time, notify the physician. If the stoma stays prolapsed, it may twist, resulting in impaired circulation to the stoma.

A nurse is practicing as a nurse-midwife in a busy OB-GYN office. Which degree in nursing is necessary to practice at this level? a. LPN b. ADN c. BSN d. MSN

d. A master's degree (MSN) prepares advanced practice nurses. Many master's graduates gain national certification in their specialty area, for example, as family nurse practitioners (FNPs) or nurse midwives.

A nurse is teaching parents of preschoolers what type of behavior to expect from their children based on developmen- tal theories. Which statements describe this stage of develop- ment? Select all that apply. a. According to Freud, the child is in the phallic stage. b. According to Erikson, the child is in the trust versus mis- trust stage. c. According to Havighurst, the child is learning to get along with others. d. According to Fowler, the child imitates religious behavior of others. e. According to Kohlberg, the child defines satisfying acts as right. f. According to Havighurst, the child is achieving gender- specific roles.

a, d, e. According to Freud, the child is in the phallic stage. According to Fowler, the child imitates religious behavior of others. According to Kohlberg, the child defines satisfying acts as right. According to Erikson, the child is in the initia- tive versus guilt stage. According to Havighurst, the child is learning sex differences, forming concepts, and getting ready to read. According to Havighurst, the adolescent, not the preschooler, is achieving gender-specific social roles.

A patient complains of severe abdominal pain. When assess- ing the vital signs, the nurse would not be surprised to find what assessments? Select all that apply. a. An increase in the pulse rate b. A decrease in body temperature c. A decrease in blood pressure d. An increase in respiratory depth e. An increase in respiratory rate f. An increase in body temperature

a, e. The pulse often increases when a person is experiencing pain. Pain does not affect body temperature and may increase (not decrease) blood pressure. Acute pain may increase res- piratory rate but decrease respiratory depth.

Following a fall that left an elderly male patient temporarily bedridden, the nurse is using the SPICES assessment tool to evaluate him for cascade iatrogenesis. Which are correct aspects of this tool? Select all that apply. a. S - Senility b. P - Problems with feeding c. I - Irritableness d. C - Confusion e. E - Edema of the legs f. S - Skin breakdown

b, d, f. The SPICES acronym is used to identify common problems in older adults and stands for: S - Sleep disorders P - Problems with eating or feeding I - Incontinence C - Confusion E - Evidence of falls S - Skin breakdown (Fulmer & Wallace, 2012).

A nurse's neighbor tells the nurse, "I have a high temperature, feel awful, and I am not going to work." What stage of illness behavior is the neighbor exhibiting? a. Experiencing symptoms b. Assuming the sick role c. Assuming a dependent role d. Achieving recovery and rehabilitation

b. When people assume the sick role, they define themselves as ill, seek validation of this experience from others, and give up normal activities. In stage 1: experiencing symp- toms, the first indication of an illness usually is recognizing one or more symptoms that are incompatible with one's per- sonal definition of health. The stage of assuming a depend- ent role is characterized by the patient's decision to accept the diagnosis and follow the prescribed treatment plan. In the achieving recovery and rehabilitation role, the person gives up the dependent role and resumes normal activities and responsibilities.

A school nurse is teaching parents about home safety and fires. What information would be accurate to include in the teaching plan? Select all that apply. a. 60% of U.S. fire deaths occur in the home. b. Most fatal fires occur when people are cooking. c. Most people who die in fires die of smoke inhalation. d. Over 1/3 of fire deaths occur in a home without a smoke detector. e. Fires are more likely to occur in homes without electricity or gas. f. More fires occur in homes occupied by single parents.

c, d, e. Of all fire deaths in the United States, 85% occur in the home (CDC, 2011a). Most fatal home fires occur while people are sleeping, and most people who die in house fires die of smoke inhalation rather than burns. More than one-third of home fire deaths occur in a home without a smoke detector (CDC, 2011a). People with limited financial resources should be asked about how they heat their house because the electricity or gas may have been turned off and space or kerosene heaters, wood stoves, or a fireplace may be the sole source of heat. Being a single parent is not a risk factor for fire occurrences.

After inspecting the skin of a patient, the nurse documents the presence of a skin lesion as a palpable solid mass measured at 1 cm. What types of skin lesions might this describe? Select all that apply. a. Macule b. Patch c. Plaque d. Nodule e. Bulla f. Pustule

c, d. Plaque and nodules are palpable, elevated, solid masses that may measure 1 cm. Macules and patches are circum- scribed, flat, nonpalpable changes in skin color. Macules are less than or equal to 1 cm and patches are greater than 1 cm. Bulla and pustules are circumscribed, superficial skin eleva- tions formed by free fluids in a cavity with skin layers. Bulla are greater than 0.5 cm and pustules are filled with pus.

A patient's visual acuity is assessed as 20/40 in both eyes using the Snellen chart. The nurse interprets this finding as: a. The patient can see twice as well as normal. b. The patient has double vision. c. The patient has less than normal vision. d. The patient has normal vision.

c. Normal vision is 20/20. A finding of 20/40 would mean that a patient has less than normal vision.

A 33-year-old businessperson is now in counseling attempt- ing to deal with a long-repressed history of sexual abuse by her father. "I guess I should feel satisfied with what I've achieved in life, but I'm never content, and nothing I achieve makes me feel good about myself . . . I hate my father for making me feel like I'm no good. This is an awful way to live." a. Personal Identity Disturbance b. Body Image Disturbance c. Self-Esteem Disturbance d. Altered Role Performance

c. This patient's self-concept disturbance is mainly one of devaluing herself and thinking that she is no good. This is a self-esteem disturbance.

Following assessment of an obese adolescent, a nurse consid- ers nursing diagnoses for the patient. Which diagnosis would be most appropriate? a. Risk for injury b. Risk for delayed development c. Social isolation d. Disturbed body image

d. Adolescents who are obese are at high risk for a disturbed body image. Risk for injury would be appropriate for a risk taker, a risk factor for delayed development may be ADHD, and social isolation may occur with low self-esteem.

A nurse enters a patient's room and examines the patient's IV fluids and cardiac monitor. The patient states, "Well, I haven't seen you before. Who are you?" What is the nurse's best response? a. "I'm just the IV therapist checking your IV." b. "I've been transferred to this division and will be caring for you." c. "I'm sorry, my name is John Smith and I am your nurse." d. "My name is John Smith, I am your nurse and I'll be car- ing for you until 11 p.m."

d. The nurse should identify himself, be sure the patient knows what will be happening, and the time period he will be with his patient.

Read the following scenario and identify the term for the characteristics of patient data that are numbered below. Place your answers on the lines provided. The nurse is conducting an initial assessment of a 79-year old female patient admitted to the hospital with a diagnosis of dehydration. The nurse: (1) uses clinical reasoning to identify the need to perform a comprehensive assessment and gather the appropriate patient data, (2) first asks the patient about the most important details leading up to her diagnosis, (3) collects as much information as possible to understand the patient's health problems, (4) collects the patient data in an organized manner, (5) verifies that the data obtained is pertinent to the patient care plan, and (6) records the data according to agency policy. (1) (2) (3) (4) (5) (6)

(1) Purposeful: The nurse identifies the purpose of the nursing assessment (comprehensive) and gathers the appropriate data. (2) Prioritized: The nurse gets the most important informa- tion first. (3) Complete: The nurse gathers as much data as possible to understand the patient health problem and develop a plan of care. (4) Systematic: The nurse gathers the information in an organized manner. (5) Factual and accurate: The nurse verifies that the informa- tion is reliable. (6) Recorded in a standard manner: The nurse records the data according to agency policy so that all caregivers can easily access what is learned.

Read the following patient scenario and identify the step of the nursing process represented by each numbered and bold- faced nursing activity. Annie seeks the help of the nurse in the student health clinic because she suspects that her roommate, Angela, suffered date rape. She is concerned because Angela chose not to report the rape and does not seem to be coping well. (1) After talking with Annie, the nurse learns that although Angela blurted out that she had been raped when she first came home, since then she has refused verbalization about the rape ("I don't want to think or talk about it"), has stopped attending all college social activities (a marked change in behavior), and seems to be having nightmares. After analyzing the data, the nurse believes that Angela might be experiencing (2) rape- trauma syndrome: silent reaction. Fortunately, Angela trusts Annie and is willing to come to the student health center for help. A conversation with Angela confirms the nurse's suspicions, and problem identification begins. The nurse talks further with Angela (3) to develop some treatment goals and formulate outcomes. The nurse also begins to think about the types of nursing interventions most likely to yield the desired outcomes. In the initial meeting with Angela, (4) the nurse encourages her expression of feelings and helps her to identify personal coping strategies and strengths. The nurse and Angela decide to meet in 1 week (5) to assess her progress toward achieving targeted outcomes. If she is not making progress, the plan of care might need to be modified. (1) (2) (3) (4) (5) _____________________________________________ _____________________________________________ _____________________________________________ _____________________________________________ _____________________________________________

(1) is an illustration of assessing: the collection of patient data. (2) is an illustration of the identification of a nurs- ing diagnosis: a health problem that independent nursing intervention can resolve. (3) is an illustration of planning: outcome identification and related nursing interventions. (4) is an illustration of implementing: carrying out the plan of care. (5) is an illustration of evaluating: measuring the extent to which Angela has achieved targeted outcomes.

A nurse caring for patients in the PACU teaches a novice nurse how to assess and document wound drainage. Which statements accurately describe a characteristic of wound drainage? Select all that apply. a. Serous drainage is composed of the clear portion of the blood and serous membranes. b. Sanguineous drainage is composed of a large number of red blood cells and looks like blood. c. Bright red sanguineous drainage indicates fresh bleeding and darker drainage indicates older bleeding. d. Purulent drainage is composed of white blood cells, dead tissue, and bacteria. e. Purulent drainage is thin, cloudy, and watery and may have a musty or foul odor. f. Serosanguineous drainage can be dark yellow or green depending on the causative organism.

a, b, c, d. Serous drainage is composed primarily of the clear, serous portion of the blood and serous membranes. Serous drainage is clear and watery. Sanguineous drainage consists of large numbers of red blood cells and looks like blood. Bright-red sanguineous drainage is indicative of fresh bleeding, whereas darker drainage indicates older bleeding. Purulent drainage is made up of white blood cells, liquefied dead tissue debris, and both dead and live bacteria. Purulent drainage is thick, often has a musty or foul odor, and varies in color (such as dark yellow or green), depending on the causative organism. Serosanguineous drainage is a mixture of serum and red blood cells. It is light pink to blood tinged.

One of the four broad aims of nursing practice is to restore health. Which examples of nursing interventions reflect this goal? Select all that apply. a. A nurse counsels adolescents in a drug rehabilitation program. b. A nurse performs range-of-motion exercises for a patient on bedrest. c. A nurse shows a diabetic patient how to inject insulin. d. A nurse recommends a yoga class for a busy executive. e. A nurse provides hospice care for a patient with end-stage cancer. f. A nurse teaches a nutrition class at a local high school.

a, b, c. Activities to restore health focus on the individual with an illness and range from early detection of a disease to rehabilitation and teaching during recovery. These activities include drug counseling, teaching patients how to administer their medications, and performing range-of-motion exercises for bedridden patients. Recommending a yoga class for stress reduction is a goal of preventing illness, and teaching a nutri- tion class is a goal of promoting health. A hospice care nurse helps to facilitate coping with disability and death.

A nurse caring for patients in a critical care unit knows that providing good oral hygiene is an essential part of nursing care. What are some of the benefits of providing this care? Select all that apply. a. It promotes the patient's sense of well-being. b. It prevents deterioration of the oral cavity. c. It contributes to decreased incidence of aspiration pneumonia. d. It eliminates the need for flossing. e. It decreases oropharyngeal secretions. f. It compensates for an inadequate diet.

a, b, c. Adequate oral hygiene is essential for promoting the patient's sense of well-being and preventing deterioration of the oral cavity. Diligent oral hygiene care can also improve oral health and limit the growth of pathogens in oropharyn- geal secretions, decreasing the incidence of aspiration pneu- monia and other systemic diseases (AACN, 2006; AACN, 2010). Oral care does not eliminate the need for flossing, decrease oropharyngeal secretions, or compensate for poor nutrition.

A high school nurse is counseling parents of teenagers who are beginning high school. Which issues would be priority topics of discussion for this age group? Select all that apply. a. The influence of peer groups b. Bullying c. Water safety d. Eating disorders e. Risk taking behavior f. Immunizations

a, b, d, e. Appropriate topics of discussion for parents of ado- lescents include peer groups, bullying, eating disorders, and risk-taking behaviors. Immunizations would be appropriate for parents of children from infants to school-age, and water safety should be taught in the preschool years.

Which assessments and interventions should the nurse consider when performing tracheal suctioning? Select all that apply. a. Closely assess the patient before, during, and after the procedure. b. Hyperoxygenate the patient before and after suctioning. c. Limit the application of suction to 20 to 30 seconds. d. Monitor the patient's pulse frequently to detect potential effects of hypoxia and stimulation of the vagus nerve. e. Use an appropriate suction pressure (80-150 mm Hg). f. Insert the suction catheter no further than 1 cm past the length of the tracheal or endotracheal tube.

a, b, d, e. Close assessment of the patient before, during, and after the procedure is necessary to limit negative effects. Risks include hypoxia, infection, tracheal tissue damage, dysrhythmias, and atelectasis. The nurse should hyperoxy- genate the patient before and after suctioning and limit the application of suction to 10 to 20 seconds. The nurse should also take the patient's pulse frequently to detect potential effects of hypoxia and stimulation of the vagus nerve. Using an appropriate suction pressure (80-150 mm Hg) will help prevent atelectasis related to the use of high negative pressure. Research suggests that insertion of the suction catheter should be limited to a predetermined length (no further than 1 cm past the length of the tracheal or endotra- cheal tube) to avoid tracheal mucosal damage, including epithelial denudement, loss of cilia, edema, and fibrosis (Roman, 2005).

The nurse is cleaning an open abdominal wound that has unapproximated edges. What are accurate steps in this procedure? Select all that apply. a. Use standard precautions or transmission-based precau- tions when indicated. b. Moisten a sterile gauze pad or swab with the prescribed cleansing agent and squeeze out excess solution. c. Clean the wound in full or half circles beginning on the outside and working toward the center. d. Work outward from the incision in lines that are parallel to it from the dirty area to the clean area. e. Clean to at least one inch beyond the end of the new dress- ing if one is being applied. f. Clean to at least three inches beyond the wound if a new dressing is not being applied.

a, b, e. The correct procedure for cleaning a wound with unapproximated edges is: (1) use standard precautions and appropriate transmission-based precautions when indicated, (2) moisten sterile gauze pad or swab with prescribed cleans- ing agent and squeeze out excess solution, (3) use a new swab or gauze for each circle, (4) clean the wound in full or half circles beginning in the center and working toward the outside, (5) clean to at least one inch beyond the end of the new dressing, and (6) clean to at least two inches beyond the wound margins if a dressing is not being applied.

A nurse working in a hospital includes abdominal assessment as part of patient assessment. In which patients would a nurse expect to find decreased or absent bowel sounds after listen- ing for 5 minutes? Select all that apply. a. A patient diagnosed with peritonitis b. A patient who is on prolonged bedrest c. A patient who has diarrhea d. A patient who has gastroenteritis e. A patient who has an early bowel obstruction f. A patient who has paralytic ileus caused by surgery

a, b, f. Decreased or absent bowel sounds—evidenced only after listening for 5 minutes (Jensen, 2011)—signify the absence of bowel motility, commonly associated with peritoni- tis, paralytic ileus, and/or prolonged immobility. Hyperactive bowel sounds indicate increased bowel motility, commonly caused by diarrhea, gastroenteritis, or early bowel obstruction.

A nurse caring for older adults in a long-term care facility knows that several physical changes occur in the aging adult. Which characteristics best describe these changes? Select all that apply. a. Fatty tissue is redistributed. b. The skin is drier and wrinkles appear. c. Cardiac output increases. d. Muscle mass increases. e. Hormone production increases. f. Visual and hearing acuity diminishes.

a, b, f. Physical changes occurring with aging include these: fatty tissue is redistributed, the skin is drier and wrinkles appear, and visual and hearing acuity diminishes. Cardiac output decreases, muscle mass decreases, and hormone production decreases, causing menopause or andropause.

The three types of responses to pain are physiologic, behavioral, and affective. Which are examples of behavioral responses to pain? Select all that apply. a. A patient cradles a wrist that was injured in a car accident. b. A child is moaning and crying due to a stomachache. c. A patient's pulse is increased following a myocardial infarction. d. A patient in pain strikes out at a nurse who attempts to bathe him. e. A patient who has chronic cancer pain is depressed and withdrawn. f. A child pulls away from a nurse trying to give him an injection.

a, b, f. Protecting or guarding a painful area, moaning and crying, and moving away from painful stimuli are behavioral responses. Examples of a physiologic or involuntary response would be increased blood pressure or dilation of the pupils. Affective responses, such as anger, withdrawal, and depres- sion, are psychological in nature.

In addition to standard precautions, the nurse would initi- ate droplet precautions for which patients? Select all that apply. a. A patient diagnosed with rubella b. A patient diagnosed with diptheria c. A patient diagnosed with varicella d. A patient diagnosed with tuberculosis e. A patient diagnosed with MRSA f. An infant diagnosed with adenovirus infection

a, b, f. Rubella, diphtheria, and adenovirus infection are illnesses transmitted by large-particle droplets and require droplet precautions in addition to standard precautions. Air- borne precautions are used for patients who have infections spread through the air with small particles, for example, tuberculosis, varicella, and rubeola. Contact precautions are used for patients who are infected or colonized by a multidrug-resistant organism (MDRO), such as MRSA.

The nurse collects objective and subjective data when con- ducting patient assessments. Which patient conditions are examples of subjective data? Select all that apply. a. A patient tells the nurse that she is feeling nauseous. b. A patient's ankles are swollen. c. A patient tells the nurse that she is nervous about her test results. d. A patient complains of having a rash on her arm that is itchy. e. A patient rates his pain as a 7 on a scale of 1 to 10. f. A patient vomits after eating supper.

a, c, d, e. Subjective data are information perceived only by the affected person; these data cannot be perceived or verified by another person. Examples of subjective data are feeling nervous, nauseated, itchy, or chilly and experiencing pain. Objective data are observable and measurable data that can be seen, heard, or felt by someone other than the person expe- riencing them. Examples of objective data are an elevated temperature reading (e.g., 101°F), edema, and vomiting.

The rectal temperature, a core temperature, is considered to be one of the most accurate routes. In which cases would taking a rectal temperature be contraindicated? Select all that apply. a. A newborn who has hypothermia b. A child who has pneumonia c. An older patient who is post myocardial infarction (heart attack) d. A teenager who has leukemia e. A patient receiving erythropoietin to replace red blood cells f. An adult patient who is newly diagnosed with pancreatitis

a, c, d, e. The rectal site should not be used in newborns, children with diarrhea, and in patients who have undergone rectal surgery. The insertion of the thermometer can slow the heart rate by stimulating the vagus nerve, thus patients post-MI should not have a rectal temperature taken. Assessing a rectal temperature is also contraindicated in patients who are neutropenic (have low white blood cell counts, such as in leukemia), in patients who have certain neurologic disorders, and in patients with low platelet counts.

A nurse is caring for patients in an isolation ward. In which situations would the nurse appropriately use an alcohol- based handrub to decontaminate the hands? Select all that apply. a. The nurse is providing a bed bath for a patient. b. The nurse has visibly soiled hands after changing the bed- ding of a patient. c. The nurse removes gloves when patient care is completed. d. The nurse is inserting a urinary catheter for a female patient. e. The nurse is assisting with a surgical placement of a cardiac stent. f. The nurse removes old magazines from a patient's table.

a, c, d, f. It is recommended to use an alcohol-based handrub in the following situations: before direct contact with patients; after direct contact with patient skin; after contact with body fluids if hands are not visibly soiled; after remov- ing gloves; before inserting urinary catheters, peripheral vascular catheters, or invasive devices that do not require surgical placement; before donning sterile gloves prior to an invasive procedure; if moving from a contaminated body site to a clean body site; and after contact with objects contami- nated by the patient.

A nurse working with adolescents in a juvenile detention center teaches parents about behaviors that place adolescents at high risk for injury. Which statements accurately describe these risks? Select all that apply. a. Each year, underage drinking claims the lives of approxi- mately 5,000 individuals under the age of 21. b. Approximately one in three high school students reported using some type of tobacco product. c. The CDC (2012i) lists motor vehicle accidents as the number-one cause of death for adolescents. d. Marijuana use among teenagers has been on the increase and the abuse of prescription medication and OTC drugs has remained at a high level. e. Homicide rates for adolescents are high, and youths aged 10-19 years committed almost 500 suicides using firearms. f. As many as 30% of children are bullied during their school years and cyber bullying is even more damaging to children.

a, c, d. Each year, underage drinking claims the lives of approximately 5,000 people under the age of 21. The CDC (2012j) lists motor vehicle accidents as the number one cause of death for adolescents. Marijuana use among teenagers has been on the increase and the abuse of prescription medication and OTC drugs has remained at a high level. Approximately one in five (20%) high school students reported using some type of tobacco product. Homicide rates for youths using firearms are higher than any other age group and the most recent statistics indicate that youths aged 10-19 years com- mitted almost 1,500 suicides using firearms (Kagler, Annest, Kresnow & Mercy, 2011). According to the American Acad- emy of Child & Adolescent Psychiatry, as many as 50% of children are bullied during their school years and some experts believe that cyber bullying is more dangerous and damaging to children than bullying that occurs in the schoolyard.

A nurse is performing patient care for a severely ill patient who has cancer. Which nursing interventions are likely to assist a severely ill patient with cancer to maintain a positive sense of self? Select all that apply. a. The nurse makes a point to address the patient by name upon entering the room. b. The nurse avoids fatiguing the patient by performing all procedures in silence. c. The nurse performs care in a manner that respects the patient's privacy and sensibilities. d. The nurse offers the patient a simple explanation before moving her in any way. e. The nurse ignores negative feelings from the patient since they are part of the grieving process. f. The nurse avoids conversing with the patient about her life, family, and occupation.

a, c, d. When assisting the patient to maintain a positive sense of self, the nurse should address the patient by name when entering the room; perform care in a manner that respects the patient's privacy; offer a simple explanation before moving the patient's body in any way; acknowledge the patient's status, role, and individuality; and converse with the patient about the patient's life experiences.

A nurse caring for a patient with chronic obstructive pulmonary disease (COPD) knows that hypoxia may occur in patients with respiratory problems. What are signs of this serious condition? Select all answers that apply. a. Dyspnea b. Hypotension c. Small pulse pressure d. Decreased respiratory rate e. Pallor f. Increased pulse rate

a, c, e, f. If a problem exists in ventilation, respiration, or per- fusion, hypoxia may occur. Hypoxia is a condition in which an inadequate amount of oxygen is available to cells. The most common symptoms of hypoxia are dyspnea (difficulty breathing), an elevated blood pressure with a small pulse pressure, increased respiratory and pulse rates, pallor, and cyanosis.

A nurse is caring for a patient who has a nasogastric tube in place for gastric decompression. Which nursing actions are appropriate when irrigating a nasogastric tube connected to suction? Select all that apply. a. Draw up 30 mL of saline solution into the syringe. b. Unclamp the suction tubing near the connection site to instill solution. c. Place the tip of the syringe in the tube to gently insert saline solution. d. Place syringe in the blue air vent of a Salem sump or double-lumen tube. e. After instilling irrigant, hold the end of the NG tube over an irrigation tray. f. Observe for return flow of NG drainage into an available container.

a, c, e, f. The nurse irrigating a nasogastric tube connected to suction should draw up 30 mL of saline solution (or amount indicated in the order or policy) into the syringe, clamp the suction tubing near the connection site to protect the patient from leakage of NG drainage, place the tip of the syringe in the tube to gently insert the saline solution, then place the syringe in the drainage port, not in the blue air vent of a Salem sump or double-lumen tube (the blue air vent acts to decrease pressure built up in the stomach when the Salem sump is attached to suction). After instilling irrigant, hold the end of the NG tube over an irrigation tray or emesis basin, and observe for return flow of NG drainage into an available container.

A nurse who is changing dressings of postoperative patients in the hospital documents various phases of wound healing on the patient charts. Which statements accurately describe these stages? Select all that apply. a. Hemostasis occurs immediately after the initial injury. b. A liquid called exudate is formed during the proliferation phase c. White blood cells move to the wound in the inflammatory phase. d. Granulation tissue forms in the inflammatory phase. e. During the inflammatory phase, the patient has generalized body response. f. A scar forms during the proliferation phase.

a, c, e. Hemostasis occurs immediately after the initial injury and exudate occurs in this phase due to the leaking out of plasma and blood components into the injured area. White blood cells, predominantly leukocytes and macrophages, move to the wound in the inflammatory phase to ingest bac- teria and cellular debris. During the inflammatory phase, the patient has a generalized body response, including a mildly elevated temperature, leukocytosis (increased number of white blood cells in the blood), and generalized malaise. New tissue, called granulation tissue, forms the foundation for scar tissue development in the proliferation phase. New collagen continues to be deposited in the maturation phase, which forms a scar.

A new nurse who is being oriented to the subacute care unit is expected to follow existing standards when providing patient care. Which nursing actions are examples of these standards? Select all that apply. a. Monitoring patient status every hour b. Using intuition to troubleshoot patient problems c. Turning a patient on bed rest every 2 hours d. Becoming a nurse mentor to a student nurse e. Administering pain medication ordered by the physician f. Becoming involved in community nursing events

a, c, e. Standards are the levels of performance accepted and expected by the nursing staff or other health care team mem- bers. They are established by authority, custom, or consent.

A nurse is providing foot care for patients in a long-term care facility. Which actions are recommended guidelines for this procedure? Select all that apply. a. Bathe the feet thoroughly in a mild soap and tepid water solution. b. Soak the feet in warm water and bath oil. c. Dry feet thoroughly, including the area between the toes. d. Use an alcohol rub if the feet are dry. e. Use an antifungal foot powder if necessary to prevent fungal infections. f. Cut the toenails at the lateral corners when trimming the nail.

a, c, e. The following are recommended guidelines for foot care: bathe the feet thoroughly in a mild soap and tepid water solution; dry feet thoroughly, including the area between the toes; and use an antifungal foot powder if necessary to prevent fungal infections. The nurse should avoid soaking the feet, use moisturizer if the feet are dry, and avoid digging into or cut- ting the toenails at the lateral corners when trimming the nails.

A nurse caring for patients in an extended-care facility performs regular assessments of the patients' urinary func- tioning. Which patients would the nurse screen for urinary retention? Select all that apply. a. A 78-year-old male patient diagnosed with an enlarged prostate b. An 83-year-old female patient who is on bedrest c. A 75-year-old female patient who is diagnosed with vagi- nal prolapse d. An 89-year-old male patient who has dementia e. A 73-year-old female patient who is taking antihistamines to treat allergies f. A 90-year-old male patient who has difficulty walking to the bathroom

a, c, e. Urinary retention occurs when urine is produced nor- mally but is not excreted completely from the bladder. Factors associated with urinary retention include medications such as antihistamines, an enlarged prostate, or vaginal prolapse. Being on bedrest, having dementia, and having difficulty walking to the bathroom may place patients at risk for urinary incontinence.

A nurse is counseling parents attending a parent workshop on how to build self-esteem in their children. Which teaching points would the nurse include to help parents achieve this goal? Select all that apply. a. Teach the parents to reinforce their child's positive qualities. b. Teach the parents to overlook occasional negative behavior. c. Teach parents to ignore neutral behavior that is a matter of personal preference. d. Teach parents to listen and "fix things" for their children. e. Teach parents to describe the child's behavior and judge it. f. Teach parents to let their children practice skills and make it safe to fail.

a, c, f. The nurse should include the following teaching points for parents: (1) reinforce their child's positive qualities; (2) address negative qualities constructively; (3) ignore neutral behavior that is a matter of taste, prefer- ence, or personal style; (4) don't feel they have to "fix things" for their children; (5) describe the child's behavior in a nonjudgmental manner; and (6) let their child know what to expect, practice the necessary skills, be patient, and make it safe to fail.

A nurse is providing range-of-motion exercises for a 53-year- old female patient who is recovering from a stroke. During the session, the patient complains that she is "too tired to go on." What would be priority nursing actions for this patient? Select all that apply. a. Stop performing the exercises. b. Decrease the number of repetitions performed. c. Re-evaluate the nursing plan of care. d. Move to the patient's other side to perform exercises. e. Encourage the patient to finish the exercises and then rest. f. Assess the patient for other symptoms.

a, c, f. When a patient complains of fatigue during range-of- motion exercises, the nurse should stop the activity, re-evalu- ate the nursing plan of care, and assess the patient for further symptoms. The exercises could then be scheduled for times of the day when the patient is feeling more rested, or spaced out at different times of the day.

A nurse is caring for a 56-year-old male patient diagnosed with bladder cancer who has a urinary diversion. Which actions would the nurse take when caring for this patient? Select all that apply a. Measure the patient's fluid intake and output. b. Keep the skin around the stoma moist. c. Empty the appliance frequently. d. Report any mucous in the urine to the primary care provider. e. Encourage the patient to look away when changing the appliance. f. Monitor the return of intestinal function and peristalsis.

a, c, f. When caring for a patient with a urinary diversion, the nurse should measure the patient's fluid intake and output to monitor fluid balance, change the appliance frequently, moni- tor the return of intestinal function and peristalsis, keep the skin around the stoma dry, watch for mucous in the urine as a normal finding, and encourage the patient to participate in care and look at the stoma.

Nurses performing skin assessments on patients must pay careful attention to cleanliness, color, texture, temperature, turgor, moisture, sensation, vascularity, and lesions. Which guidelines should nurses follow when performing these assessments? Select all that apply. a. Compare bilateral parts for symmetry. b. Proceed in a toe-to-head systematic manner. c. Use standard terminology to report and record findings. d. Do not allow data from the nursing history to direct the assessment. e. Document only skin abnormalities on the patient record. f. Perform the appropriate skin assessment when risk factors are identified.

a, c, f. When performing a skin assessment, the nurse should compare bilateral parts for symmetry, use standard terminol- ogy to report and record findings, and perform the appropriate skin assessment when risk factors are identified. The nurse should proceed in a head-to-toe systematic manner, and allow data from the nursing history to direct the assessment. When documenting a physical assessment of the skin, the nurse should describe exactly what is observed or palpated, includ- ing appearance, texture, size, location or distribution, and characteristics of any findings.

A nurse is using critical pathway methodology for choosing interventions for a patient who is receiving chemotherapy for breast cancer. Which nursing actions are characteristics of this system being used when planning care? Select all that apply. a. The nurse uses a minimal practice standard and is able to alter care to meet the patient's individual needs. b. The nurse uses a binary decision tree for stepwise assessment and intervention. c. The nurse is able to measure the cause-and-effect relationship between pathway and patient outcomes. d. The nurse uses broad, research-based practice recommen- dations that may or may not have been tested in clinical practice. e. The nurse uses preprinted provider orders used to expedite the order process after a practice standard has been vali- dated through research. f. The nurse uses a decision tree that provides intense speci- ficity and no provider flexibility.

a, c. A critical pathway represents a sequential, interdis- ciplinary, minimal practice standard for a specific patient population that provides flexibility to alter care to meet individualized patient needs. It also offers the ability to measure a cause-and-effect relationship between pathway and patient outcomes. An algorithm is a binary decision tree that guides stepwise assessment and intervention with intense specificity and no provider flexibility. Guidelines are broad, research-based practice recommendations that may or may not have been tested in clinical practice, and an order set is a preprinted provider order used to expedite the order process after a practice standard has been validated through analytical research.

During a nursing staff meeting, the nurses resolve a problem of delayed documentation by agreeing unanimously that they will make sure all vital signs are reported and charted within 15 minutes following assessment. This is an example of which characteristics of effective communication? Select all that apply. a. Group decision making b. Group leadership c. Group power d. Group identity e. Group patterns of interaction f. Group cohesiveness

a, d, e, f. Solving problems involves group decision making; ascertaining that the staff completes a task on time and that all members agree the task is important is a characteristic of group identity; group patterns of interaction involve hon- est communication and member support; and cohesiveness occurs when members generally trust each other, have a high commitment to the group, and a high degree of cooperation. Group leadership occurs when groups use effective styles of leadership to meet goals; with group power, sources of power are recognized and used appropriately to accomplish group outcomes.

A nurse is assessing the vital signs of patients who presented at the emergency department. Based on the knowledge of age-related variations in normal vital signs, which patients would the nurse document as having a normal vital sign? Select all that apply. a. A 4-month old infant whose temperature is 38.1°C (100.5°F) b. A 3-year old whose blood pressure is 118/80 c. A 9-year old whose temperature is 39°C (102.2°F) d. An adolescent whose pulse rate is 70 bpm e. An adult whose respiratory rate is 20 bpm f. A 72-year old whose pulse rate is 42 bpm

a, d, e, f. The normal temperature range for infants is 37.1°C to 38.1°C (98.7°F-100.5°F). The normal pulse rate for an adolescent is 55 to 105. The normal respiratory rate for an adult is 12 to 20 bpm and the normal pulse for an older adult is 40 to 100 bpm. The normal blood pressure for a toddler is 89/46 and the normal temperature for a child is 36.8°C to 37.8°C (98.2°F-100°F; refer to Table 24-1, Age-Related Variations in Normal Vital Signs).

A nurse caring for patients in a long-term care facility is performing a functional assessment of a new patient. Which questions would the nurse ask? Select all that apply. a. Are you able to dress yourself ? b. Do you have a history of smoking? c. What is the problem for which you are seeking care? d. Do you prepare your own meals? e. Do you manage your own finances? f. Whom do you rely on for support?

a, d, e. A functional health assessment focuses on the effects of health or illness on a patient's quality of life, including the strengths of the patient and areas that need to improve. The nurse would assess the patient's ability to perform ADLs and IADLs such as dressing, grooming, preparing meals, and managing finances. A history of smoking is a lifestyle factor and the chief complaint is the reason for seeking health care, both assessed during the health history. Social networks and support persons are assessed as psychosocial factors related to the health history.

A nurse is teaching a student nurse how to cleanse the perineal area of both male and female patients. What are accurate guidelines when performing this procedure? Select all that apply. a. For male and female patients, wash the groin area with a small amount of soap and water and rinse. b. For a female patient, spread the labia and move the wash- cloth from the anal area toward the pubic area. c. For male and female patients, always proceed from the most contaminated area to the least contaminated area. d. For male and female patients, use a clean portion of the washcloth for each stroke. e. For a male patient, clean the tip of the penis first, mov- ing the washcloth in a circular motion from the meatus outward. f. In an uncircumcised male patient do not retract the fore- skin (prepuce) while washing the penis.

a, d, e. Wash and rinse the groin area (both male and female patients) with a small amount of soap and water, and rinse. For male and female patients, always proceed from the least contaminated area to the most contaminated area and use a clean portion of the washcloth for each stroke. For a male patient, clean the tip of the penis first, moving the washcloth in a circular motion from the meatus outward. For a female patient, spread the labia and move the washcloth from the pubic area toward the anal area. In an uncircumcised male patient (teenage or older), retract the foreskin (prepuce) while washing the penis.

A nurse is using the implementation step of the nursing process to provide care for patients in a busy hospital setting. Which nursing actions best represent this step? Select all that apply. a. The nurse carefully removes the bandages from a burn victim's arm. b. The nurse assesses a patient to check nutritional status c. The nurse formulates a nursing diagnosis for a patient with epilepsy. d. The nurse turns a patient in bed every 2 hours to prevent pressure ulcers. e. The nurse checks a patient's insurance coverage at the initial interview. f. The nurse checks for community resources for a patient with dementia.

a, d, f. During the implementing step of the nursing process, nursing actions planned in the previous step are carried out. The purpose of implementation is to assist the patient in achieving valued health outcomes: promote health, prevent disease and illness, restore health, and facilitate coping with altered functioning. Assessing a patient for nutritional status or insurance coverage occurs in the assessment step, and formulating nursing diagnoses occurs in the diagnosing step.

A nurse is preparing an exercise program for a 65-year-old male patient who has COPD. Which instructions would the nurse include in a teaching plan for this patient? Select all that apply. a. Instruct the patient to avoid sudden position changes that may cause dizziness. b. Recommend that the patient restrict fluid until after exercising is finished. c. Instruct the patient to push a little further beyond fatigue each session. d. Instruct the patient to avoid exercising in very cold or very hot temperatures. e. Encourage the patient to modify exercise if weak or ill. f. Recommend that the patient consume a high-carb, low protein diet.

a, d. Teaching points for exercising for a patient with COPD include avoiding sudden position changes that may cause dizziness and avoiding extreme temperatures. The nurse should also instruct the patient to provide for adequate hydration, respect fatigue by not pushing to the point of exhaustion, and avoid exercise if weak or ill. Older adults should consume a high-protein, high-calcium, and vitamin D-enriched diet.

The nurse uses blended competencies when caring for patients in a rehabilitation facility. Which examples of interventions involve cognitive skills? Select all that apply. a. The nurse uses critical thinking skills to plan care for a patient. b. The nurse correctly administers IV saline to a patient who is dehydrated. c. The nurse assists a patient to fill out an informed consent form. d. The nurse learns the correct dosages for patient pain medications. e. The nurse comforts a mother whose baby was born with Down syndrome. f. The nurse uses the proper procedure to catheterize a female patient.

a, d. Using critical thinking and learning medication dosages are cognitive competencies. Performing procedures correctly is a technical skill, helping a patient with an informed consent form is a legal/ethical issue, and comforting a patient is an interpersonal skill.

hospital for treatment of preeclampsia. The patient asks the nurse: "Why are you doing a history and physical exam when the doctor just did one?" Which statements best explain the primary reasons a nursing assessment is performed? Select all that apply. a. "The nursing assessment will allow us to plan and deliver individualized, holistic nursing care that draws on your strengths." b. "It's hospital policy. I know it must be tiresome, but I will try to make this quick!" c. "I'm a student nurse and need to develop the skill of assessing your health status and need for nursing care." d. "We want to make sure that your responses to the medical exam are consistent and that all our data are accurate." e. "We need to check your health status and see what kind of nursing care you may need." f. "We need to see if you require a referral to a physician or other health care professional.

a, e, f. Medical assessments target data pointing to patho- logic conditions, whereas nursing assessments focus on the patient's responses to health problems. The initial comprehensive nursing assessment results in baseline data that enable the nurse to make a judgment about a patient's health status, the ability to manage his or her own health care and the need for nursing. It also helps nurses plan and deliver individualized, holistic nursing care that draws on the patient's strengths and promotes optimum functioning, independence, and well-being, and enables the nurse to refer the patient to a physician or other health care professional, if indicated. The fact that this is hospital policy is a secondary reason, and although it may be true that a nurse may need to develop assessment skills, it is not the chief reason the nurse performs a nursing history and exam. The assessment is not performed to check the accuracy of the medical examination.

A nurse is caring for an adolescent with severe acne. Which recommendations would be most appropriate to include in the teaching plan for this patient? Select all that apply. a. Wash the skin twice a day with a mild cleanser and warm water b. Use cosmetics liberally to cover blackheads. c. Use emollients on the area. d. Squeeze blackheads as they appear. e. Keep hair off the face and wash hair daily. f. Avoid sun-tanning booth exposure and use sunscreen.

a, e, f. Washing the skin removes oil and debris, hair should be kept off the face and washed daily to keep oil from the hair off the face, and sunbathing should be avoided when using acne treatments. Liberal use of cosmetics and emollients can clog the pores. Squeezing blackheads is always discouraged because it may lead to infection.

A nurse performs an assessment of a family consisting of a single mother, a grandmother, and two children. Which inter- view questions directed to the single mother could the nurse use to assess the affective and coping family function? Select all that apply. a. Who is the person you depend on for emotional support? b. Who is the breadwinner in your family? c. Do you plan on having any more children? d. Who keeps your family together in times of stress? e. What family traditions do you pass on to your children? f. Do you live in an environment that you consider safe?

a,d.Thefivemajorareasoffamilyfunctionarephysical, economic, reproductive, affective and coping, and socializa- tion. Asking who provides emotional support in times of stress assesses the affective and coping function. Assessing the breadwinner focuses on the economic function. Inquiring about having more children assesses the reproductive function, ask- ing about family traditions assesses the socialization function, and checking the environment assesses the physical function.

A nurse is prioritizing the following patient diagnoses accord- ing to Maslow's hierarchy of human needs: (1) Disturbed Body Image (2) Ineffective Airway Clearance (3) Spiritual Distress (4) Impaired Social Interaction Which answer choice below lists the problems in order of highest priority to lowest priority based on Maslow's model? a. 2, 4, 1, 3 b. 3, 1, 4, 2 c. 2, 4, 3, 1 d. 3, 2, 4, 1

a. 2, 4, 1, 3. Because basic needs must be met before a person can focus on higher ones, patient needs may be prioritized according to Maslow's hierarchy: (1) physiologic needs, (2) safety needs, (3) love and belonging needs, (4) self- esteem needs, and (5) self-actualization needs. #2 is an example of a physiologic need, #4 is an example of a love and belonging need, #1 is an example of a self-esteem need, and #3 is an example of a self-actualization need.

When the initial nursing assessment revealed that a patient had not had a bowel movement for 2 days, the student nurse wrote the diagnostic label "constipation." Which of the following comments is the nurse most likely to hear from the instructor? a. "Hold on a minute . . . Nursing diagnoses should always be derived from clusters of significant data rather than from a single cue." b. "Job well done . . . you've identified this problem early and we can manage it before it becomes more acute." c. "Is this an actual or a possible diagnosis?" d. "This is a medical, not a nursing problem."

a. A data cluster is a grouping of patient data or cues that points to the existence of a patient health problem. Nursing diagnoses should always be derived from clusters of signifi- cant data rather than from a single cue. There may be a reason for the lack of a bowel movement for 2 days, or it might be this individual's normal pattern.

A nurse performing an assessment of a newborn in the neonatal unit records these findings: heart rate 85 bpm, irregular respiratory rate, normal muscle tone, weak crying, and bluish tint to skin. Using the APGAR scoring chart (see Table 18-1, p. 389) what would be the score for this newborn? a.5 b.7 c.8 d. 10

a. A newborn with a heart rate less than 100 bpm, irregular respiratory effort, normal muscle tone, weak cry, and bluish tint to the skin scores a 5 on the APGAR chart.

A registered nurse assumes the role of nurse coach to provide teaching to patients who are recovering from strokes. One example of an intervention the nurse may provide related to this role is: a. The nurse uses discovery to identify the patients' personal goals and create an agenda that will result in change. b. The nurse is the expert in providing teaching and education strategies to provide dietary and activity modifications. c. The nurse becomes a mentor to the patients and encour- ages them to create their own fitness programs. d. The nurse assumes an authoritative role to design the structure of the coaching session and support the achieve- ment of patient goals.

a. A nurse coach establishes a partnership with a patient and, using discovery, facilitates the identification of the patient's personal goals and agenda to lead to change rather than using teaching and education strategies with the nurse as the expert. A nurse coach explores the patient's readiness for coach- ing, designs the structure of a coaching session, supports the achievement of the patient's desired goals, and with the patient determines how to evaluate the attainment of patient goals.

A nurse sees a patient walking to the bathroom with a stooped gait, facial grimacing, and gasping sounds. It is important that the nurse assess the patient for: a. Pain b. Anxiety c. Depression d. Fluid volume deficit

a. A patient who presents with nonverbal communication of a stooped gait, facial grimacing, and gasping sounds is most likely experiencing pain. The nurse should clarify this nonverbal behavior.

To determine the significance of a blood pressure reading of 148/100, it is first necessary for the nurse to: a. Compare this reading to standards. b. Check the taxonomy of nursing diagnoses for a pertinent label. c. Check a medical text for the signs and symptoms of high blood pressure. d. Consult with colleagues.

a. A standard, or a norm, is a generally accepted rule, meas- ure, pattern, or model to which data can be compared in the same class or category. For example, when determining the significance of a patient's blood pressure reading, appropri- ate standards include normative values for the patient's age group, race, and illness category. Deviation from an appropri- ate norm may be the basis for writing a diagnosis.

A nurse auscultates the thorax and lungs and hears coarse, low-pitched, continuous sounds on expiration. When the patient coughs, the sounds clear up somewhat. The nurse would document these sounds as: a. Adventitious breath sounds b. Bronchovesicular breath sounds c. Vesicular breath sounds d. Bronchial sounds

a. Adventitious breath sounds are sounds not normally heard in the lungs. Bronchovesicular breath sounds are normal sounds heard on inspiration and expiration. Vesicular breath sounds are soft, low-pitched, whispering sounds; heard over most of the lung fields. Bronchial sounds are blowing, hollow sounds, auscul- tated over the larynx and trachea.

A nurse is caring for a postpartum patient who has stitches in the perineum from an episotomy (surgically planned incision to prevent vaginal tears). Which medication would the nurse most likely administer to this patient? a. A stool softener (Colace) b. An osmotic laxative (Miralax) c. A bulk-forming laxative (Metamucil) d. An emollient laxative (mineral oil)

a. Although all the choices are laxatives that would soften the stool and make it easier to expel, a stool softener, such as Colace, is the one recommended for a patient who must avoid straining. In this case, it would help to prevent disturbing the stitches in the perineum.

A nurse is assessing a 15-year-old female patient who is diagnosed with anorexia. Following the assessment, the nurse recommends that the patient meet with a nutritionist. This action best exemplifies the use of: a. Clinical judgment b. Clinical reasoning c. Critical thinking d. Blended competencies

a. Although all the options refer to the skills used by nurses in practice, the best choice is clinical judgment as it refers to the result or outcome of critical thinking or clinical reasoning—in this case the recommendation to meet with a nutritionist. Clin- ical reasoning usually refers to ways of thinking about patient care issues (determining, preventing, and managing patient problems). Critical thinking is a broad term that includes rea- soning both outside and inside of the clinical setting. Blended competencies are the cognitive, technical, interpersonal, and ethical/legal skills combined with the willingness to use them creatively and critically when working with patients.

A nurse caring for patients with bowel alterations formulates a nursing diagnosis for a patient with a new ileostomy. Which diagnosis is most appropriate? a. Disturbed Body Image b. Constipation c. Delayed Growth and Development d. Excess Fluid Volume

a. An ileostomy may cause disturbed body image due to the invasive nature of the procedure and the presence of the stoma. Constipation does not normally occur with an ileos- tomy because the drainage is liquid. Growth and development are not generally affected by the formation of an ileostomy. Excess fluid volume is unlikely to occur because the drainage is liquid and probably continual.

A patient has been in the United States only 3 months and has recently suffered the loss of her husband and job. She states that nothing feels familiar . . . "I don't know who I am supposed to be here" and she misses home (Nicaragua) terribly. Answer: a. Personal Identity Disturbance b. Body Image Disturbance c. Self-Esteem Disturbance d. Altered Role Performance

a. An unfamiliar culture, coupled with traumatic life events and loss of husband and job, result in this patient's total loss of her sense of self: "I don't know who I am supposed to be here." Her very sense of identity is at stake, not merely her body image, self-esteem, or role performance.

A nurse is caring for an 80-year-old female patient who is living in a long-term care facility. To help this patient adapt to her present circumstances, the nurse is using reminiscence as therapy. Which question would encourage reminiscence? a. "Tell me about how you celebrated Christmas when you were young." b. "Tell me how you plan to spend your time this weekend." c. "Did you enjoy the choral group that performed here yesterday? d. "Why don't you want to talk about your feelings?"

a. Asking questions about events in the past can encourage the older adult to relive and restructure life experiences.

A nurse practicing in a physician's office assesses self- concept in patients during the patient interview. Which patient is least likely to develop problems related to self- concept? a. A 55-year-old woman television news reporter undergoing a hysterectomy (removal of uterus) b. A young clergyperson whose vocal cords are paralyzed after a motorbike accident c. A 32-year-old accountant who survives a massive heart attack d. A 23-year-old model who just learned that she has breast cancer

a. Based simply on the facts given, the 55-year-old news reporter would be least likely to experience body image or role performance disturbance because she is beyond her childbearing years, and the hysterectomy should not impair her ability to report the news. The young clergyperson's inability to preach (b), the 32-year-old's massive myocardial infarction (c), and the model's breast resection (d) have much greater potential to result in self-concept problems.

The physician has ordered an indwelling catheter inserted in a hospitalized male patient. What consideration would the nurse keep in mind when performing this procedure? a. The male urethra is more vulnerable to injury during insertion. b. In the hospital, a clean technique is used for catheter inser- tion. c. The catheter is inserted 2′′ to 3′′ into the meatus. d. Since it uses a closed system, the risk for urinary tract infection is absent.

a. Because of its length, the male urethra is more prone to injury and requires that the catheter be inserted 6′′ to 8′′. This procedure requires surgical asepsis to prevent introducing bacteria into the urinary tract. The presence of an indwelling catheter places the patient at risk for a UTI.

Anexperiencednursetellsabeginningnursenottobother studying too hard, since most clinical reasoning becomes "second nature" and "intuitive" once you start practicing. What thinking below should underlie the beginning nurse's response? a. Intuitive problem solving comes with years of practice and observation, and novice nurses should base their care on scientific problem solving. b. For nursing to remain a science, nurses must continue to be vigilant about stamping out intuitive reasoning. c. The emphasis on logical, scientific, evidence-based reason- ing has held nursing back for years; it's time to champion intuitive, creative thinking! d. It's simply a matter of preference; some nurses are logi- cal, scientific thinkers, and some are intuitive, creative thinkers.

a. Beginning nurses must use nursing knowledge and scien- tific problem solving as the basis of care they give; intuitive problem solving comes with years of practice and observa- tion. If the beginning nurse has an intuition about a patient, that information should be discussed with the faculty mem- ber, preceptor, or supervisor. Answer b is incorrect because there is a place for intuitive reasoning in nursing, but it will never replace logical, scientific reasoning. Critical thinking is contextual and changes depending on the circumstances, not on personal preference.

A patient's spinal cord was severed, and he is paralyzed from the waist down. When obtaining data about this patient, which component of the sensory experience would be most important for the nurse to assess? a. Transmission of tactile stimuli b. Adequate stimulation in the environment c. Reception of visual and auditory stimuli d. General orientation and ability to follow commands

a. Below-the-waist paralysis makes the transmission of tactile stimuli a problem. Although the other options may be assessed, they are indirectly related to his paralysis and of lesser importance at this time.

A nurse is teaching parents in a parenting class about the use of car seats and restraints for infants and children. Which information is accurate and should be included in the teach- ing plan? a. Booster seats should be used for children until they are 4′9′′ tall and weigh between 80 and 100 pounds. b. Most U.S. states mandate the use of infant car seats and carriers when transporting a child in a motor vehicle. c. Infants and toddlers up to 2 years of age (or up to the maximum height and weight for the seat) should be in a front-facing safety seat. d. Children older than 6 years may be restrained using a car seat belt in the back seat.

a. Booster seats should be used for children until they are 4′9′′ tall and weigh between 80 and 100 pounds. All 50 U.S. states mandate the use of infant car seats and carriers when transporting a child in a motor vehicle. Infants and toddlers up to 2 years of age (or up to the maximum height and weight for the seat) should be in a rear-facing safety seat. Many children older than 6 years should still be in a booster seat.

A nurse is preparing a clinical outcome for a 32-year-old female runner who is recovering from a stroke that caused right-sided paresis. An example of this type of outcome is: a. After receiving 3 weeks of physical therapy, patient will demonstrate improved movement on the right side of her body. b. By 8/15/15, patient will be able to use right arm to dress, comb hair, and feed herself. c. Following physical therapy, patient will begin to gradually participate in walking/running events. d. By 8/15/15, patient will verbalize feeling sufficiently prepared to participate in running events.

a. Clinical outcomes describe the expected status of health issues at certain points in time, after treatment is complete. Functional outcomes (b) describe the person's ability to function in relation to the desired usual activities. Quality-of-life outcomes (c) focus on key factors that affect someone's ability to enjoy life and achieve personal goals. Affective outcomes (d) describe changes in patient values, beliefs, and attitudes.

A patient who is febrile may lose body heat through perspira- tion. The nurse recognizes that this is an example of what mechanism of heat loss? a. Evaporation b. Convection c. Radiation d. Conduction

a. Evaporation is the conversion of a liquid to a vapor as occurs when body fluid in the form of perspiration is vapor- ized from the skin. With convection, the heat is disseminated by motion between areas of unequal density, for example, the action of a fan blowing cool air over the body. An example of radiation (diffusion of heat by electromagnetic waves) is the body giving off heat from uncovered areas. In conduction, the heat is transferred to another object during direct contact, for example, body heat melting an ice pack.

When inspecting the skin of a patient who has cirrhosis of the liver, the nurse notes that the skin has a yellow tint. What would the nurse document related to this finding? a. Jaundice b. Cyanosis c. Erythema d. Pallor

a. Jaundice is a yellowish skin color caused by liver disease. Cyanosis is a bluish skin color caused by a cold environment or decreased oxygenation. Erythema is a reddish color caused by blushing, alcohol intake, fever, injury trauma, or infection. Pallor is a paleness caused by anemia or shock.

A charge nurse meets with staff to outline a plan to provide transcultural nursing care for patients in their health care facility. Which theorist promoted this type of caring as the central theme of nursing care, knowledge, and practice? a. Madeline Leininger b. Jean Watson c. Dorothy E. Johnson d. Betty Newman

a. Madeline Leininger's theory provides the foundations of transcultural nursing care by making caring the central theme of nursing. Jean Watson stated that nursing is concerned with promoting and restoring health, preventing illness, and car- ing for the sick. The central theme of Dorothy E. Johnson's theory is that problems arise due to disturbances in the system or subsystem or functioning below optimal level. Betty Newman proposed that humans are in constant relationship with stressors in the environment and the major concern for nursing is keeping the patient system stable through accurate assessment of these stressors.

Mr. Wright is recovering from abdominal surgery. When the nurse assists him to walk, she observes that he grimaces, moves stiffly, and becomes pale. She is aware that he has consistently refused his pain medication. What would be a priority nursing diagnosis for this patient? a. Acute Pain related to fear of taking prescribed post- operative medications b. Impaired Physical Mobility related to surgical procedure c. Anxiety related to outcome of surgery d. Risk for Infection related to surgical incision

a. Mr. Wright's immediate problem is his pain that is unre- lieved because he refuses to take his pain medication for an unknown reason. The other nursing diagnoses are plausible, but not a priority in this situation.

Nurse practice acts are established in each state of the United States to regulate nursing practice. What is a common element of every state practice act? a. Defining the legal scope of nursing practice b. Providing continuing education programs c. Determining the content covered in the NCLEX examination d. Creating institutional policies for health care practices

a. Nurse practice acts are established in each state to regulate the practice of nursing by defining the legal scope of nurs- ing practice, creating a state board of nursing to make and enforce rules and regulations, define important terms and activities in nursing, and establish criteria for the education and licensure of nurses. The acts do not determine the content covered on the NCLEX, but they do have the legal authority to allow graduates of approved schools of nursing to take the licensing examination. The acts also may determine edu- cational requirements for licensure, but do not provide the education. Institutional policies are created by the institutions themselves.

A nurse is teaching new mothers about infant care and safety. What would the nurse accurately include as a teaching point? a. Keep infants younger than 6 months out of direct sunlight. b. Use honey instead of sugar in homemade baby food. c. Place the baby on his or her stomach for sleeping. d. Keep crib rails down at all times.

a. Nurses should teach parents to keep infants younger than 6 months out of direct sunlight and cover them with protective clothing and hats. The nurse should also teach parents not to add honey or sugar to homemade baby food, to place the baby on the back for sleeping to prevent SIDS, and to keep the crib rails up at all times.

A student nurse interacting with patients on a cardiac unit recognizes the four concepts in nursing theory that determine nursing practice. Of these four, which is most important? a. Person b. Environment c. Health d. Nursing

a. Of the four concepts, the most important is the person. The focus of nursing, regardless of definition or theory, is the person.

A school nurse notices that a female adolescent student is losing weight and decides to perform a focused assessment of her nutritional status to determine if she has an eating disor- der. How should the nurse proceed? a. Perform the focused assessment. This is an independent nurse-initiated intervention. b. Request an order from Jill's physician since this is a physician-initiated intervention. c. Request an order from Jill's physician since this is a collaborative intervention. d. Request an order from the nutritionist since this is a collaborative intervention.

a. Performing a focused assessment is an independent nurse- initiated intervention, thus the nurse does not need an order from the physician or the nutritionist.

A nurse is preparing a hospitalized patient for a colonoscopy. Which nursing action is the recommended preparation for this test? a. Have the patient follow a clear liquid diet 24 to 48 hours before the test. b. Have the patient take Dulcolax and ingest a gallon of bowel cleaner on day 1. c. Prepare the patient for the use of general anesthesia during the test. d. Explain that barium contrast mixture will be given to drink before the test.

a. Preparation for a colonoscopy includes a clear liquid diet 24 to 48 hours before the test along with a 2-day bowel prep of a strong cathartic and Dulcolax on day 1 and enema on day 2 of the test, or a 1-day bowel prep that consists of inges- tion of a gallon of bowel cleanser in a short period of time. Conscious sedation, not general anesthesia, will be given for the colonoscopy. A chalky-tasting barium contrast mixture is given to drink before an upper gastrointestinal and small- bowel series of tests.

A mother tells the nurse that she is worried about her 4-year- old daughter because she is "overly attached to her father and won't listen to anything I tell her to do." What would be the nurse's best response to this parental concern? a. Tell the mother that this is normal behavior for a preschooler. b. Tell the mother that she and her family should see a counselor. c. Tell the mother that she should try to spend more time with her daughter. d. Tell the mother that her child should be tested for autism.

a. Preschoolers, according to Freud, are in the phallic stage, with the biologic focus primarily genital. The child has a sexual desire for the opposite-sex parent, but as means of defense strongly identifies with the same-sex parent. This is normal behavior for a preschooler, and the family does not need counseling or autism testing. Spending more time with the child is always a good idea, but is not the solution to this concern.

A nurse manager schedules a clinic for the staff to address common nursing interventions used in the facility and to explore how they can be performed more efficiently and effectively. What is the term for this type of theory affecting change in clinical nursing practice? a. Prescriptive theory b. Descriptive theory c. Developmental theory d. General systems theory

a. Prescriptive theories address nursing interventions and are designed to control, promote, and change clinical nursing practice. Descriptive theories describe a phenomenon, an event, a situation, or a relationship. Developmental theory outlines the process of growth and development of humans as orderly and predictable, beginning with conception and ending with death. General systems theory describes how to break whole things into parts and then to learn how the parts work together in "systems."

A nurse working in a long-term care facility uses proper patient-care ergonomics when handling and transferring patients to avoid back injury. Which action should be the focus of these preventive measures? a. Carefully assessing the patient care environment b. Using two nurses to lift a patient who cannot assist c. Wearing a back belt to perform routine duties d. Properly documenting the patient lift

a. Preventive measures should focus on careful assessment of the patient care environment so that patients can be moved safely and effectively. Using lifting teams and assistive patient handling equipment rather than two nurses to lift increases safety. The use of a back belt does not prevent back injury. The methods used for safe patient handling and move- ment should be documented but are not the primary focus of interventions related to injury prevention.

A nurse is caring for a patient who is taking phenazopyridine (Pyridium, a urinary tract analgesic). The patient questions the nurse: "My urine was bright orangish-red today; is there something wrong with me?" What would be the nurse's best response? a. "This is a normal finding when taking phenazopyridine." b. "This may be a sign of blood in the urine." c. "This may be the result of an injury to your bladder." d. "This is a sign that you are allergic to the medication and must stop it."

a. Pyridium is noted for turning the urine orange-red; the patient needs to be aware of this.

A nurse caring for elderly patients in a long-term care facility encourages an older adult to reminisce about her past life events. This life review, according to Erikson, is demonstrat- ing what developmental stage of the later adult years? a. Ego integrity b. Generativity c. Intimacy d. Initiative

a. Reminiscence during the older years of one's life provides a sense of fulfillment and purpose (ego integrity). Generativ- ity is a developmental stage of the middle adult years. Inti- macy is a developmental task of the adolescent to adult years, and initiative is a task of the preschooler to early school-age years.

In a group home in which most patients have slight to moder- ate visual or hearing impairment and some are periodically confused, what would be a nurse's first priority in caring for sensory concerns? a. Maintaining safety and preventing sensory deterioration b. Insisting that every patient participate in as many self-care activities as possible c. Emphasizing and reinforcing individual patient strengths d. Encouraging reminiscence and life review in groups

a. Safety is a basic physiologic need that must be met before higher-level needs—such as love and belonging, self-esteem, and self-actualization—can be met.

A student nurse is on a clinical rotation at a busy hospital unit. The RN in charge tells the student to change a surgical dressing on a patient while she takes care of other patients. The student has not changed dressings before and does not feel confident with the procedure. What would be the student's best response? a. Tell the RN that he or she lacks the technical competencies to change the dressing independently. b. Assemble the equipment for the procedure and follow the steps in the procedure manual. c. Ask another student nurse to work collaboratively with him or her to change the dressing. d. Report the RN to his or her instructor for delegating a task that should not be assigned to student nurses.

a. Student nurses should notify their nursing instructor or nurse mentor if they believe they lack any competencies needed to safely implement the plan of care. It is within the realm of a student nurse to change a dressing if he or she is technically prepared to do so.

When interacting with a patient, the nurse answers, "I am sure everything will be fine. You have nothing to worry about." This is an example of what type of inappropriate com- munication technique? a. Cliché b. Giving advice c. Being judgmental d. Changing the subject

a. Telling a patient that everything is going to be all right is a cliché. This statement gives false assurance and gives the patient the impression that the nurse is not interested in the patient's condition.

The nurse is assessing the pain of a neonate who is admitted to the NICU with a heart defect. Which pain assessment scale would be the best tool to use with this patient? a. CRIES scale b. COMFORT scale c. FLACC scale d. FACES scale

a. The CRIES Pain Scale is a tool intended for use with neonates and infants from 0 to 6 months. The COMFORT Scale, used to assess pain and distress in critically ill pediatric patients, relies on six behavioral and two physiologic factors that determine the level of analgesia needed to adequately relieve pain in these children. The FLACC scale (F—Faces, L—Legs, A—Activity, C—Cry, C—Consolability) was designed for infants and children from age 2 months to 7 years who are unable to validate the presence or severity of pain. The FACES scale is used for children who can compare their pain to the faces depicted on the scale.

Thirty-six hours after having surgery, a patient has a slightly elevated body temperature and generalized malaise, as well as pain and redness at the surgical site. Which intervention is most important to include in this patient's nursing care plan? a. Document the findings and continue to monitor the patient. b. Administer antipyretics, as ordered. c. Increase the frequency of assessment to every hour and notify the patient's primary care provider. d. Increase the frequency of wound care and contact the primary care provider for an antibiotic order.

a. The assessment findings are normal for this stage of heal- ing following surgery. The patient is in the inflammatory phase of the healing process, which involves a response by the immune system. This acute inflammation is characterized by pain, heat, redness, and swelling at the site of the injury (surgery, in this case). The patient also has a generalized body response, including a mildly elevated temperature, leukocyto- sis, and generalized malaise.

A nurse is caring for a male patient who had a condom cath- eter applied following hip surgery. What action would be a priority when caring for this patient? a. Preventing the tubing from kinking to maintain free urinary drainage b. Not removing the catheter for any reason c. Fastening the condom tightly to prevent the possibility of leakage d. Maintaining bedrest at all times to prevent the catheter from slipping off

a. The catheter should be allowed to drain freely through tub- ing that is not kinked. It also should be removed daily to pre- vent skin excoriation and should not be fastened too tightly or restriction of blood vessels in the area is likely. Confining a patient to bedrest increases the risk for other hazards related to immobility.

Two nurses are taking an apical-radial pulse and note a dif- ference in pulse rate of 8 beats per minute. The nurse would document this difference as which of the following? a. Pulse deficit b. Pulse amplitude c. Ventricular rhythm d. Heart arrhythmia

a. The difference between the apical and radial pulse rate is called the pulse deficit.

A student nurse tells the instructor that a patient is fine and has "no complaints." What would be the instructor's best response? a. "You made an inference that she is fine because she has no complaints. How did you validate this?" b. "She probably just doesn't trust you enough to share what she is feeling. I'd work on developing a trusting relation- ship." c. "Sometimes everyone gets lucky. Why don't you try to help another patient?" d. "Maybe you should reassess the patient. She has to have a problem—why else would she be here?"

a. The instructor is most likely to challenge the inference that the patient is "fine" simply because she is telling you that she has no problems. It is appropriate for the instruc- tor to ask how the student nurse validated this inference. Jumping to the conclusion that the patient does not trust the student nurse is premature and is an invalidated infer- ence. Answer c is wrong because it accepts the invalidated inference. Answer d is wrong because it is possible that the condition is resolving.

When a fire occurs in a patient's room, what would be the nurse's priority? a. Rescue the patient. b. Extinguish the fire. c. Sound the alarm. d. Run for help.

a. The patient's safety is always the priority. Sounding the alarm and extinguishing the fire are important after the patient is safe. Calling for help, if possible, rather than running for assistance, allows you to remain with your patient and is more appropriate.

A nurse is counseling a husband and wife who have decided that the wife will get a job so that the husband can go to pharmacy school, as he has wanted to do for some time. Their three teenagers, who were involved in the decision, are also getting jobs to buy their own clothes. The husband plans to work 12 to 16 hours weekly and states, "I was always an A student, but I may have to settle for Bs now because I don't want to neglect my family, and I need to work a few hours so that my wife won't have to work overtime." How would the nurse document the husband's self-expectations? a. Realistic and positively motivating his development b. Unrealistic and negatively motivating his development c. Unrealistic but positively motivating his development d. Realistic but negatively motivating his development

a. The patient's self-expectations are realistic, given his multiple commitments, and seem to be positively motivating his development.

A nurse is instructing a patient who is recovering from a stroke how to use a cane. Which step would the nurse include in the teaching plan for this patient? a. Support weight on stronger leg and cane and advance weaker foot forward. b. Hold the cane in the same hand of the leg with the most severe deficit. c. Stand with as much weight distributed on the cane as possible. d. Do not use the cane to rise from a sitting position, as this is unsafe.

a. The proper procedure for using a cane is to (1) stand with weight distributed evenly between the feet and cane; (2) support weight on the stronger leg and the cane and advance the weaker foot forward, parallel with the cane; (3) support weight on the weaker leg and cane and advance the stronger leg forward ahead of the cane; (4) move the weaker leg forward until even with the stronger leg and advance the cane again as in step 2. The patient should keep the cane within easy reach and use it for support to rise safely from a sitting position.

A nurse is assessing the abdomen of a patient who is experi- encing frequent bouts of diarrhea. The nurse first observes the contour of the abdomen, noting any masses, scars, or areas of distention. What action would the nurse perform next? a. Auscultate the abdomen using an orderly clockwise approach in all abdominal quadrants. b. Percuss all quadrants of the abdomen in a systematic clock- wise manner to identify masses, fluid, or air in the abdomen. c. Lightly palpate over the abdominal quadrants; first check- ing for any areas of pain or discomfort. d. Deeply palpate over the abdominal quadrants, noting mus- cular resistance, tenderness, organ enlargement, or masses.

a. The sequence for abdominal assessment proceeds from inspection, auscultation, percussion, and then palpation. Inspection and auscultation are performed before palpation because palpation may disturb normal peristalsis and bowel motility. Percussion and deep palpation are usually performed by advanced practice professionals.

A nurse caring for a patient who is hospitalized following a double mastectomy is preparing a discharge plan for the patient. Which action should be the focus of this termination phase of the helping relationship? a. Determining the progress made in achieving established goals b. Clarifying when the patient should take medications c. Reporting the progress made in teaching to the staff d. Including all family members in the teaching session

a. The termination phase occurs when the conclusion of the initial agreement is acknowledged. Discharge planning coor- dinates with the termination phase of a helping relationship. The nurse should determine the progress made in achieving the goals related to the patient's care.

A nurse is measuring the depth of a patient's puncture wound. Which technique is recommended? a. Moisten a sterile, flexible applicator with saline and insert it gently into the wound at a 90-degree angle with the tip down. b. Draw the shape of the wound and describe how deep it appears in centimeters. c. Gently insert a sterile applicator into the wound and move it in a clockwise direction. d. Insert a calibrated probe gently into the wound and mark the point that is even with the surrounding skin surface with a marker.

a. To measure the depth of a wound, the nurse should perform hand hygiene and put on gloves; moisten a sterile, flexible applicator with saline and insert it gently into the wound at a 90-degree angle with the tip down; mark the point on the swab that is even with the surrounding skin surface, or grasp the applicator with the thumb and forefinger at the point cor- responding to the wound's margin; and remove the swab and measure the depth with a ruler.

A nurse is palpating the breast of a woman during an assess- ment. Which technique is performed correctly? a. The nurse starts at the tail of Spence and moves in increas- ing smaller circles. b. The nurse uses the palms of the hands to gently compress the breast tissue against the chest wall. c. The nurse works in a counterclockwise direction and palpates from the periphery toward the areola. d. The nurse starts at the inner edge of the breast and palpates up and down the breast.

a. When palpating the breast, the nurse would palpate each quadrant of each breast in a systematic method using either the circular, wedge, or vertical strip technique and then use the pads of the first three fingers to gently compress the breast tissue against the chest wall. In the circular method, the nurse would start at the tail of Spence and move in increasing smaller circles. In the wedge method, the nurse would work in a clockwise direction and palpate from the periphery toward the areola. In the vertical strip method, the nurse would start at the outer edge of the breast and palpate up and down the breast.

According to the Health Insurance Portability and Account- ability Act of 1996, if a health institution wants to release a patient's health information (PHI) for purposes other than treatment, payment, and routine health care operations, the patient must be asked to sign an authorization. The nurse is aware that there are exceptions to this requirement. In which of the following cases is an authorization form not needed? Select all that apply. a. News media are preparing a report on the condition of a public figure. b. Data are needed for the tracking and notification of disease outbreaks. c. Protected health information is needed by a coroner. d. Child abuse and neglect are suspected. e. Protected health information is needed to facilitate organ donation. f. The sister of a patient with Alzheimer's wants to help provide care.

b, c, d, e. Authorization is not required for tracking disease outbreaks, providing PHI to a coroner, reporting incidents of child abuse, or facilitating organ donations. Under no cir- cumstance can a nurse provide information to a news reporter without the patient's express authorization. An authorization form is still needed to provide PHI for a patient who has Alzheimer's disease.

Despite a national focus on health promotion, nurses working with patients in inner-city clinics continue to see disparities in health care for vulnerable populations. Which patients would be considered vulnerable populations? Select all that apply. a. A White male diagnosed with HIV b. An African American teenager who is 6 months pregnant c. A Hispanic male who has type II diabetes d. A low-income family living in rural America e. A middle-class teacher living in a large city f. A White baby who was born with cerebral palsy

b, c, d, f. National trends in the prevention of health dispari- ties are focused on vulnerable populations, such as racial and ethnic minorities, those living in poverty, women, children, older adults, rural and inner-city residents, and people with disabilities and special health care needs.

A nurse is developing a plan of care for an 86-year-old woman who has been admitted for right hip arthroplasty (hip replacement). Which assessment finding(s) indicate a high risk for pressure ulcer development for this patient? Select all that apply. a. The patient takes time to think about her responses to questions. b. The patient's age of 86 years. c. Patient reports inability to control urine. d. A scheduled hip arthroplasty e. Lab findings include BUN 12 (elderly normal 8-23 mg/dL) and creatinine 0.9 (adult female normal 0.61-1 mg/dL). f. Patient reports increased pain in right hip when reposition- ing in bed or chair.

b, c, d, f. Pressure, friction, and shear, as well as other factors, usually combine to contribute to pressure ulcer development. The skin of older adults is more susceptible to injury; incontinence contributes to prolonged moisture on the skin, as well as negative effects related to urine in contact with skin; hip surgery involves decreased mobility during the postoperative period, as well as pain with move- ment, contributing to immobility; and increased pain in the hip may contribute to increased immobility. All these factors are related to an increased risk for pressure ulcer develop- ment. Apathy, confusion, and/or altered mental status are risk factors for pressure ulcer development. Dehydration (indicated by an elevated BUN and creatinine) is a risk for pressure ulcer development.

A nurse caring for patients in a skilled nursing facility performs risk assessments on the patients for foot and nail problems. Which patients would be at a higher risk? Select all that apply. a. A patient who is taking antibiotics for chronic bronchitis b. A patient diagnosed with type II diabetes c. A patient who is obese d. A patient who has a nervous habit of biting his nails e. A patient diagnosed with prostate cancer f. A patient whose job involves frequent hand washing

b, c, d, f. Variables known to cause nail and foot problems include deficient self-care abilities, vascular disease, arthritis, diabetes mellitus, history of biting nails or trimming them improperly, frequent or prolonged exposure to chemicals or water, trauma, ill-fitting shoes, and obesity.

Which actions would the nurse perform when using the technique of palpation during the physical assessment of a patient? Select all that apply. a. The nurse compares the patient's bilateral body parts for symmetry. b. The nurse takes a patient's pulse. c. The nurse touches a patient's skin to test for turgor. d. The nurse checks a patient's lymph nodes for swelling. e. The nurse taps a patient's body to check the organs. f. The nurse uses a stethoscope to listen to a patient's heart sounds

b, c, d. During palpation, the nurse uses the sense of touch to take a pulse, test for skin turgor, and check lymph nodes. With inspection, a comparison of bilateral body parts is necessary for recognizing abnormal findings. During per- cussion, the fingertips are used to tap the body over body tissues to produce vibrations and sound waves. The char- acteristics of the sounds produced are used to assess the location, shape, size, and density of tissues. Auscultation is the act of listening with a stethoscope to sounds produced within the body.

A nurse is planning care for a male adolescent patient who is admitted to the hospital for treatment of a drug overdose. Which nursing actions are related to the outcome identifica- tion and planning step of the nursing process? Select all that apply. a. The nurse formulates nursing diagnoses. b. The nurse identifies expected patient outcomes. c. The nurse selects evidence-based nursing interventions. d. The nurse explains the nursing care plan to the patient. e. The nurse assesses the patient's mental status. f. The nurse evaluates the patient's outcome achievement.

b, c, d. During the outcome identification and planning step of the nursing process, the nurse works in partnership with the patient and family to establish priorities, identify and write expected patient outcomes, select evidence-based nursing interventions, and communicate the plan of nursing care. Although all these steps may overlap, formulating and validating nursing diagnoses occurs most frequently during the diagnosing step of the nursing process. Assessing mental status is part of the assessment step, and evaluating patient outcomes occurs during the evaluation step of the nursing process.

A nurse is caring for a 16-year-old male patient who has been hospitalized for an acute asthma exacerbation. Which testing methods might the nurse use to measure the patient's oxygen saturation? Select all that apply. a. Thoracentesis b. Spirometry c. Pulse oximetry d. Peak expiratory flow rate e. Diffusion capacity f. Maximal respiratory pressure

b, c, d. Spirometers are used to monitor the health status of patients with respiratory disorders, such as asthma. Pulse oximetry is used to obtain baseline information about the patient's oxygen saturation level and is also performed for patients with asthma, along with PEFR to monitor airflow. These three tests may be administered by the nurse. Diffu- sion capacity estimates the patient's ability to absorb alveo- lar gases and determines if a gas exchange problem exists. Maximal respiratory pressures help evaluate neuromuscular causes of respiratory dysfunction. Both tests are usually performed by a respiratory therapist. The physician or other advanced practice professional can perform a thoracentesis at the bedside with the nurse assisting, or in the radiology department.

The nurse instructor is teaching student nurses about the factors that may affect a patient's blood pressure. Which state- ments accurately describe these factors? Select all that apply. a. Blood pressure decreases with age. b. Blood pressure is usually lowest on arising in the morning. c. Women usually have lower blood pressure than men until menopause. d. Blood pressure decreases after eating food. e. Blood pressure tends to be lower in the prone or supine position. f. Increased blood pressure is more prevalent in African Americans.

b, c, e, f. Blood pressure increases with age due to a decreased elasticity of the arteries, increasing peripheral resistance. Blood pressure is usually lowest on arising in the morning. Women usually have lower blood pressure than men until menopause occurs. Blood pressure increases after eating food. Blood pressure tends to be lower in the prone or supine position. Increased blood pressure is more prevalent and severe in Afri- can American men and women.

An RN working on a busy hospital unit delegates patient care to unlicensed assistive personnel (UAPs). Which patient care could the nurse most likely delegate to a UAP safely? Select all that apply. a. Performing the initial patient assessments b. Making patient beds c. Giving patients bed baths d. Administering patient medications e. Ambulating patients f. Assisting patients with meals

b, c, e, f. Performing the initial patient assessment and administering medications are the responsibility of the registered nurse. In most cases, patient hygiene, bed-making, ambulating patients, and helping to feed patients can be delegated to a UAP.

A nurse is using general systems theory to describe the role of nursing to provide health promotion and patient teaching. Which statements reflect key points of this theory? Select all that apply. a. A system is a set of individual elements that rarely interact with each other. b. The whole system is always greater than its parts. c. Boundaries separate systems from each other and their environments. d. A change in one subsystem will not affect other subsystems. e. To survive, open systems maintain balance through feedback. f. A closed system allows input from and to the environment.

b, c, e. According to general systems theory, a system is a set of interacting elements contributing to the overall goal of the system. The whole system is always greater than its parts. Boundaries separate systems from each other and their environments. Systems are hierarchical in nature and are composed of interrelated subsystems that work together in such a way that a change in one element could affect other subsystems, as well as the whole. To survive, open systems maintain balance through feedback. An open system allows energy, matter, and information to move freely between sys- tems and boundaries, whereas a closed system does not allow input from or output to the environment.

A nurse uses critical thinking skills to focus on the care plan of an elderly patient who has dementia and needs placement in a long-term care facility. Which statements describe char- acteristics of this type of critical thinking applied to clinical reasoning? Select all that apply. a. It functions independently of nursing standards, ethics, and state practice acts. b. It is based on the principles of the nursing process, prob- lem solving, and the scientific method. c. It is driven by patient, family, and community needs as well as nurses' needs to give competent, efficient care. d. It is not designed to compensate for problems created by human nature, such as medication errors. e. It is constantly re-evaluating, self-correcting, and striving for improvement. f. It focuses on the big picture rather than identifying the key problems, issues, and risks involved with patient care.

b, c, e. Critical thinking applied to clinical reasoning and judgment in nursing practice is guided by standards, policies and procedures, and ethics codes. It is based on principles of nursing process, problem solving, and the scientific method. It carefully identifies the key problems, issues, and risks involved, and is driven by patient, family, and community needs, as well as nurses' needs to give competent, efficient care. It also calls for strategies that make the most of human potential and compensate for problems created by human nature. It is constantly re-evaluating, self-correcting, and striving to improve (Alfaro-LeFevre, 2014).

A nurse is assisting a postoperative patient with condition- ing exercises to prepare for ambulation. The nurse correctly instructs the patient to do which actions? Select all that apply. a. Do full-body pushups in bed six to eight times daily. b. Breathe in and out smoothly during quadriceps drills. c. Place the bed in the lowest position or use a footstool for dangling. d. Dangle on the side of the bed for 30 to 60 minutes. e. Allow the nurse to bathe the patient completely to prevent fatigue. f. Perform quadriceps 2 to 3 times per hour, 4 to 6 times a day.

b, c, f. Breathing in and out smoothly during quadriceps drills maximizes lung inflation. The patient should perform quadri- ceps 2 to 3 times per hour, 4 to 6 times a day, or as ordered. The patient should never hold their breath during exercise drills because this places a strain on the heart. Pushups are usually done three or four times a day and involve only the upper body. Dangling for 30 to 60 minutes is unsafe. The nurse should place the bed in the lowest position or use a footstool for dangling. The nurse should also encourage the patient to be as independent as possible to prepare for return to normal ambulation and ADLs.

A nurse instructor is teaching a class of student nurses about the nature of pain. Which statements accurately describe this phenomenon? Select all that apply. a. Pain is whatever the physician treating the pain says it is. b. Pain exists whenever the person experiencing it says it exists. c. Pain is an emotional and sensory reaction to tissue damage. d. Pain is a simple, universal, and easy-to-describe phenomenon. e. Pain that occurs without a known cause is psychological in nature. f. Pain is classified by duration, location, source, transmis- sion, and etiology.

b, c, f. Margo McCaffery (1979, p. 11) offers the classic definition of pain that is probably of greatest benefit to nurses and their patients: "Pain is whatever the experiencing person says it is, existing whenever he (or she) says it does." The International Association for the Study of Pain (IASP) further defines pain as an unpleasant sensory and emotional experi- ence associated with actual or potential tissue damage (IASP, 1994). Pain is an elusive and complex phenomenon, and despite its universality, its exact nature remains a mystery. Pain is present whenever a person says it is, even when no specific cause of the pain can be found. Pain may be classi- fied according to its duration, its location or source, its mode of transmission, or its etiology.

A nurse is caring for an older adult patient who presents with labored respirations, productive cough, and fever. What would be appropriate nursing diagnoses for this patient? Select all that apply. a. Bronchial pneumonia b. Impaired gas exchange c. Ineffective airway clearance d. Potential complication: sepsis e. Infection related to pneumonia f. Risk for septic shock

b, c, f. Nursing diagnoses are actual or potential health problems that can be prevented or resolved by independent nursing interventions, such as impaired gas exchange, inef- fective airway clearance, or risk for septic shock. Bronchial pneumonia and infection are medical diagnoses, and "poten- tial complication: sepsis" is a collaborative problem.

Health promotion activities may occur on a primary, second- ary, or tertiary level. Which activities are considered tertiary health promotion? Select all that apply. a. A nurse runs an immunization clinic in the inner city. b. A nurse teaches a patient with an amputation how to care for the residual limb. c. A nurse provides range-of-motion exercises for a para- lyzed patient. d. A nurse teaches parents of toddlers how to childproof their homes. e. A school nurse provides screening for scoliosis for the students. f. A nurse teaches new parents how to choose and use an infant car seat.

b, c. Tertiary health promotion and disease prevention begins after an illness is diagnosed and treated to reduce disability and to help rehabilitate patients to a maximum level of func- tioning. These activities include providing ROM exercises and patient teaching for residual limb care. Providing immu- nizations and teaching parents how to childproof their homes and use an appropriate car seat are primary health promotion activities. Providing screenings is a secondary health promo- tion activity.

A nurse who is caring for older adults in a senior daycare center documents findings as related to which normal aging process? Select all that apply. a. A patient's increased skin elasticity causes wrinkles on the face and arms. b. Exposure to sun over the years causes a patient's skin to be pigmented. c. A patient's toenails have become thinner with a bluish tint to the nail beds. d. A patient experiences a hip fracture due to porous and brit- tle bones. e. Fragile blood vessels in the dermis allow for more easy bruising of a patient's forearm. f. Increased bladder capacity causes decreased voiding in an older patient.

b, d, e. Exposure to sun over the years can cause a patient's skin to be pigmented. Bone demineralization occurs with aging, causing bones to become porous and brittle, making fractures more common. The blood vessels in the dermis become more fragile, causing an increase in bruising and purpura. Wrinkling and sagging of skin occur with decreased skin elasticity. A patient's toenails may become thicker, with a yellowish tint to the nail beds. Bladder capacity decreases by 50%, making voiding more frequent; two or three times a night is usual.

A nurse who is assessing an older female patient in a long- term care facility notes that the patient is at risk for sensory deprivation related to severe rheumatoid arthritis limiting her activity. Which interventions would the nurse recommend based on this finding? Select all that apply. a. Use a lower tone when communicating with the patient. b. Provide interaction with children and pets. c. Decrease environmental noise. d. Ensure that the patient shares meals with other patients. e. Discourage the use of sedatives. f. Provide adequate lighting and clear pathways of clutter.

b, d, e. For a patient who has sensory deprivation, the nurse should provide interaction with children and pets, ensure that the patient shares meals with other patients, and discourage the use of sedatives. Using a lower tone of voice is appro- priate for a patient who has a hearing deficit, decreasing environmental noise is an intervention for sensory overload, and providing adequate lighting and removing clutter is an intervention for a vision deficit.

A nurse on a busy surgical unit relies on informal planning to provide appropriate nursing responses to patients in a timely manner. What are examples of this type of planning? Select all that apply. a. A nurse sits down with a patient and prioritizes existing diagnoses. b. A nurse assesses a woman for postpartum depression during routine care. c. A nurse plans interventions for a patient who is diagnosed with epilepsy. d. A busy nurse takes time to speak to a patient who received bad news. e. A nurse reassesses a patient whose PRN pain medication is not working. f. A nurse coordinates the home care of a patient being discharged.

b, d, e. Informal planning is a link between identifying a patient's strength or problem and providing an appropriate nursing response. This occurs, for example, when a busy nurse first recognizes postpartum depression in a patient, takes time to assess a patient who received bad news about tests, or reassesses a patient for pain. Formal planning involves prioritizing diagnoses, formally planning interven- tions, and coordinating the home care of a patient being discharged.

A nurse working in a long-term care facility is providing teaching to patients with altered oxygenation due to condi- tions such as asthma and COPD. Which measures would the nurse recommend? Select all that apply. a. Refrain from exercise. b. Reduce anxiety. c. Eat meals 1 to 2 hours prior to breathing treatments. d. Eat a high-protein/high-calorie diet. e. Maintain a high-Fowler's position when possible. f. Drink 2 to 3 pints of clear fluids daily.

b, d, e. When caring for patients with COPD, it is important to create an environment that is likely to reduce anxiety and ensure that they eat a high-protein/high-calorie diet. People with dyspnea and orthopnea are most comfortable in a high Fowler's position because accessory muscles can easily be used to promote respiration. Patients with COPD should pace physical activities and schedule frequent rest periods to conserve energy. Meals should be eaten 1 to 2 hours after breathing treatments and exercises, and drinking 2 to 3 quarts (1.9-2.9 L) of clear fluids daily is recommended.

A nurse is planning teaching strategies for patients addicted to alcohol, in the affective domain of learning. What are examples of strategies promoting behaviors in this domain? Select all that apply. a. The nurse prepares a lecture on the harmful long-term effects of alcohol on the body. b. The nurse explores the reasons alcoholics drink and pro- motes other methods of coping with problems. c. The nurse asks patients for a return demonstration for using relaxation exercises to relieve stress. d. The nurse helps patients to reaffirm their feelings of self- worth and relate this to their addiction problem. e. The nurse uses a pamphlet to discuss the tenants of the Alcoholics Anonymous program to patients. f. The nurse reinforces the mental benefits of gaining self- control over an addiction.

b, d, f. Affective learning includes changes in attitudes, values, and feelings (e.g., the patient expresses renewed self-confidence to be able to give up drinking). Cognitive learning involves the storing and recalling of new knowledge in the brain, such as the learning that occurs during a lecture or by using a pamphlet for teaching. Learning a physical skill involving the integration of mental and muscular activity is called psychomotor learning, which may involve a return demonstration of a skill.

The nurse caring for patients in a long-term care facility knows that there are factors that place certain patients at a higher risk for falls. Which patients would the nurse consider to be in this category? Select all that apply. a. A patient who is older than 60 years b. A patient who has already fallen twice c. A patient who is taking antibiotics d. A patient who experiences postural hypotension e. A patient who is experiencing nausea from chemotherapy f. A 70-year old patient who is transferred to long-term care

b, d, f. Risk factors for falls include age over 65 years, docu- mented history of falls, postural hypotension, and unfamiliar environment. A medication regimen that includes diuretics, tranquilizers, sedatives, hypnotics, or analgesics is also a risk factor, not chemotherapy or antibiotics.

A registered nurse is writing a diagnosis for a 28-year-old male patient who is in traction due to multiple fractures from a motor vehicle accident. Which nursing actions are related to this step in the nursing process? Select all that apply. a. The nurse uses the nursing interview to collect patient data. b. The nurse analyzes data collected in the nursing assessment. c. The nurse develops a care plan for the patient. d. The nurse points out the patient's strengths. e. The nurse assesses the patient's mental status. f. The nurse identifies community resources to help his family cope.

b, d, f. The purposes of diagnosing are to identify how an indi- vidual, group, or community responds to actual or potential health and life processes; identify factors that contribute to or cause health problems (etiologies); and identify resources or strengths the individual, group, or community can draw on to prevent or resolve problems. In the diagnosing step of the nursing process, the nurse interprets and analyzes data gath- ered from the nursing assessment, identifies patient strengths, and identifies resources the patient can use to resolve problems. The nurse assesses and collects patient data in the assessment step and develops a care plan in the planning step of the nursing process.

Which nursing actions reflect the use of philosophy as a knowledge base when delivering evidence-based care to patients? Select all that apply. a. A nurse interviews and examines a new patient diagnosed with prehypertension to formulate a care plan. b. A nurse draws from personal experiences of being a patient to establish a therapeutic relationship with a patient. c. A nurse searches the Internet to find the latest treatments for type 2 diabetes. d. A nurse uses spiritual training to draw strength when coun- seling a patient who is in hospice for an inoperable brain tumor. e. A nurse follows the protocol for assessing postoperative patients in the ICU. f. A nursing student studies anatomy and physiology of the body systems to understand the disease states of assigned patients.

b, d. Philosophy is the study of wisdom, fundamental knowl- edge, and the processes used to develop and construct one's perceptions of life. A philosophy is developed from personal experiences (such as the experience of being the patient), through formal and informal educational experiences, through religion and culture (such as using spiritual training as a source of strength), and from the environment. Interviewing and examining patients to formulate a care plan and using protocol for assessing patients in the ICU involve knowledge of processes. Researching the Internet and studying anatomy and physiology use scientific knowledge to deliver evidence- based care.

The nurse is prioritizing nursing care for a patient in a long-term care facility. Which examples of nursing interventions help meet physiologic needs? Select all that apply. a. Preventing falls in the facility b. Changing a patient's oxygen tank c. Providing materials for a patient who likes to draw d. Helping a patient eat his dinner e. Facilitating a visit from a spouse f. Referring a patient to a cancer support group.

b, d. Physiologic needs—oxygen, water, food, elimination, temperature, sexuality, physical activity, and rest—must be met at least minimally to maintain life. Providing food and oxygen are examples of interventions to meet these needs. Preventing falls helps meet safety and security needs, provid- ing art supplies may help meet self-actualization needs, facili- tating visits from loved ones helps meet self-esteem needs, and referring a patient to a support group helps meet love and belonging needs.

A nurse assesses the stool of patients who are experiencing gastrointestinal problems. In which patients would diarrhea be a possible finding? Select all that apply. a. A patient who is taking narcotics for pain b. A patient who is taking laxatives c. A patient who is taking diuretics d. A patient who is dehydrated e. A patient who is taking amoxicillin for an infection f. A patient taking over-the-counter antacids

b, e, f. Diarrhea is a potential adverse effect of treatment with amoxicillin clavulanate (Augmentin), laxatives, or over-the- counter antacids. Narcotics, diuretics, and dehydration may lead to constipation.

A nurse is performing hand hygiene after providing patient care. The nurse's hands are not visibly soiled. Which steps in this procedure are performed correctly? Select all that apply. a. The nurse removes all jewelry including a platinum wed- ding band. b. The nurse washes hands to one inch above the wrists. c. The nurse uses approximately two teaspoons of liquid soap. d. The nurse keeps hands higher than elbows when placing under faucet e. The nurse uses friction motion when washing for at least 15 seconds. f. The nurse rinses thoroughly with water flowing toward fingertips.

b, e, f. Proper hand hygiene includes removing jewelry with the exception of a plain wedding band, wetting the hands and wrist area with the hands lower than the elbows, using about one teaspoon of liquid soap, using friction motion for at least 15 seconds, washing to one inch above the wrists with a fric- tion motion for at least 15 seconds, and rinsing thoroughly with water flowing toward fingertips.

A nurse is assessing a patient's eyes for accommodation. What actions would the nurse perform during this test? Select all that apply. a. Bring a penlight from the side of the patient's face and briefly shine the light on the pupil. b. Hold a forefinger, a pencil, or other straight object about 10 to 15 cm (4′′ to 6′′) from the bridge of the patient's nose. c. Hold a finger about 6′′ to 8′′ from the bridge of the patient's nose. d. Darken the room. e. Ask the patient to look straight ahead. f. Ask the patient to first look at a close object, then at a distant object, then back to the close object.

b, f. To test accommodation the nurse would hold the fore- finger, a pencil, or other straight object about 10 to 15 cm (4′′ to 6′′) from the bridge of the patient's nose. Then the nurse would ask the patient to first look at the object, then at a distant object, then back to the object being held. The pupil normally constricts when looking at a near object and dilates when looking at a distant object. To test for conver- gence, the nurse would darken the room and ask the patient to look straight ahead. The nurse would then bring the penlight from the side of the patient's face and briefly shine the light on the pupil, observing the reaction. When testing convergence the nurse would hold a finger about 6′′ to 8′′ from the bridge of the patient's nose and move it toward the patient's nose.

A nurse writes nursing diagnoses for patients and their families visiting a community health clinic. Which nursing diagnoses are correctly written as three-part nursing diagnoses? (1) Disabled Family Coping related to lack of knowledge about home care of child on ventilator (2) Imbalanced Nutrition: Less Than Body Requirements related to inadequate caloric intake while striving to excel in gymnastics as evidenced by 20-pound weight loss since beginning the gymnastic program, and greatly less than ideal body weight when compared to standard height weight charts (3) Need to learn how to care for child on ventilator at home related to unexpected discharge of daughter after 3-month hospital stay as evidenced by repeated com- ments "I cannot do this," "I know I'll harm her because I'm not a nurse," and "I can't do medical things" (4) Spiritual Distress related to inability to accept diagnosis of terminal illness as evidenced by multiple comments such as "How could God do this to me?," "I don't deserve this," "I don't understand. I've tried to live my life well," and "How could God make me suffer this way?" (5) Caregiver Role Strain related to failure of home health aides to appropriately diagnose needs of family caregivers and initiate a plan to facilitate coping as evidenced by caregiver's loss of weight and clinical depression a. (1) and (3) b. (2) and (4) c. (1), (2), and (3) d. All of the above

b. (1) is a two-part diagnosis, (3) is written in terms of needs and not an unhealthy response, and (5) is a legally inadvisable statement.

A nurse makes a clinical judgment that an African American male patient in a stressful job is more vulnerable to devel- oping hypertension than White male patients in the same or similar situation. The nurse has formulated what type of nursing diagnosis? a. Actual b. Risk c. Possible d. Wellness

b. A clinical judgment that an individual, family, or commu- nity is more vulnerable to develop the problem than others in the same or similar situation is a Risk nursing diagnosis.

A nurse who is caring for a morbidly obese male teenager forms a contractual agreement with him to achieve his weight goals. Which statement best describes the nature of this agreement? a. "This agreement forms a legal bond between the two of us to achieve your weight goals." b. "This agreement will motivate the two of us to do what is necessary to meet your weight goals." c. "This agreement will help us determine what learning outcomes are necessary to achieve your weight goals." d. "This agreement will limit the scope of the teaching ses- sion and make stated weight goals more attainable."

b. A contractual agreement is a pact two people make setting out mutually agreed-on goals. Contracts are usually informal and not legally binding. When teaching a patient, such an agreement can help motivate both the patient and the teacher to do what is necessary to meet the patient's learning out- comes. The agreement notes the responsibilities of both the teacher and the learner, emphasizing the importance of the mutual commitment.

The nurse records an APGAR score of 4 for a newborn. What would be the priority intervention for this newborn? a. No interventions are necessary; this is a normal score. b. Provide respiratory assistance. c. Perform CPR. d. Wait 5 minutes and repeat the scoring process.

b. A newborn who scores a 4 on the APGAR chart requires special assistance such as respiratory assistance. Normal APGAR scores are 7 to 10. Neonates who score between 4 and 6 require special assistance, and those who score below 4 are in need of life-saving support.

When planning care for a patient with chronic lung disease who is receiving oxygen through a nasal cannula, what does the nurse expect? a. The oxygen must be humidified. b. The rate will be no more than 2 to 3 L/min or less. c. Arterial blood gases will be drawn every 4 hours to assess flow rate. d. The rate will be 6 L/min or more.

b. A rate higher than 3 L/min may destroy the hypoxic drive that stimulates respirations in the medulla in a patient with chronic lung disease. Oxygen delivered at low rates does not necessarily have to be humidified, and arterial blood gases are not required at regular intervals to determine the flow rate.

A nurse working with adolescents in a group home discusses the developmental tasks appropriate for adolescents with the staff. What is an example of a primary developmental task of the adolescent? a. Working hard to succeed in school b. Spending time developing relationships with peers c. Developing athletic activities and skills d. Accepting the decisions of parents

b. Adolescence is a time to establish more mature relation- ships with both boys and girls of the same age.

A 16-year-old patient has a nursing diagnosis of Body Image Disturbance related to severe acne. In planning nursing care, what is an appropriate goal for this nursing diagnosis? a. The patient will make above-B grades in all tests at school." b. The patient will demonstrate by diet control and skin care increased interest in control of acne. c. The patient reports that she feels more self-confidence in her music and art, which she enjoys. d. The patient expresses that she is very smart in school.

b. All of these patient goals may be appropriate for the patient but the only goal that directly addresses her body image disturbance is

A nurse assisting with a patient bed bath observes that an older female patient has dry skin. The patient states that her skin is always "itchy." Which nursing action would be the nurse's best response? a. Bathe the patient more frequently. b. Use an emollient on the dry skin. c. Massage the skin with alcohol. d. Discourage fluid intake.

b. An emollient soothes dry skin, whereas frequent bathing increases dryness, as does alcohol. Discouraging fluid intake leads to dehydration and, subsequently, dry skin.

A nurse is diagnosing an 11-year-old 6th grade student following a physical assessment. The nurse notes that the student's grades have dropped, she has difficulty complet- ing her work on time, and she frequently rubs her eyes and squints. Her visual acuity on a Snellen's eye chart is 160/20. Which nursing diagnosis would be most appropriate? a. Deficient Knowledge related to visual impairment b. Ineffective Role Performance (Student) related to visual impairment c. Disturbed Body Image related to visual impairment d. Delayed Growth and Development related to visual impairment

b. An important role for an 11-year-old is that of student. Her impaired vision is clearly disturbing her role performance as a student, as evidenced by her lower grades. Although the other options may also represent accurate diagnoses for this patient, they do not flow from the data presented.

A nurse is collecting more patient data to confirm a diagnosis of emphysema for a 68-year-old male patient. What type of diagnosis does this intervention seek to confirm? a. Actual b. Possible c. Risk d. Collaborative

b. An intervention for a possible diagnosis is to collect more patient data to confirm or rule out the problem. An inter- vention for an actual diagnosis is to reduce or eliminate contributing factors to the diagnosis. Interventions for a risk diagnosis focus on reducing or eliminating risk factors, and interventions for collaborative problems focus on monitoring for changes in status and managing these changes with nurse- and physician-prescribed interventions.

A home health care nurse takes the vital signs of a patient who is receiving supplemental oxygen at home for chronic obstructive pulmonary disease (COPD). This is the nurse's fourth visit to the patient's home. The nurse records the data collected on the patient's chart. What type of assessment has this nurse performed? a. Comprehensive b. Ongoing partial assessment c. Focused assessment d. Emergency assessment

b. An ongoing partial assessment, or follow-up assessment, is conducted at regular intervals (e.g., at the beginning of each home health visit or each hospital shift) during care of the patient. This type of assessment concentrates on identified health problems to monitor positive or negative changes and evaluate the effectiveness of interventions. A comprehen- sive assessment with a health history and complete physical examination is usually conducted when a patient first enters a health care setting, with information providing a baseline for comparing later assessments. A focused assessment is conducted to assess a specific problem. An emergency assess- ment is a type of rapid focused assessment conducted when addressing a life-threatening or unstable situation.

A school nurse is performing an assessment of a student who states: "I'm too tired to keep my head up in class." The student has a low-grade fever. The nurse would interpret these findings as indicating which stage of infection? a. Incubation period b. Prodromal stage c. Full stage of illness d. Convalescent period

b. During the prodromal stage, the person has vague signs and symptoms, such as fatigue and a low-grade fever. There are no obvious symptoms of infection during the incubation period, and they are more specific during the full stage of ill- ness, before disappearing by the convalescent period.

A nurse is caring for a patient who is on bed rest following a spinal injury. In which position would the nurse place the patient's feet to prevent footdrop? a. Supination b. Dorsiflexion c. Hyperextension d. Abduction

b. For a patient who has footdrop, the nurse should support the feet in dorsiflexion, and use a footboard or high-top sneakers to further support the foot. Supination involves lying patients on their back or facing a body part upward, and hyperextension is a state of exaggerated extension. Abduction involves lateral movement of a body part away from the midline of the body. These positions would not be used to prevent footdrop.

A nurse is irrigating the colostomy of a patient and is unable to get the irrigation solution to flow. What would be the nurse's next action in this situation? a. Assist the patient to a prone position on a waterproof pad and try again. b. Check the clamp on the tubing to make sure that the tubing is open. c. Quickly pull the cone from the stoma and check for bleeding. d. Remove the equipment and call the primary care provider.

b. If irrigation solution is not flowing, the nurse should first check the clamp on the tubing to make sure the tubing is open. Next, the nurse should gently manipulate the cone in the stoma and check for a blockage of stool. If there is a blockage, the nurse should remove the cone from the stoma, clean the area, and gently reinsert. Alternately, the nurse could assist the patient to a side-lying or sitting position in bed, place a waterproof pad under the irrigation sleeve, and place the drainage end of the sleeve in a bedpan.

A nurse is caring for an 82-year-old woman in a long-term care facility who has had two urinary tract infections in the past year related to immobility. Which finding would the nurse expect in this patient? a. Improved renal blood supply to the kidneys b. Urinary stasis c. Decreased urinary calcium d. Acidic urine formation

b. In a nonerect patient, the kidneys and ureters are level. In this position, urine remains in the renal pelvis for a longer period of time before gravity causes it to move into the ureters and bladder, resulting in urinary stasis. Urinary stasis favors the growth of bacteria that may cause urinary tract infections.Regular exercise, not immobility, improves blood flow to the kidneys. Immobility predisposes the patient to increased levels of urinary calcium and alkaline urine, contributing to renal calculi and urinary tract infection, respectively.

The nurse caring for families in a free health care clinic identifies psychosocial risk factors for altered family health. Which example describes one of these risk factors? a. The family does not have dental care insurance or resources to pay for it. b. Both parents work and leave a 12-year old child to care for his younger brother. c. Both parents and their children are considerably overweight. d. The youngest member of the family has cerebral palsy and needs assistance from community services.

b. Inadequate childcare resources is a psychosocial risk factor. Not having access to dental care and obese family members are lifestyle risk factors. Having a family member with birth defects is a biologic risk factor.

A nurse is providing care based on Maslow's hierarchy of basic human needs. For which nursing activities is this approach useful? a. Making accurate nursing diagnoses b. Establishing priorities of care c. Communicating concerns more concisely d. Integrating science into nursing care

b. Maslow's hierarchy of basic human needs is useful for establishing priorities of care.

A student nurse attempts to perform a nursing history for the first time. The student nurse asks the instructor how anyone ever learns all the questions the nurse must ask to get good baseline data. What would be the instructor's best reply? a. "There's a lot to learn at first, but once it becomes part of you, you just keep asking the same questions over and over in each situation until you can do it in your sleep!" b. "You make the basic questions a part of you and then learn to modify them for each unique situation, asking yourself how much you need to know to plan good care." c. "No one ever really learns how to do this well because each history is different! I often feel like I'm starting afresh with each new patient." d. "Don't worry about learning all of the questions to ask. Every agency has its own assessment form you must use."

b. Once you learn what constitutes the minimum data set, you can adapt this to any patient situation. It is not true that each assessment is the same even when you are using the same minimum data set, nor is it true that each assessment is uniquely different. Nurses committed to thoughtful, person- centered practice tailor their questions to the uniqueness of each patient and situation. Answer d is incorrect because rely- ing solely on standard agency assessment tools does not allow for individualized patient care or critical thinking.

Which action would be most important for a nurse to include in the plan of care for a patient who is 85 years old and has presbycusis? a. Obtaining large-print written material b. Speaking distinctly, using lower frequencies c. Decreasing tactile stimulation d. Initiating a safety program to prevent falls

b. Presbycusis is a normal loss of hearing as a result of the aging process. Speaking distinctly in lower frequencies is indicated. The other choices refer to interventions for other sensory problems.

The nurse uses the RYB wound classification system to assess the wound of a client who cut his arm on a factory machine. The nurse documents the wound as "red." What would be the priority nursing intervention for this type of wound? a. Irrigate the wound. b. Provide gentle cleansing of the wound. c. Débride the wound. d. Change the dressing frequently.

b. Red wounds are in the proliferative stage of healing and reflect the color of normal granulation tissue. Wounds in this stage need protection with nursing interventions that include gentle cleansing, use of moist dressings, and changing of the dressing only when necessary, and/or based on product manufacturer's recommendations. To cleanse yellow wounds, nursing interventions include the use of wound cleansers and irrigating the wound. The eschar found in black wounds requires débridement (removal) before the wound can heal.

A nurse is caring for a 42-year-old male patient who is admitted to the hospital with injuries sustained in a motor vehicle accident. While he is in the hospital, his wife tells him that the bottom level of their house flooded, damaging their belongings. When the nurse enters his room, she notes that the patient is visibly upset. The nurse is aware that the patient will most likely be in need of which type of counseling? a. Long-term developmental b. Short-term situational c. Short-term motivational d. Long-term motivational

b. Short-term counseling might be used during a situational crisis, which occurs when a patient faces an event or situation that causes a disruption in life, such as a flood. Long-term counseling extends over a prolonged period; a patient experi- encing a developmental crisis, for example, might need long- term counseling. Motivational interviewing is an evidence- based counseling approach that involves discussing feelings and incentives with the patient. A caring nurse can motivate patients to become interested in promoting their own health.

A nurse is teaching first aid to counselors of a summer camp for children with asthma. This is an example of what aim of health teaching? a. Promoting health b. Preventing illness c. Restoring health d. Facilitating coping

b. Teaching first aid is a function of the goal to prevent ill- ness. Promoting health involves helping patients to value health and develop specific health practices that promote wellness. Restoring health occurs once a patient is ill, and teaching focuses on developing self-care practices that pro- mote recovery. When facilitating coping, nurses help patients come to terms with whatever lifestyle modification is needed for their recovery or to enable them to cope with permanent health alterations.

The National Advisory Council on Nurse Education and Practice identifies critical challenges to nursing practice in the 21st century. What is a current health care trend contributing to these challenges? a. Decreased numbers of hospitalized patients b. Older and more acutely ill patients c. Decreasing health care costs due to managed care d. Slowed advances in medical knowledge and technology

b. The National Advisory Council on Nurse Education and Practice identifies the following critical challenges to nursing practice in the 21st century: A growing popula- tion of hospitalized patients who are older and more acutely ill, increasing health care costs, and the need to stay current with rapid advances in medical knowledge and technolog

Nurses use the Nursing Interventions Classification Taxonomy structure as a resource when planning nursing care for patients. What information would be found in this structure? a. Case studies illustrating a complete set of activities that a nurse performs to carry out nursing interventions b. Nursing interventions, each with a label, a definition, and a set of activities that a nurse performs to carry it out, with a short list of background readings c. A complete list of nursing diagnoses, outcomes, and related nursing activities for each nursing intervention d. A complete list of reimbursable charges for each nursing intervention

b. The Nursing Interventions Classification Taxonomy lists nursing interventions, each with a label, a definition, a set of activities that a nurse performs to carry it out, and a short list of background readings. It does not contain case studies, diagnoses, or charges.

A nursing student is preparing to administer morning care to a patient. What is the most important question that the nurs- ing student should ask the patient about personal hygiene? a. "Would you prefer a bath or a shower?" b. "May I help you with a bed bath now or later this morning?" c. "I will be giving you your bath. Do you use soap or shower gel?" d. "I prefer a shower in the evening. When would you like your bath?"

b. The nurse should ask permission to assist the patient with a bath. This allows for consent to assist the patient with care that invades the patient's private zones.

The Joint Commission issues guidelines regarding the use of restraints. In which case is a restraint properly used? a. The nurse positions a patient in a supine position prior to applying wrist restraints. b. The nurse ensures that two fingers can be inserted between the restraint and patient's ankle. c. The nurse applies a cloth restraint to the left hand of a patient with an IV catheter in the right wrist. d. The nurse ties an elbow restraint to the raised side rail of a patient's bed.

b. The nurse should be able to place two fingers between the restraint and a patient's wrist or ankle. The patient should not be put in a supine position with restraints due to risk of aspiration. Due to the IV in the right wrist, alternative forms of restraints should be tried, such as a cloth mitt or an elbow restraint. Securing the restraint to a side rail may injure the patient when the side rail is lowered.

A friend of a nurse calls and asks if she is still working at Memorial Hospital. The nurse replies, "Yes." The friend tells the nurse that his girlfriend's father was just admitted as a patient and he wants the nurse to find out how he is. The friend states, "Sue seems unusually worried about her dad, but she won't talk to me and I want to be able to help her." What is the best initial response the nurse should make? a. "You shouldn't be asking me to do this. I could be fined or even lose my job for disclosing this information." b. "Sorry, but I'm not able to give information about patients to the public—even when my best friend or a family mem- ber asks." c. "Because of the Health Insurance Portability and Account- ability Act, you shouldn't be asking for this information unless the patient has authorized you to receive it! This could get you in trouble!" d. "Why do you think Sue isn't talking about her worries?"

b. The nurse should immediately clarify what he or she can and cannot do. Since the primary reason for refusing to help is linked to the responsibility to protect patient privacy and confidentiality, the nurse should not begin by mention- ing the real penalties linked to abuses of privacy. Finally, it is appropriate to ask about Sue and her worries, but this should be done after the nurse clarifies what he or she is able to do.

A nurse is choosing a catheter to use to suction a patient's endotracheal tube via an open system. On which variable would the nurse base the size of the catheter to use? a. The age of the patient b. The size of the endotracheal tube c. The type of secretions to be suctioned d. The height and weight of the patient

b. The nurse would base the size of the suctioning catheter on the size of the endotracheal tube. The external diameter of the suction catheter should not exceed half of the internal diam- eter of the endotracheal tube. Larger catheters can contribute to trauma and hypoxemia.

A nurse caring for a patient's hemodialysis access documents the following: "5/10/15 0930 Arteriovenous fistula patent in right upper arm. Area is warm to touch and edematous. Patient denies pain and tenderness. Positive bruit and thrill noted." Which documented finding would the nurse report to the primary care provider? a. Positive bruit noted. b. Area is warm to touch and edematous. c. Patient denies pain and tenderness. d. Positive thrill noted.

b. The nurse would report a site that is warm and edematous as this could be a sign of a site infection. The thrill and bruit are normal findings caused by arterial blood flowing into the vein. If these are not present, the access may be cutting off. No report of pain is a normal finding.

A nurse is caring for an elderly male patient who is receiving fluids for dehydration. Which outcome for this patient is cor- rectly written? a. Offer the patient 60 mL fluid every 2 hours while awake. b. During the next 24-hour period, the patient's fluid intake will total at least 2,000 mL. c. Teach the patient the importance of drinking enough fluids to prevent dehydration by 1/15/15 d. At the next visit, 12/23/15, the patient will know that he should drink at least 3 liters of water per day.

b. The outcomes in a and c make the error of expressing the patient goal as a nursing intervention. Incorrect: "Offer the patient 60 mL fluid every 2 hours while awake." Correct: "The patient will drink 60 mL fluid every 2 hours while awake, beginning 1/3/15." The outcome in d makes the error of using verbs that are not observable and measurable. Verbs to be avoided when writing goals include "know," "under- stand," "learn," and "become aware."

A nurse asks a 25-year-old patient to make a list of 20 words that describe him. After 15 minutes, the patient listed the following: 25 years old, male, named Joe; then declared he couldn't think of anything else. The nurse documents that the patient has demonstrated: a. Lack of self-esteem b. Deficient self-knowledge c. Unrealistic self-expectation d. Inability to evaluate himself

b. The patient's inability to list more than three items about himself indicates deficient self-knowledge. There are not enough data provided to determine whether he lacks self- esteem, has unrealistic self-expectations, or is unable to evaluate himself.

A patient complains of abdominal pain that is difficult to localize. The nurse documents this as which type of pain? a. Cutaneous b. Visceral c. Superficial d. Somatic

b. The patient's pain would be categorized as visceral pain, which is poorly localized and can originate in body organs in the abdomen. Cutaneous pain (superficial pain) usually involves the skin or subcutaneous tissue. A paper cut that produces sharp pain with a burning sensation is an example of cutaneous pain. Deep somatic pain is diffuse or scattered and originates in tendons, ligaments, bones, blood vessels, and nerves. Strong pressure on a bone or damage to tissue that occurs with a sprain causes deep somatic pain.

A nurse is caring for a patient in the ICU who is being moni- tored for a possible cerebral aneurysm following a loss of consciousness in the emergency room. The nurse anticipates preparing the patient for ordered diagnostic tests. This nurse's knowledge of the diagnostic procedures for this condition reflects which aspect of nursing? a. The art of nursing b. The science of nursing c. The caring aspect of nursing d. The holistic approach to nursing

b. The science of nursing is the knowledge base for care that is provided. In contrast, the skilled application of that knowledge is the art of nursing. Providing holistic care to patients based on the science of nursing is considered the art of nursing.

A nurse is assessing the level of consciousness of a patient who sustained a head injury in a motor vehicle accident. The nurse notes that the patient appears drowsy most of the time but makes spontaneous movements. The nurse is able to wake the patient by gently shaking him and calling his name. What level of consciousness would the nurse document? a. Awake and alert b. Lethargic c. Stuporous d. Comatose

b. The stages of consciousness are: Awake and alert: fully awake; oriented to person, place, and time; responds to all stimuli, including verbal commands. Lethargic: appears drowsy or asleep most of the time but makes spontaneous movements; can be aroused by gentle shaking and saying patient's name. Stuporous: unconscious most of the time; has no spontaneous movement; must be shaken or shouted at to arouse; can make verbal responses, but these are less likely to be appropriate; responds to painful stimuli with purposeful movements. Comatose: cannot be aroused, even with use of painful stimuli; may have some reflex activity (such as gag reflex); if no reflexes present, is in a deep coma

A nurse is assessing a patient's eyes for extraocular move- ments. Which action correctly describes a step the nurse would take when performing this test? a. Ask the patient to sit about 3 feet away facing the nurse. b. Keep a penlight about 1 foot from the patient's face and move it slowly through the cardinal positions. c. Move a penlight in a circular motion in front of the patient's eyes. d. Ask the patient to cover one eye with a hand or index card.

b. The steps in testing for extraocular movement are: (1) Ask the patient to sit or stand about 2 feet away, facing the nurse, who is sitting or standing at eye level with the patient; (2) ask the patient to hold the head still and follow the movement of a forefinger or a penlight with the eyes; (3) keeping the finger or light about 1 foot from the patient's face, move it slowly through the cardinal position—up and down, left and right, diagonally up and down to the left, diagonally up and down to the right.

A nurse is using the Katz Index of Independence in Activi- ties of Daily Living to assess the mobility of an 80-year-old hospitalized female patient. During the patient interview the nurse documents the following patient data: "Patient bathes self completely but needs help with dressing. Patient toilets independently and is continent. Patient needs help moving from bed to chair. Patient follows directions and can feed self." Based on this data, which score would the patient receive on the Katz index? a. 2 b. 4 c. 5 d. 6

b. The total score for this patient is 4. On the Katz Index of Independence in ADLs, one point is awarded for independ- ence in each of the following activities: bathing, dressing, toileting, transferring, continence, and feeding.

After one nursing unit with an excellent safety record meets to review the findings of the audit, the nurse manager states, "We're doing well, but we can do better! Who's got an idea to foster increased patient well-being and satisfaction?" This is an example of leadership that values: a. Quality assurance b. Quality improvement c. Process evaluation d. Outcome evaluation

b. Unlike quality assurance, quality improvement is internally driven, focuses on patient care rather than organizational structure, focuses on processes rather than individuals, and has no end points. Its goal is improving quality rather than assur- ing quality. Answers c and d are types of quality-assurance programs.

A nurse who is caring for a patient diagnosed with HIV/AIDS incurs a needlestick injury when administering the patient's medications. What would be the priority action of the nurse following the exposure? a. Report the incident to the appropriate person and file an incident report. b. Wash the exposed area with warm water and soap. c. Consent to postexposure prophylaxis at appropriate time. d. Set up counseling sessions regarding safe practice to pro- tect self.

b. When a needlestick injury occurs, the nurse should wash the exposed area immediately with warm water and soap, report the incident to the appropriate person and complete an incident injury report, consent to and await the results of blood tests, consent to postexposure prophylaxis, and attend counseling sessions regarding safe practice to protect self and others.

After assessing a patient who is recovering from a stroke in a rehabilitation facility, a nurse interprets and analyzes the patient data. Which of the four basic conclusions has the nurse reached when identifying the need to collect more data to confirm a diagnosis of situational low self-esteem? a. No problem b. Possible problem c. Actual nursing diagnosis d. Clinical problem other than nursing diagnosis

b. When a possible problem exists, such as situational low self-esteem related to effects of stroke, the nurse must col- lect more data to confirm or disprove the suspected problem. The conclusion "no problem" means no nursing response is indicated. When an actual problem exists, the nurse begins planning, implementing, and evaluating care to prevent, reduce, or resolve the problem. A clinical problem other than nursing diagnosis requires that the nurse consult with the appropriate health care professional to work collabora- tively on the problem.

A nurse is caring for a 25-year-old male patient who is coma- tose following a head injury. The patient has several piercings in his ears and nose. The piercing in his nose appears to be new and is crusted and slightly inflamed. Which action would be appropriate when caring for this patient's piercings? a. Do not remove or wash the piercings without permission from the patient. b. Rinse the sites with warm water and remove crusts with a cotton swab. c. Wash the sites with alcohol and apply an antibiotic ointment. d. Remove the jewelry and allow the sites to heal over.

b. When providing care for piercings, the nurse should perform hand hygiene and put on gloves, then cleanse the site of all crusts and debris by rinsing the site with warm water, removing the crusts with a cotton swab. The nurse should then apply a dab of liquid medicated cleanser to the area, turn the jewelry back and forth to work the cleanser around the opening, rinse well, remove gloves, and perform hand hygiene. The nurse should not use alcohol, peroxide, or ointments at the site or remove the piercings unless it is absolutely necessary (e.g., when an MRI is ordered.)

A patient who has a large abdominal wound suddenly calls out for help because she feels as though something is falling out of her incision. Inspection reveals a gaping open wound with tissue bulging outward. In which order should the nurse perform the following interventions? Arrange from first to last. a. Notify the physician immediately of the situation. b. Cover the exposed tissue with sterile towels moistened with sterile NSS. c. Place the patient in the low Fowler's position.

c, b, a. Dehiscence and evisceration is a postoperative emer- gency that requires prompt surgical repair. The correct order of implementation by the nurse is to place the patient in the low Fowler's position, cover the exposed tissue with sterile towels moistened with sterile NSS, and notify the physician immediately of the situation.

A nurse is teaching patients of all ages in a hospital setting. Which examples demonstrate teaching that is appropriately based on the patient's developmental level? Select all that apply. a. The nurse plans long teaching sessions to discuss diet modi- fications for an older adult diagnosed with type 2 diabetes. b. The nurse recognizes that a female adolescent diagnosed with anorexia is still dependent on her parents and includes them in all teaching sessions. c. The nurse designs an exercise program for a sedentary older adult male patient based on the activities he prefers. d. The nurse includes an 8-year-old patient in the teaching plan for managing cystic fibrosis. e. The nurse demonstrates how to use an inhaler to an 11-year-old male patient and includes his mother in the session to reinforce the teaching. f. The nurse continues a teaching session on STIs for a sexu- ally active male adolescent despite his protest that "I've heard enough already!"

c, d, e. Successful teaching plans for older adults incorpo- rate extra time, short teaching sessions, accommodation for sensory deficits, and reduction of environmental distractions. Older adults also benefit from instruction that relates new information to familiar activities or information. School- aged children are capable of logical reasoning and should be included in the teaching-learning process whenever possible; they are also open to new learning experiences but need learn- ing to be reinforced by either a parent or health care provider as they become more involved with their friends and school activities. Teaching strategies designed for an adolescent patient should recognize the adolescent's need for independ- ence, as well as the need to establish a trusting relationship that demonstrates respect for the adolescent's opinions.

During an interaction with a patient diagnosed with epilepsy, a nurse notes that the patient is silent after she communicates the plan of care. What would be appropriate nurse responses in this situation? Select all that apply. a. Fill the silence with lighter conversation directed at the patient. b. Use the time to perform the care that is needed uninter- rupted. c. Discuss the silence with the patient to ascertain its mean- ing. d. Allow the patient time to think and explore inner thoughts. e. Determine if the patient's culture requires pauses between conversation. f. Arrange for a counselor to help the patient cope with emo- tional issues.

c, d, e. The nurse can use silence appropriately by taking the time to wait for the patient to initiate or to continue speak- ing. During periods of silence, the nurse should reflect on what has already been shared and observe the patient without having to concentrate simultaneously on the spoken word. In due time, the nurse might discuss the silence with the patient in order to understand its meaning. Also, the patient's culture may require longer pauses between verbal communication. Fear of silence sometimes leads to too much talking by the nurse, and excessive talking tends to place the focus on the nurse rather than on the patient. The nurse should not assume silence requires a consult with a counselor.

A school nurse is preparing a talk on safety issues for par- ents of school-aged children to present at a parent-teacher meeting. Which topics should the nurse include based on the age of the children? Select all that apply. a. Child-proofing the home b. Choosing a car seat c. Teaching pedestrian traffic safety d. Providing swimming lessons and water safety rules e. Discussing alcohol and drug consumption related to motor vehicle safety f. Teaching child how to "stop, drop, and roll"

c, d, f. Important safety topics for school-aged children include pedestrian traffic safety, water safety, and fire safety. Childproofing a home would be appropriate for parents of a toddler, choosing a car seat would be an appropriate topic for parents of an infant or toddler, and teaching drug and alcohol as it relates to motor vehicle safety would be a more appropri- ate topic for parents of adolescents.

A nurse performs nurse-initiated nursing actions when caring for patients in a skilled nursing facility. Which are examples of these types of interventions? Select all that apply. a. A nurse administers 500 mg of ciprofloxacin to a patient with pneumonia. b. A nurse consults with a psychiatrist for a patient who abuses pain killers. c. A nurse checks the skin of bedridden patients for skin breakdown. d. A nurse orders a kosher meal for an orthodox Jewish patient. e. A nurse records the I&O of a patient as prescribed by his physician. f. A nurse prepares a patient for minor surgery according to facility protocol.

c, d, f. Nurse-initiated interventions, or independent nursing actions, involve carrying out nurse-prescribed interventions resulting from their assessment of patient needs written on the nursing plan of care, as well as any other actions that nurses initiate without the direction or supervision of another health care professional. Protocols and standard orders empower the nurse to initiate actions that ordinarily require the order or supervision of a physician. Consulting with a psychiatrist is a collaborative intervention.

A nurse is documenting patient data in the medical record of a patient admitted to the hospital with a diagnosis of appen- dicitis. The physician has ordered 10 mg morphine IV every 3 to 4 hours. Which examples of documentation of care for this patient follows recommended guidelines? Select all that apply. a. 6/12/15 0945 Morphine 10 mg administered IV. Patient's response to pain appears to be exaggerated. M. Patrick, RN b. 6/12/15 0945 Morphine 10 mg administered IV. Patient seems to be comfortable. M. Patrick, RN c. 6/12/15 0945 30 minutes following administration of mor- phine 10 mg IV patient reports pain as 2 on a scale of 1 to 10. M. Patrick, RN d. 6/12/15 0945 Patient reports severe pain in right lower quadrant. M. Patrick, RN e. 6/12/15 0945 Morphine IV 10 mg will be administered to patient every 3 to 4 hours. M. Patrick, RN f. 6/12/15 0945 Patient states she does not want pain medica- tion despite return of pain. After discussing situation, patient agrees to medication administration.

c, d, f. The nurse should enter information in a complete, accurate, concise, current, and factual manner and indicate in each entry the date and both the time the entry was written and the time of pertinent observations and interventions. When charting, the nurse should avoid the use of stereotypes or derogatory terms as well as generalizations such as "seems comfortable today." The nurse should never document an intervention before carrying it out.

A nurse caring for adults in a physician's office notes that some patients age more rapidly that other patients of the same age. The nurse researches aging theories that attempt todescribe how and why aging occurs. Which statements apply to the immunity theory of aging? Select all that apply. a. Chemical reactions in the body produce damage to the DNA. b. Free radicals have adverse effects on adjacent molecules. c. Decrease in size and function of the thymus causes infections. d. There is much interest in the role of vitamin supplementation. e. Lifespan depends on a great extent to genetic factors. f. Organisms wear out from increased metabolic functioning.

c, d. The immunity theory of aging focuses on the func- tions of the immune system and states that the immune response declines steadily after younger adulthood as the thymus loses size and function, causing more infec- tions. There is much interest in vitamin supplements (such as vitamin E) to improve immune function. In the cross-linkage theory, cross-linkage is a chemical reac- tion that produces damage to the DNA and cell death. The free radical theory states that free radicals, formed during cellular metabolism, are molecules with separated high-energy electrons, which can have adverse effects on adjacent molecules. The genetic theory of aging holds that lifespan depends to a great extent on genetic fac- tors. According to the wear-and-tear theory, organisms wear out from increased metabolic functioning, and cells become exhausted from continual energy depletion from adapting to stressors (Eliopoulos, 2010).

A nurse is preparing a brochure to teach patients how to prevent urinary tract infections. Which teaching points would the nurse include? Select all that apply. a. Wear underwear with a synthetic crotch. b. Take baths rather than showers. c. Drink eight to ten 8-oz glasses of water per day. d. Drink a glass of water before and after intercourse and void afterwards. e. Limit caffeine-containing beverages. f. Drink 10 oz of cranberry or blueberry juice daily.

c, e, f. It is recommended that a healthy adult drink eight to ten 8-oz glasses of fluid daily, limit caffeine because it is irritating to the bladder mucosa, and drink 10 oz of cranberry or blueberry juice daily to help prevent bacteriuria. It is also recommended to wear underwear with a cotton crotch, take showers rather than baths, and drink two glasses of water before and after sexual intercourse and void immediately after intercourse.

Nursing is recognized increasingly as a profession based on which defining criteria? Select all that apply. a. Well defined body of general knowledge b. Interventions dependent upon the medical practice c. Recognized authority by a professional group d. Regulation by the medical industry e. Code of ethics f. Ongoing research

c, e, f. Nursing is recognized increasingly as a profession based on the following defining criteria: well-defined body of specific and unique knowledge, strong service orienta- tion, recognized authority by a professional group, code of ethics, professional organization that sets standards, ongoing research, and autonomy and self-regulation.

The nurse caring for patients in a long-term care facility knows that the highest level on Maslow's hierarchy of needs is self-actualization needs. Which statements accurately describe the achievement of self-actualization? Select all that apply. a. Humans are born with a fully developed sense of self- actualization. b. Self-actualization needs are met by depending on others for help. c. The self-actualization process continues throughout life. d. Loneliness and isolation occur when self-actualization needs are unmet. e. A person achieves self-actualization by focusing on prob- lems outside self. f. Self-actualization needs may be met by creatively solving problems.

c, e, f. Self-actualization, or reaching one's full potential, is a process that continues throughout life. A person achieves self-actualization by focusing on problems outside oneself and using creativity as a guideline for solving problems and pursuing interests. Humans are not born with a fully devel- oped sense of self-actualization, and self-actualization needs are not met specifically by depending on others for help. Loneliness and isolation are not always the result of unmet self-actualization needs.

A nurse is assessing a 78-year-old male patient for kinesthetic and visceral disturbances. Which techniques would the nurse use for this assessment? Select all that apply. a. The nurse asks the patient if he is bored, and if so, why. b. The nurse asks the patient if anything interferes with the functioning of his senses. c. The nurse asks the patient if he noticed any changes in the way he perceives his body. d. The nurse asks the patient if he has found it difficult to communicate verbally. e. The nurse notes if the patient withdraws from being touched. f. The nurse notes if the patient seems unsure of his body parts and/or position.

c, e, f. To assess for kinesthetic and visceral disturbances, the nurse would assess for perceived body changes inside and out, and changes in body parts or position. Asking if the patient is bored assesses stimulation, asking if anything interferes with his senses assesses reception, and asking about difficulty communicating assesses for transmission- perception-reaction.

The nurse practices using critical thinking indicators (CTIs) when caring for patients in the hospital setting. The best description of CTIs is: a. Evidence-based descriptions of behaviors that demonstrate the knowledge that promotes critical thinking in clinical practice b. Evidence-based descriptions of behaviors that demonstrate the knowledge and skills that promote critical thinking in clinical practice c. Evidence-based descriptions of behaviors that demonstrate the knowledge, characteristics, and skills that promote critical thinking in clinical practice d. Evidence-based descriptions of behaviors that demonstrate the knowledge, characteristics, standards, and skills that promote critical thinking in clinical practice

c, evidence-based descriptions of behaviors that demonstrate the knowledge, characteristics, and skills that promote critical thinking in clinical practice.

A nurse is ambulating a 48-year-old female patient for the first time following surgery for a knee replacement. Shortly after beginning to walk, the patient tells the nurse that she is dizzy and feels like she might fall. Place these nursing actions in the order in which the nurse should perform them to pro- tect the patient: a. Grasp the gait belt. b. Stay with the patient and call for help. c. Place feet wide apart with one foot in front. d. Gently slide patient down to the floor, protecting her head. e. Pull the weight of the patient backward against your body. f. Rock your pelvis out on the side of the patient.

c, f, a, e, d, b. If a patient being ambulated starts to fall, you should place your feet wide apart with one foot in front, rock your pelvis out on the side nearest the patient, grasp the gait belt, support the patient by pulling her weight backward against your body, gently slide her down your body toward the floor, protecting her head, and stay with the patient and call for help.

The nurse assesses the wound of a patient who cut himself on the upper thigh with a chain saw. The nurse then docu- ments the presence of biofilms in the wound. What is the effect of this condition on the wound? Select all that apply. a. Enhanced healing due to the presence of sugars and proteins b. Delayed healing due to dead tissue present in the wound c. Decreased effectiveness of antibiotics against the bacteria d. Impaired skin integrity due to overhydration of the cells of the wound e. Delayed healing due to cells dehydrating and dying f. Decreased effectiveness of the patient's normal immune process

c, f. Wound biofilms are the result of wound bacteria grow- ing in clumps, imbedded in a thick, self-made, protective, slimy barrier of sugars and proteins. This barrier contributes to decreased effectiveness of antibiotics against the bacteria (antibiotic resistance) and decreases the effectiveness of the normal immune response by the patient (Beitz, 2012). Necrosis (dead tissue) in the wound delays healing. Macera- tion or overhydration of cells related to urinary and fecal incontinence can lead to impaired skin integrity. Desiccation is the process of drying up, in which cells dehydrate and die in a dry environment.

A nurse formulates the following diagnosis for an older female patient in a long-term care facility: Disturbed Sensory Perception: Chronic Sensory Deprivation related to the effects of aging. The patient walked out the door unobserved and was lost for several hours. Which interventions would be most effective for this patient? Select all that apply. a. Ignore when the patient is confused or go along to prevent embarrassment. b. Reduce the number and type of stimuli in the patient's room. c. Orient the patient to time, place, and person frequently. d. Provide daily contact with children, community people, and pets. e. Decrease background or loud noises in the environment. f. Provide a radio and television in the patient's room.

c,d,f.Evenifwellmotivated,ignoringapatient'sconfusion to prevent embarrassment may be dangerous, as it was in this case in which the appropriate safety precautions were never implemented. Reducing the type of stimuli in the room and decreasing environmental noise is appropriate for a patient who is experiencing sensory overload. The other options are related to sensory deprivation and are appropriate for this patient.

A nurse is following the principles of medical asepsis when performing patient care in a hospital setting. Which nursing action performed by the nurse follows these recommended guidelines? a. The nurse carries the patients' soiled bed linens close to the body to prevent spreading microorganisms into the air. b. The nurse places soiled bed linens and hospital gowns on the floor when making the bed. c. The nurse moves the patient table away from the nurse's body when wiping it off after a meal. d. The nurse cleans the most soiled items in the patient's bathroom first and follows with the cleaner items.

c. According to the principles of medical asepsis, the nurse should move equipment away from the body when brush- ing, scrubbing, or dusting articles to prevent contaminated particles from settling on the hair, face, or uniform. The nurse should carry soiled items away from the body to prevent them from touching the clothing. The nurse should not put soiled items on the floor, as it is highly contaminated. The nurse should also clean the least soiled areas first and then move to the more soiled ones to prevent having the cleaner areas soiled by the dirtier areas.

A nurse is caring for a male patient with a severe hearing deficit who is able to read lips and use sign language. Which nursing intervention would be best to prevent sensory altera- tions for this patient? a. Turn the radio or television volume up very loud and close the door to his room. b. Prevent embarrassment and emotional discomfort as much as possible. c. Provide daily opportunity for him to participate in a social hour with six to eight people. d. Encourage daily participation in exercise and physical activity.

c. Although all the options listed are appropriate, providing daily opportunities for this patient to participate in a social hour builds on his strength of being able to lip-read and provides sufficient sensory stimulation to prevent sensory deprivation resulting from his hearing loss, thereby meeting his needs.

A nurse formulated the following nursing diagnosis for an 8-month-old infant: Disturbed Sensory Perception: Sensory Deprivation related to inadequate parenting. Since that diag- nosis was made, both parents have attended parenting classes. However, both parents work while the infant stays with her 86-year-old grandmother, who has reduced vision. The parents provide appropriate stimulation in the evening. At an evaluation conference at the age of 11 months, the infant lies on the floor sucking her thumb and rocking her body. Her facial expression is dull, and she vocalizes only in a low monotone ("uh-h-h"). Which statement accurately reflects evaluation about the child's sensory deprivation? a. The infant's parents lack motivation to provide necessary stimulation. b. The grandmother is unable to improve the infant's care. c. The infant's sensory deprivation is still severe. d. This is normal behavior for an 11-month-old infant.

c. Although the data show that the parents have been motivated to improve their parenting skills, it is clear from the data that the infant's sensory deprivation is still severe. The data suggest that the grandmother is not improving the infant's care, but there is nothing to suggest that she is unable to do so if shown how.

A school nurse is teaching parents how to foster a healthy development of self in their children. Which statement made by one of the parents needs to be followed up with further teaching? a. "I love my child so much I 'hug him to death' every day." b. "I think children need challenges, don't you?" c. "My husband and I both grew up in very restrictive fami- lies. We want our children to be free to do whatever they want." d. "My husband and I have different ideas about discipline, but we're talking this out because we know it's important for Johnny that we be consistent."

c. Each option with the exception of c correctly addresses some aspect of fostering healthy development in children. Because children need effective structure and development, giving them total freedom to do as they please may actually hinder their development.

A nurse uses a whirlpool to relax a patient following intense physical therapy to restore movement in her legs. What is a potent pain-blocking neuromodulator, released through relaxation techniques? a. Prostaglandins b. Substance P c. Endorphins d. Serotonin

c. Endorphins are produced at neural synapses at various points along the CNS pathway. They are powerful pain- blocking chemicals that have prolonged analgesic effects and produce euphoria. It is thought that endorphins are released through pain relief measures, such as relaxation techniques. Prostaglandins, substance P, and serotonin (a hormone that can act to stimulate smooth muscles, inhibit gastric secre- tion, and produce vasoconstriction) are neurotransmitters or substances that either excite or inhibit target nerve cells.

A nurse is preparing a sterile field using a packaged sterile drape for a confused patient who is scheduled for a surgical procedure. When setting up the field, the patient accidentally touches an instrument in the sterile field. What is the appro- priate nursing action in this situation? a. Ask another nurse to hold the hand of the patient and continue setting up the field. b. Remove the instrument that was touched by the patient and continue setting up the sterile field. c. Discard the supplies and prepare a new sterile field with another person holding the patient's hand. d. No action is necessary since the patient has touched his or her own sterile field.

c. If the patient touches a sterile field, the nurse should dis- card the supplies and prepare a new sterile field. If the patient is confused, the nurse should have someone assist by holding the patient's hand and reinforcing what is happening.

The nurse assesses patients to determine their risk for health care-associated infections. Which hospitalized patient is most at risk for developing this type of infection? a. A 60-year-old patient who smokes two packs of cigarettes daily b. A 40-year-old patient who has a white blood cell count of 6,000/mm3 c. A 65-year-old patient who has an indwelling urinary cath- eter in place d. A 60-year-old patient who is a vegetarian and slightly underweight

c. Indwelling urinary catheters have been implicated in most health care-associated infections. Cigarette smoking, a normal white blood cell count, and a vegetarian diet have not been implicated as risk factors for HAIs.

A nurse is caring for an alert, ambulatory, older resident in a long-term care facility who voids frequently and has difficulty making it to the bathroom in time. Which nursing intervention would be most helpful for this patient? a. Teach the patient that incontinence is a normal occurrence with aging. b. Ask the patient's family to purchase incontinence pads for the patient. c. Teach the patient to perform Kegel exercises at regular intervals daily. d. Insert an indwelling catheter to prevent skin breakdown.

c. Kegel exercises may help a patient regain control of the micturition process. Incontinence is not a normal consequence of aging. Using absorbent products may remove motivation from the patient and caregiver to seek evaluation and treatment of the incontinence; they should be used only after careful evaluation by a health care provider. An indwelling catheter is the last choice of treatment.

A nurse is interviewing Anthony, a 42-year-old patient who visits his internist for a blood pressure screening. Anthony tells the nurse that he is currently a sales associate but is con- sidering a different career. He states that he is "a little anxious about the process." According to Levinson, what phase of adult life is Anthony experiencing? a. Entering the adult world b. Settling down c. Midlife transition d. The pay-off years

c. Midlife transition (age 40-45) involves a reappraisal of one's goals and values. The established lifestyle may con- tinue, or the individual may choose to reorganize and change careers. This is an unsettled time, with the individual often anxious and fearful. The years of the middle to late 20s (age 22-28) are a time to build on previous decisions and choices and to try different careers and lifestyles. In the settling-down phase (age 30-40), the adult invests energy into the areas of life that are most personally important. The years from 45 to 65 are a time of maximum self-direction and self-approval.

A nurse is conducting an assessment of a patient's cranial nerves. The nurse asks the patient to raise the eyebrows, smile, and show the teeth to assess which cranial nerve? a. Olfactory b. Optic c. Facial d. Vagus

c. Motor function of the facial nerve (cranial nerve VII) is assessed by asking the patient to raise the eyebrow, smile, and show the teeth. The olfactory nerve (cranial nerve I) is tested by testing smell reception with various agents. The nurse tests the optic nerve (cranial nerve II) for acuity and visual fields and the vagus nerve (cranial nerve X) by asking the patient to swallow and speak, noting hoarseness.

A nurse is providing patient teaching regarding the use of negative-pressure wound therapy. Which explanation provides the most accurate information to the patient? a. The therapy is used to collect excess blood loss and pre- vent the formation of a scab. b. The therapy will prevent infection, ensuring that the wound heals with less scar tissue. c. The therapy provides a moist environment and stimulates blood flow to the wound. d. The therapy irrigates the wound to keep it free from debris and excess wound fluid.

c. Negative-pressure wound therapy (or topical negative pressure [TNP]) promotes wound healing and wound closure through the application of uniform negative pressure on the wound bed, reduction in bacteria in the wound, and the removal of excess wound fluid, while providing a moist wound healing environment. The negative pressure results in mechanical tension on the wound tissues, stimulating cell proliferation, blood flow to wounds, and the growth of new blood vessels. It is used to treat a variety of acute or chronic wounds, wounds with heavy drainage, wounds failing to heal, or healing slowly.

When helping a patient turn in bed, the nurse notices that his heels are reddened and plans to place him on precautions for skin breakdown. This is an example of what type of planning? a. Initial planning b. Standardized planning c. Ongoing planning d. Discharge planning

c. Ongoing planning is problem oriented and has as its pur- pose keeping the plan up to date as new actual or potential problems are identified. Initial planning addresses each problem listed in the prioritized nursing diagnoses and identifies appropriate patient goals and the related nursing care. Standardized care plans are prepared plans of care that identify the nursing diagnoses, outcomes, and related nurs- ing interventions common to a specific population or health problem. During discharge planning, the nurse uses teaching and counseling skills effectively to help the patient and family develop sufficient knowledge of the health problem and the therapeutic regimen to carry out necessary self-care behaviors competently at home.

A nurse examining a toddler in a pediatric office documents that the child is in the 90th percentile for height and weight and has blue eyes. These physical characteristics are primar- ily determined by: a. Socialization with caregivers b. Maternal nutrition during pregnancy c. Genetic information on chromosomes d. Meeting developmental tasks

c. Physical appearance and growth have a predetermined genetic base in inheritance patterns carried on the chromosomes.

Mr. James has an eye infection with a moderate amount of discharge. Which action would be most appropriate for the nurse to use when cleaning his eyes? a. Using hydrogen peroxide b. Wiping from the outer canthus to the inner canthus c. Positioning him on the same side as the eye to be cleansed d. Using only one cotton ball per eye

c. Positioning the patient on the same side as the involved eye discourages contamination of the other eye. Always cleanse from the inner canthus to the outer canthus to avoid forcing debris into the nasolacrimal duct. Water or normal saline should be used for cleansing the eye of any discharge, and one cotton ball should be used for each stroke.

When developing the plan of care for a patient with chronic pain, the nurse plans interventions based on the knowledge that chronic pain is most effectively relieved when analgesics are administered in what matter? a. On a PRN (as needed) basis b. Conservatively c. Around the clock (ATC) d. Intramuscularly

c. The PRN protocol is totally inadequate for patients experiencing chronic pain. ATC doses of analgesics are more effective, whereas conservative pain management for whatever reason may also prove ineffective. Intramuscular administration is not practical on a long-range basis for a patient with chronic pain.

A nurse is performing a psychological assessment of a 19-year-old male patient who has Down syndrome. The patient is mildly developmentally disabled with an intelli- gence quotient of 82. He told his nurse, "I'm a good helper. You see I can carry these trays because I'm so strong. But I'm not very smart, so I have just learned to help with the things I know how to do." The patient most likely has which of the following? a. Negative self-concept and low self-esteem b. Negative self-concept and high self-esteem c. Positive self-concept and fairly high self-esteem d. Positive self-concept and low self-esteem

c. The data point to the patient having a positive self-concept ("I'm a good helper") and fairly high self-esteem (realizes his strengths and limitations). The statement "But I'm not very smart" is accurate and is not an indication of a negative self- concept.

A mother of a 10-year-old daughter tells the nurse: "I feel incompetent as a parent and don't know how to discipline my daughter." What should be the nurse's first intervention when counseling this patient? a. Recommend that she discipline her daughter more strictly and consistently. b. Make a list of things her husband can do to give her more time and help her improve her parenting skills. c. Assist the mother to identify both what she believes is preventing her success and what she can do to improve. d. Explore with the mother what the daughter can do to improve her behavior and make the mother's role as a parent easier.

c. The first intervention priority with a mother who feels incompetent to parent a daughter is to assist the mother to identify what is preventing her from being an effective parent and then to explore solutions aimed at improving her parent- ing skills. The other interventions may prove helpful, but they do not directly address the mother's problem with her feelings of incompetence.

A female patient who is receiving chemotherapy for breast cancer tells the nurse, "The treatment for this cancer is worse than the disease itself. I'm not going to come for my therapy anymore." The nurse responds by using critical thinking skills to address this patient problem. Which action is the first step the nurse would take in this process? a. The nurse judges whether the patient database is adequate to address the problem. b. The nurse considers whether or not to suggest a counseling session for the patient. c. The nurse reassesses the patient and decides how best to intervene in her care. d. The nurse identifies several options for intervening in the patient's care and critiques the merit of each option.

c. The first step when thinking critically about a situation is to identify the purpose or goal of your thinking. Reassess- ing the patient helps to discipline thinking by directing all thoughts toward the goal. Once the problem is addressed, it is important for the nurse to judge the adequacy of the knowl- edge, identify potential problems, use helpful resources, and critique the decision.

A nurse uses the following classic elements of evaluation when caring for patients. Which item below places them in their correct sequence? (1) Interpreting and summarizing findings (2) Collecting data to determine whether evaluative criteria and standards are met (3) Documenting one's judgment (4) Terminating, continuing, or modifying the plan (5) Identifying evaluative criteria and standards (what one is looking for when evaluating, e.g., expected patient outcomes) a. 1,2,3,4,5 b. 3,2,1,4,5 c. 5,2,1,3,4 d. 2,3,1,4,5

c. The five classic elements of evaluation in order are (1) identifying evaluative criteria and standards (what you are looking for when you evaluate, e.g., expected patient out- comes), (2) collecting data to determine whether these criteria and standards are met, (3) interpreting and summarizing find- ings, (4) documenting your judgment, and (5) terminating, continuing, or modifying the plan.

The nurse is surprised to detect an elevated temperature (102°F) in a patient scheduled for surgery. The patient has been afebrile and shows no other signs of being febrile. What is the first thing the nurse should do? a. Inform the charge nurse. b. Inform the surgeon. c. Validate the finding. d. Document the finding.

c. The nurse should first validate the finding if it is unusual, deviates from normal, and is unsupported by other data. Should the initial recording prove to be in error, it would have been premature to notify the charge nurse or the surgeon. The nurse should be sure that all data recorded are accurate, thus all data should be validated before documentation if there are any doubts about accuracy.

A new RN is being oriented to a nursing unit that is currently understaffed and is told that the UAPs have been trained to obtain the initial nursing assessment. What is the best response of the RN? a. Allow the UAPs to do the admission assessment and report the findings to the RN. b. Do his or her own admission assessments but don't interfere with the practice if other professional RNs seem comfortable with the practice. c. Tell the charge nurse that he or she chooses not to delegate the admission assessment until further clarification is received from administration. d. Contact his or her labor representative and complain about this practice.

c. The nurse should not delegate this nursing admission assessment because only nurses can perform this intervention. The nurse should seek clarification for this policy from the nursing administration.

A patient has a fecal impaction. The nurse correctly adminis- ters an oil-retention enema by: a. Administering a large volume of solution (500-1,000 mL) b. Mixing milk and molasses in equal parts for an enema c. Instructing the patient to retain the enema for at least 30 minutes d. Administering the enema while the patient is sitting on the toilet

c. The patient should be instructed to retain the enema solution for at least 30 minutes or as indicated in the manufacturer's instructions. The usual amount of solution administered with a retention enema is 150 to 200 mL for an adult. The milk and molasses mixture is a carminative enema that helps to expel flatus. The patient should be instructed to lie on the left side of the bed as dictated by patient condition and comfort.

Which clinic patient is most likely to have annual breast examinations and mammograms based on the physical human dimension? a. Jane, whose her best friend had a benign breast lump removed b. Sarah, who lives in a low-income neighborhood c. Tricia, who has a family history of breast cancer d. Nancy, whose family encourages regular physical examinations

c. The physical dimension includes genetic inheritance, age, developmental level, race, and gender. These components strongly influence the person's health status and health prac- tices. A family history of breast cancer is a major risk factor.

A nurse is caring for a 26-year-old male patient who has been hospitalized for a spinal cord injury following a motor vehicle accident. Which action would the nurse perform when logrolling the patient to reposition him on his side? a. Have the patient extend his arms outward and cross his legs on top of a pillow. b. Stand at the side of the bed in which the patient will be turned while another nurse gently pushes the patient from the other side. c. Have the patient cross his arms on his chest and place a pillow between his knees. d. Place a cervical collar on the patient's neck and gently roll him to the other side of the bed.

c. The procedure for logrolling a patient is: (1) Have the patient cross the arms on the chest and place a pillow between the knees; (2) have two nurses stand on one side of the bed opposite the direction the patient will be turned with the third helper standing on the other side and if necessary, a fourth helper at the head of the bed to stabilize the neck; (3) fanfold or roll the drawsheet tightly against the patient and carefully slide the patient to the side of the bed toward the nurses; (4) have one helper move to the other side of the bed so that two nurses are on the side to which the patient is turning; (5) face the patient and have everyone move on a predetermined time, holding the draw- sheet taut to support the body, and turn the patient as a unit toward the two nurses.

A nurse is teaching a 50-year-old male patient how to care for his new ostomy appliance. Which teaching aid would be most appropriate to confirm that the patient has learned the information? a. Ask Me 3 b. Newest Vital Sign c. Teach-back tool d. TEACH acronym

c. The teach-back tool is a method of assessing literacy and confirming that the learner understands health informa- tion received from a health professional. The Ask Me 3 is a brief tool intended to promote understanding and improve communication between patients and their providers. The Newest Vital Sign (NVS) is a reliable screening tool to assess low health literacy, developed to improve communications between patients and providers. The TEACH acronym is used to maximize the effectiveness of patient teaching by tuning into the patient, editing patient information, acting on every teaching moment, clarifying often, and honoring the patient as a partner in the process.

A nurse is securing a patient's endotracheal tube with tape and observes that the tube depth changed during the retaping. Which action would be appropriate related to this incident? a. Instruct assistant to notify the primary care provider. b. Assess the patient's vital signs. c. Remove the tape, adjust the depth to ordered depth and reapply the tape. d. No action is required as depth will adjust automatically.

c. The tube depth should be maintained at the same level unless otherwise ordered by the physician. If the depth changes, the nurse should remove the tape, adjust the tube to ordered depth, and reapply the tape.

A nurse is about to bathe a female patient who has an intravenous access in place in her forearm. The patient's gown, which does not have snaps on the sleeves, needs to be removed prior to bathing. What should the nurse do? a. Temporarily disconnect the IV tubing at a point close to the patient and thread it through the gown sleeve. b. Cut the gown with scissors to allow arm movement. c. Thread the bag and tubing through the gown sleeve, keeping the line intact. d. Temporarily disconnect the tubing from the IV container, threading it through the gown.

c. Threading the bag and tubing through the gown sleeve keeps the system intact. Opening an IV line, even temporar- ily, causes a break in a sterile system and introduces the potential for infection. Cutting a gown is not an alternative except in an emergency.

Anurseisperformingdigitalremovalofstoolona74-yearold female patient with a fecal impaction. During the procedure the patient tells the nurse she is feeling dizzy and nauseated, and then she vomits. What should be the nurse's next action? a. Reassure the patient that this is a normal reaction to the procedure. b. Stop the procedure, prepare to administer CPR, and notify the physician. c. Stop the procedure, assess vital signs, and notify the physician. d. Stop the procedure, wait five minutes, and then resume the procedure.

c. When a patient complains of dizziness or lightheadedness and has nausea and vomiting during digital stool removal, the nurse should stop the procedure, assess heart rate and blood pressure, and notify the physician. The vagal nerve may have been stimulated.

A nurse develops a detailed plan of care for a 16-year-old female who is a new single mother of a premature infant. The plan includes collaborative care measures and home health care visits. When presented with the plan, the patient states, "We will be fine on our own. I don't need any more care." What would be the nurse's best response? a. "You know your personal situation better than I do, so I will respect your wishes." b. "If you don't accept these services, your baby's health will suffer." c. "Let's take a look at the plan again and see if we can adjust it to fit your needs." d. "I'm going to assign your case to a social worker who can explain the services better."

c. When a patient does not follow the plan of care despite your best efforts, it is time to reassess strategy. The first objective is to identify why the patient is not following the therapy. If the nurse determines, however, that the plan of care is adequate, the nurse must identify and remedy the factors contributing to the patient's noncompliance.

A visiting nurse performs a community assessment in an area of the city in which the nurse will be working. What is one element of a healthy community? a. Meets all the needs of its inhabitants b. Has mixed residential and industrial areas c. Offers access to health care services d. Has modern housing and condominiums

c.Ahealthycommunityoffersaccesstohealthcareservices to treat illness and to promote health. A healthy community does not usually meet all the needs of its residents, but should be able to help with health issues such as nutrition, education, recreation, safety, and zoning regulations to separate residential sections from industrial ones. The age of housing is irrelevant as long as residences are maintained properly according to code.

A nurse is administering a large-volume cleansing enema to a patient prior to surgery. Once the enema solution is intro- duced, the patient complains of severe cramping. What would be the appropriate nursing intervention in this situation? a. Elevate the head of the bed 30 degrees and reposition the rectal tube. b. Place the patient in a supine position and modify the amount of solution. c. Lower the solution container and check the temperature and flow rate. d. Remove the rectal tube and notify the primary care provider.

c.Ifthepatientcomplainsofseverecrampingwithintroduction of an enema solution, the nurse should lower the solution con- tainer and check the temperature and flow rate. If the solution is too cold or the flow rate too fast, severe cramping may occur.

Anurseiscaringforapatientwhohascomplicationsrelatedto type 2 diabetes mellitus. The nurse researches new procedures to care for foot ulcers when developing a plan of care for this patient. Which QSEN competency does this action represent? a. Patient-centered care b. Evidence-based practice c. Quality improvement d. Informatics

c.Qualityimprovementinvolvesroutinelyupdatingnurs- ing policies and procedures. Providing patient-centered care involves listening to the patient and demonstrating respect and compassion. Evidence-based practice is used when adhering to internal policies and standardized skills. The nurse is employ- ing informatics by using information and technology to com- municate, manage knowledge, and support decision making.

A nurse is documenting a blood pressure of 120/80 mm Hg. The nurse interprets the 120 to represent: a. The rhythmic distention of the arterial walls as a result of increased pressure due to surges of blood with ventricular contraction b. The lowest pressure present on arterial walls while the ventricles relax c. The highest pressure present on arterial walls while the ventricles contract d. The difference between the pressure on arterial walls with ventricular contraction and relaxation

c.Thesystolicpressureis120mmHg.Thediastolicpressureis 80 mm Hg, the lowest pressure present on arterial walls when the heart rests between beats. The difference between the systolic and diastolic pressures is called the pulse pressure. The rhythmic distention of the arterial walls as a result of increased pressure due to surges of blood with ventricular contraction is the pulse.

Prioritization: Place the following descriptions of the phases of Korotkoff sounds in order from Phase I to Phase V. a. Characterized by muffled or swishing sounds that may temporarily disappear; also known as the auscultatory gap b. Characterized by distinct, loud sounds as the blood flows relatively freely through an increasingly open artery c. The last sound heard before a period of continuous silence, known as the second diastolic pressure d. Characterized by the first appearance of faint but clear tap- ping sounds that gradually increase in intensity; known as the systolic pressure e. Characterized by a distinct, abrupt, muffling sound with a soft, blowing quality; considered to be the first diastolic pressure

d, a, b, e, c. Phase I is characterized by the first appearance of faint but clear tapping sounds that gradually increase in intensity; the first tapping sound is the systolic pressure. Phase II is characterized by muffled or swishing sounds, which may temporarily disappear, especially in hypertensive people; the disappearance of the sound during the latter part of phase I and during phase II is called the auscultatory gap. Phase III is characterized by distinct, loud sounds as the blood flows rela- tively freely through an increasingly open artery. Phase IV is characterized by a distinct, abrupt, muffling sound with a soft, blowing quality; in adults, the onset of this phase is considered to be the first diastolic pressure. Phase V is the last sound heard before a period of continuous silence; the pressure at which the last sound is heard is the second diastolic pressure.

A nurse is using the ISBARR physician reporting system to report the deteriorating mental status of Mr. Sanchez, a male patient who has been prescribed morphine via a patient- controlled analgesia pump (PCA) for pain related to pancre- atic cancer. Place the following nursing statements related to this call in the order in which they should be performed. a. "I am calling about Mr. Sanchez in Room 202 who is receiving morphine via a PCA pump for pancreatic cancer." b. "Mr. Sanchez has been difficult to arouse and his mental status has changed over the past 12 hours since using the pump." c. "You want me to discontinue the PCA pump until you see him tonight at patient rounds." d. "I am Rosa Clark, an RN working on the second floor of South Street Hospital." e. "Mr. Sanchez was admitted two days ago following a diagnosis of pancreatic cancer." f. "I think the dosage of morphine in Mr. Sanchez's PCA pump needs to be lowered."

d, a, e, b, f, c. The order for ISBARR is: identity/introduction, situation, background, assessment, recommendation, and read-back.

A nurse caring for patients in a long-term care facility is often required to collect urine specimens from patients for laboratory testing. Which techniques for urine collection are performed correctly? Select all that apply. a. The nurse catheterizes a patient to collect a sterile urine sample for routine urinalysis. b. The nurse collects a clean-catch urine specimen in the morning from a patient and stores it at room temperature until an afternoon pick-up. c. The nurse collects a sterile urine specimen from the collec- tion receptacle of a patient's indwelling catheter. d. The nurse collects about 3 mL of urine from a patient's indwelling catheter to send for a urine culture. e. The nurse collects a urine specimen from a patient with a urinary diversion by catheterizing the stoma. f. The nurse discards the first urine of the day when perform- ing a 24-hour urine specimen collection on a patient.

d, e, f. A urine culture requires about 3 mL of urine, whereas routine urinalysis requires at least 10 mL of urine. The preferred method of collecting a urine specimen from a urinary diversion is to catheterize the stoma. For a 24-hour urine specimen, the nurse should discard the first voiding, then collect all urine voided for the next 24 hours. A sterile urine specimen is not required for a routine urinalysis. Urine chemistry is altered after urine stands at room temperature for a long period of time. A specimen from the collecting recep- tacle (drainage bag) may not be fresh urine and could result in an inaccurate analysis.

A nurse is teaching a patient how to use a meter-dosed inhaler for her asthma. Which comments from the patient assure the nurse that the teaching has been effective? Select all that apply. a. "I will be careful not to shake up the canister before using it." b. "I will hold the canister upside-down when using it." c. "I will inhale the medication through my nose." d. "I will continue to inhale when the cold propellant is in my throat." e. "I will only inhale one spray with one breath." f. "I will activate the device while continuing to inhale."

d, e, f. Common mistakes that patients make when using MDIs include failing to shake the canister, holding the inhaler upside down, inhaling through the nose rather than the mouth, inhaling too rapidly, stopping the inhalation when the cold propellant is felt in the throat, failing to hold their breath after inhalation, and inhaling two sprays with one breath.

A nurse prepares to assist a patient with her newly created ile- ostomy. Which recommended patient teaching points would the nurse stress? Select all that apply. a. "When you inspect the stoma, it should be dark purple-blue." b. "The size of the stoma will stabilize within 2 weeks." c. "Keep the skin around the stoma site clean and moist." d. "The stool from an ileostomy is normally liquid." e. "You should eat dark green vegetables to control the odor of the stool." f. "You may have a tendency to develop food blockages."

d, e, f. Ileostomies normally have liquid, foul-smelling stool. The nurse should encourage the intake of dark green vegetables because they contain chlorophyll, which helps to deodorize the feces. Patients with ileostomies need to be aware they may experience a tendency to develop food blockages, especially when high-fiber foods are consumed. The stoma should be dark pink to red and moist. Stoma size usually stabilizes within 4 to 6 weeks, and the skin around the stoma site (peristomal area) should be kept clean and dry.

A nurse is scheduling tests for a patient who has been experiencing epigastric pain. The physician ordered the fol- lowing tests: (a) barium enema, (b) fecal occult blood test, (c) endoscopic studies, and (d) upper gastrointestinal series. Which is the correct order in which the tests would normally be performed? a. c, b, d, a b. d, c, a, b c. a, b, d, c d. b, a, d, c

d. A fecal occult blood test should be done first to detect gastrointestinal bleeding. Barium studies should be per- formed next to visualize gastrointestinal structures and reveal any inflammation, ulcers, tumors, strictures, or other lesions. A barium enema and routine radiography should precede an upper gastrointestinal series because retained barium from an upper gastrointestinal series could take several days to pass through the gastrointestinal tract and cloud anatomic detail on the barium enema studies. Noninvasive procedures usually take precedence over invasive procedures, such as endoscopic studies, when sufficient diagnostic data can be obtained from them.

A nurse is changing the stoma appliance on a patient's ileal conduit. Which characteristic of the stoma would alert the nurse that the patient is experiencing ischemia? a. The stoma is hard and dry. b. The stoma is a pale pink color. c. The stoma is swollen. d. The stoma is a purple-blue color.

d. A purple-blue stoma may reflect compromised circulation or ischemia. A pale stoma may indicate anemia. The stoma may be swollen at first, but that condition should subside with time. A normal stoma should be moist and dark pink to red in color.

While taking an adult patient's pulse, a nurse finds the rate to be 140 beats/min. What should the nurse do next? a. Check the pulse again in 2 hours. b. Check the blood pressure. c. Record the information. d. Report the rate to the primary care provider.

d. A rate of 140 beats/min in an adult is an abnormal pulse and should be reported to the primary care provider or the nurse in charge of the patient.

A nurse is discharging a patient from the hospital following a heart stent procedure. The patient asks to see and copy his medical record. What is the nurse's best response? a. "I'm sorry, but patients are not allowed to copy their medi- cal records." b. "I can make a copy of your record for you right now." c. "You can read your record while you are still a patient, but copying records is not permitted according to HIPAA rules." d. "I will need to check with our records department to get you a copy."

d. According to HIPAA, patients have a right to see and copy their health record; update their health record; get a list of the disclosures a health care institution has made independent of disclosures made for the purposes of treatment, payment, and health care operations; request a restriction on certain uses or disclosures; and choose how to receive health information. The nurse should be aware of agency policies regarding the patient's right to access and copy records.

The school nurse uses the principles and theories of growth and development when planning programs for high school students. According to Havighurst, what is a developmental task for this age group? a. Finding a congenial social group b. Developing a conscience, morality, and a scale of values c. Achieving personal independence d. Achieving a masculine or feminine gender role

d. According to Havighurst, it is the role of the adolescent to achieve a masculine or feminine gender role. Developing a conscience, morality, and a scale of values and achieving personal independence are roles of middle childhood. Finding a congenial social group is a role of young adulthood.

A nurse assesses an oral temperature for an adult patient. The patient's temperature is 37.5°C (99.5°F). What term would the nurse use to report this temperature? a. Febrile b. Hypothermia c. Hypertension d. Afebrile

d. Afebrile means without fever. This temperature is within the normal range for an adult. Fever (pyrexia) is an elevation of body temperature; a person with fever is said to be febrile. Hypothermia is a low body temperature and hyperthermia is a high body temperature.

A nurse is identifying outcomes for a patient who has a leg ulcer related to diabetes. An example of an affective outcome for this patient is: a. Within 1 day after teaching, the patient will list three ben- efits of continuing to apply moist compresses to leg ulcer after discharge. b. By 6/12/15, the patient will correctly demonstrate applica- tion of wet-to-dry dressing on leg ulcer. c. By 6/19/15, the patient's ulcer will begin to show signs of healing (e.g., size shrinks from 3′′ to 2.5′′). d. By 6/12/15, the patient will verbalize valuing health sufficiently to practice new health behaviors to prevent recurrence of leg ulcer.

d. Affective outcomes describe changes in patient values, beliefs, and attitudes. Cognitive outcomes (a) describe increases in patient knowledge or intellectual behaviors; psychomotor outcomes (b) describe the patient's achievement of new skills. c is an outcome describing a physical change in the patient.

A student nurse is organizing clinical responsibilities for an 84-year-old female patient who is diabetic and is being treated for foot ulcers. The patient tells the student, "I need to have my hair washed before I can do anything else today; I'm ashamed of the way I look." The patient's needs include diagnostic testing, dressing changes, meal planning and coun- seling, and assistance with hygiene. How would the nurse best prioritize this patient's care? a. Explain to the patient that there is not enough time to wash her hair today because of her busy schedule. b. Schedule the testing and meal planning first and complete hygiene as time permits. c. Perform the dressing changes first, schedule the testing and counseling, and complete hygiene last. d. Arrange to wash the patient's hair first, perform hygiene, and schedule diagnostic testing and counseling.

d. As long as time constraints permit, the most important priorities when scheduling nursing care are priorities iden- tified by the patient as being most important. In this case, washing the patient's hair and assisting with hygiene puts the patient first and sets the tone for an effective nurse- patient partnership.

When discussing emergency preparedness with a group of first responders, what information would be important to include about preparation for a terrorist attack? a. Posttraumatic stress disorders can be expected in most survivors of a terrorist attack. b. The FDA has collaborated with drug companies to create stockpiles of emergency drugs. c. Even small doses of radiation result in bone marrow depression and cancer. d. Blast lung injury is a serious consequence following deto- nation of an explosive device.

d. Blast lung injury is a recognized consequence following exposure to an explosive device. The CDC is the federal agency that has collaborated with the pharmaceutical com- panies to stockpile drugs for an emergency. A high dose of radiation exposure can result in bone marrow depression and cancer. Most survivors of a terrorist event will experience stress and some (possibly one-third of survivors) may exhibit posttraumatic stress disorder.

When providing health promotion classes, a nurse uses con- cepts from models of health. What do both the health-illness continuum and the high-level wellness models demonstrate? a. Illness as a fixed point in time b. The importance of family c. Wellness as a passive state d. Health as a constantly changing state

d. Both these models view health as a dynamic (constantly changing state).

The nurse has opened the sterile supplies and put on two ster- ile gloves to complete a sterile dressing change, a procedure that requires surgical asepsis. The nurse must: a. Keep splashes on the sterile field to a minimum. b. Cover the nose and mouth with gloved hands if a sneeze is imminent. c. Use forceps soaked in a disinfectant. d. Consider the outer 1 inch of the sterile field as contaminated.

d. Considering the outer inch of a sterile field as contami- nated is a principle of surgical asepsis. Moisture such as from splashes contaminates the sterile field, and sneezing would contaminate the sterile gloves. Forceps soaked in disinfectant are not considered sterile.

A nurse is writing nursing diagnoses for patients in a psychia- trist's office. Which nursing diagnoses are correctly written as two-part nursing diagnoses? (1) Ineffective Coping related to inability to maintain marriage (2) Defensive Coping related to loss of job and economic security (3) Altered Thought Processes related to panic state (4) Decisional Conflict related to placement of parent in a long-term care facility a. (1) and (2) b. (3) and (4) c. (1), (2), and (3) d. All of the above

d. Each of the four diagnoses is a correctly written two-part diagnostic statement that includes the problem or diagnostic label and the etiology or cause.

A patient is in the late stages of AIDS, which is now affect- ing his brain as well other major organ systems. The patient confides to the nurse that he feels terribly alone because most of his friends are afraid to visit. The nurse determines that the least likely underlying etiology for his sensory problems would be: a. Stimulation b. Reception c. Transmission-perception-reaction d. Emotional responses

d. Emotional responses are an effect of sensory deprivation, and although they may be occurring with this patient, they are not the underlying etiology for his condition. This patient is receiving decreased environmental stimuli (a) (e.g., from his friends), is more than likely experiencing problems with reception because of major organ involvement (b), and his impaired brain function will impair impulse transmission- perception- reaction (c).

Which nurse who was influential in the development of nurs- ing in North America is regarded as the founder of American nursing? a. Clara Barton b. Lillian Wald c. Lavinia Dock d. Florence Nightingale

d. Florence Nightingale elevated the status of nursing to a respected occupation, improved the quality of nursing care, and founded modern nursing education. Clara Barton estab- lished the Red Cross in the United States in 1882. Lillian Wald is the founder of public health nursing. Lavinia Dock was a nursing leader and women's rights activist instrumental in womens' right to vote.

A nurse is teaching a patient with frequent constipation how to implement a bowel-training program. What is a recom- mended teaching point? a. Using a diet that is low in bulk b. Decreasing fluid intake to 1,000 mL c. Administering an enema once a day to stimulate peristalsis d. Allowing ample time for evacuation

d. For a bowel-training program to be effective, the patient must have ample time for evacuation (usually 20-30 min- utes). Fluid intake is increased to 2,500 to 3,000 mL, food high in bulk is recommended as part of the program, and a daily enema is not administered in a bowel-training program. A cathartic suppository may be used 30 minutes before the patient's usual defecation time to stimulate peristalsis.

A nurse working in a busy emergency department is caring for a teenage patient who presents with a burning pain in his mouth, edema of the lips, vomiting, and hemoptysis. The teen admits that he was playing a dare game with friends and was forced to swallow a drain opener preparation. What would be the nurse's priority intervention? a. Induce vomiting and call the primary care provider. b. Perform stomach lavage and call the poison control center. c. Give activated charcoal orally and call the physician. d. Dilute the poison with milk and call the primary care provider.

d. For the ingestion of drain opener, the nurse should never induce vomiting; instead, the poison should be diluted with milk or water and the primary care provider should be called. For vitamin preparations, stomach lavage is used to remove undigested pills and for acetaminophen poisoning, activated charcoal may be used.

A nurse asks a 25-year-old patient to make a list of 20 words that describe him. After 15 minutes, the patient listed the following: 25 years old, male, named Joe; then declared he couldn't think of anything else. A nurse asks the same patient in question 2 to list facts, traits, or qualities that he would like to be descriptive of himself. The patient quickly lists 25 traits, all of which are characteris- tic of a successful man. When asked if he knows anyone like this, he replies, "My father." This discrepancy between the patient's description of himself as he is and as he would like to be indicates: a. Negative self-concept b. Joe's modesty (lack of conceit) c. Body image disturbance d. Low self-esteem

d. Low self-esteem is characterized by great discrepancy between the ideal and real selves. There are no data given here that suggest that Joe has either a negative self-concept or a body image disturbance. The data do indicate something more serious than modesty.

Which of the following nursing diagnoses would be appropri- ate for many middle adults? a. Risk for Imbalanced Nutrition: Less Than Body Require- ments b. Delayed Growth and Development c. Self-Care Deficit d. Caregiver Role Strain

d. Many middle adults help care for aging parents and have concerns about their own health and ability to continue to care for an older family member. Caregivers often face 24-hour care responsibilities for extended periods of time, which creates physical and emotional problems for the caregiver.

A nurse is counseling a 60-year-old female patient who refuses to look at or care for a new colostomy. She tells the nurse, "I don't care what I look like anymore, I don't even feel like washing my hair, let alone changing this bag." The nurse diagnoses Altered Health Maintenance. This is an example of what type of problem? a. Collaborative problem b. Interdisciplinary problem c. Medical problem d. Nursing problem

d. Nursing Problem, because it describes a problem that can be treated by nurses within the scope of independent nursing practice. Collaborative and interdisciplinary problems require a teamwork approach with other health care professionals to resolve the problem. A medical problem is a traumatic or disease condition validated by medical diagnostic studies.

A nurse observes a marked inflammation of the gums, along with recession and bleeding of the gums, and documents this observation using which term? a. Glossitis b. Caries c. Cheilosis d. Periodontitis

d. Periodontitis is a marked inflammation of the gums, whereas caries refers to the presence of tooth decay. Cheilosis is ulceration of the lips, and glossitis is an inflammation of the tongue.

The nurse caring for patients in a hospital setting institutes CDC standard precaution recommendations for which cat- egory of patients? a. Only patients with diagnosed infections b. Only patients with visible blood, body fluids, or sweat c. Only patients with nonintact skin d. All patients receiving care in hospitals

d. Standard precautions apply to all patients receiving care in hospitals, regardless of their diagnosis or possible infection status. These recommendations include blood; all body fluids, secretions, and excretions except sweat; nonintact skin; and mucous membranes.

A nurse working in a long-term care facility bases patient care on five caring processes: knowing, being with, doing for, enabling, and maintaining belief. This approach to patient care best describes the theory of which theorist? a. Travelbee b. Watson c. Benner d. Swanson

d. Swanson (1991) identifies five caring processes and defines caring as "a nurturing way of relating to a valued other toward whom one feels a personal sense of commit- ment and responsibility. Travelbee (1971), an early nurse theorist, developed the Human-to-Human Relationship Model and defined nursing as an interpersonal process whereby the professional nurse practitioner assists an indi- vidual, family, or community to prevent or cope with the experience of illness and suffering, and if necessary to find meaning in these experiences. Benner and Wrubel (1989) write that caring is a basic way of being in the world and that caring is central to human expertise, curing, and heal- ing. Watson's theory is based on the belief that all humans are to be valued, cared for, respected, nurtured, understood, and assisted.

A 36-year-old female who was divorced 5 years earlier enters the emergency department with severe burns and cuts on her face after an auto accident in a car driven by her fiancé of 3 months. Three weeks later, her fiancé has not yet contacted her. The patient states that he is very busy and she is too tired to have visitors anyway. The patient frequently lies with her eyes closed and head turned away. These data suggest that: a. There is no disturbance in self-concept. b. This patient has ego strength and high self-esteem but may have a disturbance of body image. c. The area of self-esteem has very low priority at this time and should be ignored until much later. d. It is probable that there are disturbances in self-esteem and body image.

d. The traumatic nature of this patient's injuries, her fiancé's failure to contact her, and her withdrawal response all point to potential problems with both body image and self-esteem. It is not true that self-esteem needs are of low priorit

An older resident who is disoriented likes to wander the halls of his long-term care facility. Which action would be most appropriate for the nurse to use as an alternative to restraints? a. Sitting him in a geriatric chair near the nurses' station b. Using the sheets to secure him snugly in his bed c. Keeping the bed in the high position d. Identifying his door with his picture and a balloon

d. This allows the resident to be on the move and be more likely to find his room when he wants to return. The alterna- tive would be to not allow him to wander. Many facilities use this kind of approach. Identifying his door with his picture and a balloon may work as an alternative to restraints. Using the geriatric chair and sheets are forms of physical restraint. Leaving the bed in the high position is a safety risk and would probably result in a fall.

A nurse is counseling a 19-year-old male athlete who had his right leg amputated below the knee following a motorcycle accident. During the rehabilitation process, the patient refuses to eat or get up to ambulate on his own. He says to the nurse, "What's the point. My life is over now and I'll never be the football player I dreamed of becoming." What is the nurse counselor's best response to this patient? a. "You're young and have your whole life ahead of you. You should focus on your rehabilitation and make something of your life." b. "I understand how you must feel. I wanted to be a famous singer, but I wasn't born with the talent to be successful at it." c. "You should concentrate on other sports that you could play even with a prosthesis." d. "I understand this is difficult for you. Would you like to talk about it now or would you prefer me to make a refer- ral to someone else?"

d. This answer communicates respect and sensitivity to the patient's needs and offers an opportunity to discuss his feel- ings with the nurse or another health care professional. The other answers do not allow the patient to express his feelings and receive the counseling he needs.

A patient in an intensive care burn unit for 1 week is in pain much of the time and has his face and both arms heavily bandaged. His wife visits every evening for 15 minutes at 6, 7, and 8 pm. A heart monitor beeps for a patient on one side, and another patient moans frequently. Assessment would suggest that that the patient probably is experiencing: a. Sufficient sensory stimulation b. Deficient sensory stimulation c. Excessive sensory stimulation d. Both sensory deprivation and overload

d. This patient's bandages may result in deficient sensory stimulation (sensory deprivation), and the monitors and other sounds in the intensive care burn unit may cause a sensory overload. All other options are incomplete responses.

A nurse is caring for a patient who is diagnosed with conges- tive heart failure. Which statement below is not an example of a well-stated nursing intervention? a. Offer patient 60 mL water or juice (prefers orange or cranberry juice) every 2 hours while awake for a total minimum PO intake of 500 mL. b. Teach patient the necessity of carefully monitoring fluid intake and output; remind patient each shift to mark off fluid intake on record at bedside. c. Walk with patient to bathroom for toileting every 2 hours (on even hours) while patient is awake. d. Manage patient's pain.

d. This statement lacks sufficient detail to effectively guide nursing intervention. The set of nursing interventions written to assist a patient to meet an outcome must be comprehen- sive. Comprehensive nursing interventions specify what observations (assessments) need to be made and how often, what nursing interventions need to be done and when they must be done, and what teaching, counseling, and advocacy needs patients and families may have.

A nurse who created a sterile field for a patient is adding a sterile solution to the field. What is an appropriate action when performing this task? a. Place the bottle cap on the table with the edges down. b. Hold the bottle inside the edge of the sterile field. c. Hold the bottle with the label side opposite the palm of the hand. d. Pour the solution from a height of 4 to 6 inches (10 to 15 cm).

d. To add a sterile solution to a sterile field, the nurse would open the solution container according to directions and place the cap on the table away from the field with the edges up. The nurse would then hold the bottle outside the edge of the sterile field with the label side facing the palm of the hand and prepare to pour from a height of 4 to 6 inches (10 to 15 cm).

A nurse is developing a plan of care related to prevention of pressure ulcers for residents in a long-term care facility. Which action would be a priority in preventing a patient from developing a pressure ulcer? a. Keeping the head of the bed elevated as often as possible b. Massaging over bony prominences c. Repositioning bed-bound patients every 4 hours d. Using a mild cleansing agent when cleansing the skin

d. To prevent pressure ulcers, the nurse should cleanse the skin routinely and whenever any soiling occurs by using a mild cleansing agent with minimal friction, and avoiding hot water. The nurse should minimize the effects of shearing force by limiting the amount of time the head of the bed is elevated, when possible. Bony prominences should not be massaged, and bed-bound patients should be repositioned every 2 hours.

When you ask an experienced nurse why it is necessary to change the patient's bed every day, the nurse says, "I guess we have just always done it that way." This answer is an example of what type of knowledge? a. Instinctive knowledge b. Scientific knowledge c. Authoritative knowledge d. Traditional knowledge

d. Traditional knowledge is the part of nursing practice passed down from generation to generation, often with- out research data to support it. Scientific knowledge is that knowledge obtained through the scientific method (implying thorough research). Authoritative knowledge comes from an expert and is accepted as truth based on the person's perceived expertise. Instinct is not a source of knowledge.

When assessing a patient's breath sounds, the nurse hears a high-pitched continuous sound. What type of breath sound would the nurse document? a. Rhonchi b. Crackles c. Stridor d. Wheezes

d. Wheezes are musical or squeaking high-pitched, continu- ous sounds heard as air passes through narrowed airways. Rhonchi are low-pitched, continuous sounds with a snoring quality that occur when air passes through secretions. Crack- les are bubbling, cracking or popping, low- to high-pitched, discontinuous sounds that occur when air passes through fluid in the airways. Stridor is a harsh, loud, high-pitched sound due to narrowing of the upper airway.

A nurse providing care of a patient's chest drainage system observes that the chest tube has become separated from the drainage device. What would be the first action that should be taken by the nurse in this situation? a. Notify the physician. b. Apply an occlusive dressing on the site. c. Assess the patient for signs of respiratory distress. d. Put on gloves and insert the chest tube in a bottle of sterile saline.

d. When a chest tube becomes separated from the drain- age device, the nurse should first put on gloves, open a sterile bottle of normal saline or water, and insert the chest tube into the bottle without contaminating the chest tube. This creates a water seal until a new drainage unit can be attached. Then the nurse should assess vital signs and notify the physician.

An older female patient has a severe visual deficit related to glaucoma. Which nursing action would be appropriate when providing care for this patient? a. Assist the patient to ambulate by walking slightly behind the person and grasping the patient's arm. b. Concentrate on the sense of sight and limit diversions that involve other senses. c. Stay outside of the patient's field of vision when perform- ing personal hygiene for the patient. d. Indicate to the patient when the conversation has ended and when the nurse is leaving the room.

d. When caring for a patient who has a visual deficit, the nurse should indicate when the conversation is over and when he or she is leaving the room, assist with ambulation by walk- ing slightly ahead of the person and allowing her to grasp the nurse's arm, provide diversions using other senses, and stay in the person's field of vision if she has partial or reduced peripheral vision.

A nurse is suctioning the nasopharyngeal airway of a patient to maintain a patent airway. For which condition would the nurse anticipate the need for a nasal trumpet? a. The patient vomits during suctioning. b. The secretions appear to be stomach contents. c. The catheter touches an unsterile surface. d. Epistaxis is noted with continued suctioning.

d. When epistaxis is noted with continued suctioning, the nurse should notify the physician and anticipate the need for a nasal trumpet. The nasal trumpet will protect the nasal mucosa from further trauma related to suctioning.

A nurse works with families in crisis at a community mental health care facility. What is the best broad definition of a family? a. A father, a mother, and children b. A group whose members are biologically related c. A unit that includes aunts, uncles, and cousins d. A group of people who live together and depend on each other for support

d.Althoughalltheresponsesmaybetrue,thebestdefinitionis a group of people who live together and depend on each other for physical, emotional, or financial support.

Shuba and Raul are a couple in their late seventies. According to Duvall, which developmental task is appropriate for this older adult family? a. Maintain a supportive home base b. Strengthen marital relationships c. Cope with loss of energy and privacy d. Adjust to retirement

d.Thedevelopmentaltasksofthefamilywitholderadultsare to adjust to retirement and possibly to adjust to the loss of a spouse and loss of independent living. Maintaining a support- ive home base and strengthening marital relationships are tasks of the family with adolescents and young adults. Coping with loss of energy and privacy is a task of the family with children.


Related study sets

Social Media and Youth Development

View Set

Chapter 11:Corporate Governance, Social responsibility, and Ethics

View Set

Module 7 factors That Affect Earth's Weather

View Set

Unit 4 Regionalism & Naturalism (1870-1910)

View Set

Chapter 7: Managerial Planning and Goal Setting

View Set

Introduction to Criminal Justice Chapter 10 (CJC 101)

View Set